Olimpiada Matematica Española 1993-2010

March 29, 2018 | Author: Vicente Vázquez | Category: Triangle, Rectangle, Angle, Euclid, Classical Geometry


Comments



Description

OLIMPIADA MATEMÁTICA ESPAÑOLAFase Nacional 1993 (Madrid) Primera sesión 1.- En una reunión hay 201 personas de 5 nacionalidades diferentes. Se sabe que, en cada grupo de 6, al menos dos tienen la misma edad. Demostrar que hay al menos 5 personas del mismo país, de la misma edad y del mismo sexo. 2.- Escrito el triángulo aritmético: 0 1 2 3 4 ........ 1991 1992 1993 1 3 5 7 ...... ........ ........ 3983 3985 4 8 12 ...... ...... ........ ........ ........ 7968 ........................................................................................... donde cada número es la suma de los dos que tiene encima (cada fila tiene un número menos y en la última sólo hay un número). Razonar que el último número es múltiplo de 1993. 3.- Justificar razonadamente que, en cualquier triángulo, el diámetro de la circunferencia inscrita no es mayor que el radio de la circunferencia circunscrita. Segunda sesión 4.- Demostrar que para todo número primo p distinto de 2 y de 5, existen infinitos múltiplos de p de la forma 1111......1 (escrito sólo con unos). 5.- Se dan 16 puntos formando una cuadrícula como en la figura: De ellos se han destacado A y D. Se pide fijar de todos los modos posibles otros dos puntos B y C con la condición de que las seis distancias determinadas por los cuatro puntos sean distintas. En ese conjunto de cuaternas, estudiar: a) Cuántas figuras de 4 puntos existen con las condiciones del enunciado. b) Cuántas de ellas son geométricamente distintas, es decir, no deducibles unas de otras por transformaciones de igualdad. c) Si cada punto se designa por un par de enteros (X i , Y i ), razonar que la suma: |X i - X j | + | Y i - Y j | extendida a los seis pares AB, AC, AD, BC, BD, CD es constante. 6.- Una máquina de juego de un casino tiene una pantalla en la que se ofrece un esquema como el de la figura. Para comenzar el juego aparece una bola en el punto S. A cada impulso que recibe del jugador, esa bola se mueve hasta una de las letras inmediatas con la misma probabilidad para cada una de ellas. La partida termina al ocurrir el primero de los dos hechos siguientes: a) La bola vuelve a S y entonces el jugador pierde. b) La bola llega a G y entonces el jugador gana. Se pide la probabilidad de que el jugador gane y la duración media de las partidas. Problemas de la XXIX Olimpiada 1.- En una reunión hay 201 personas de 5 nacionalidades diferentes. Se sabe que, en cada grupo de 6, al menos dos tienen la misma edad. Demostrar que hay al menos 5 personas del mismo país, de la misma edad y del mismo sexo. Solución (Mª Gaspar Alonso-Vega) Si en cada grupo de 6 personas, 2 son de la misma edad, sólo puede haber 5 edades diferentes, ya que, si hubiese 6 edades diferentes, eligiendo una persona de cada edad tendríamos 6 personas de edades distintas contra la hipótesis. Como 200 = 2 · 100 + 1⇒ al menos hay 101 personas del mismo sexo. 101 = 5 · 20 + 1 ⇒ al menos hay 21 personas de la misma edad y sexo. 21 = 4 · 5 + 1 ⇒ al menos hay 5 personas de la misma nacionalidad, edad y sexo. 2.- Escrito el triángulo aritmético: 0 1 2 3 4 ............. 1991 1992 1993 1 3 5 7 ............................ 3983 3985 4 8 12 ........................................ 7968 ......................................................................... donde cada número es la suma de los dos que tiene encima (cada fila tiene un número menos y en la última sólo hay un número). Razonar que el último número es múltiplo de 1993. Solución. Si representamos los elementos de la primera fila por a 0 , a 1 , a 2 , ........ los elementos de la segunda serán: a 0 + a 1 , a 1 + a 2 , a 2 + a 3 , .............. los de la tercera serán : a 0 + 2a 1 + a 1 , a 1 + 2a 2 + a 3 , .............. para la cuarta : a 0 + 3a 1 + 3a 1 + a 1 , a 1 + 3a 2 + 3a 3 + a 4 ,............ Supongamos que los dos primeros elementos b p,0 y b p,1 de la fila p-ésima son: b p a p a p p a p p , ....... 0 0 1 1 1 0 1 1 1 1 = −       + −       + + − −       − ; b p a p a p p a p p , ....... 1 1 2 1 0 1 1 1 1 = −       + −       + + − −       entonces, el primer elemento de la fila siguiente será : b p a p a p p a p p + =       +       + +       1 0 0 1 0 1 , ....... (*) en nuestro caso la primera fila tiene 1994 elementos, la segunda 1993, ... y la última corresponde a p + 1 = 1994 y su único elemento será b 1994 1993 0 0 1993 1 1 1993 1993 1993 =       +       + +       • • ....... • Al ser 1993 primo, 1993 k       es múltiplo de 1993 para todo k menor que 1993 y por tanto b 1993 es múltiplo de 1993. 3.- Justificar razonadamente que, en cualquier triángulo, el diámetro de la circunferencia inscrita no es mayor que el radio de la circunferencia circunscrita. Solución (F. Bellot) La desigualdad propuesta, R - 2r ≥ 0 es una consecuencia del teorema de Euler. “Si I, O son el incentro y el circuncentro de un triángulo, r y R los radios de las circunferencias inscrita y circunscritas, se verifica: IO 2 = R 2 - 2Rr”. Entonces IO 2 = R (R - 2r) ≥ 0 ⇒ R - 2r ≥ 0. 4.- Demostrar que para todo número primo p distinto de 2 y de 5, existen infinitos múltiplos de p de la forma 1111......1 (escrito sólo con unos). Solución (Alvaro Begué Aguado) Veamos primero que p tiene infinitos múltiplos de la forma 999...9. Consideremos la sucesión: 9, 99, 999, ......,999...9 (el último tiene n nueves). Entonces se tiene: 9 = 10 - 1; 99 = 10 2 - 1; 999 = 10 3 - 1;.......999..9 = 10 n - 1 en la sucesión hay infinitos términos de la forma 10 p-1 - 1 con p ≠ 2, p ≠ 5 y p primo. Puesto que, por el teorema de Fermat: 10 p-1 - 1 ≡ 1 (mód p) si p ≠ 2, p ≠ 5 la afirmación queda demostrada. Finalmente 999...9 = 9 · 111...1 entonces si p es primo con 9 (p ≠ 3), p divide al producto, es primo con 9 luego divide a 111...1. Queda el caso p = 3 que es evidente ya que los infinitos números: 111; 111111, .......... son múltiplos de tres. 5.- Se dan 16 puntos formando una cuadrícula como en la figura: De ellos se han destacado A y D. Se pide fijar de todos los modos posibles otros dos puntos B y C con la condición de que las seis distancias determinadas por los cuatro puntos sean distintas. En ese conjunto de cuaternas, estudiar: a) Cuántas figuras de 4 puntos existen con las condiciones del enunciado. b) Cuántas de ellas son geométricamente distintas, es decir, no deducibles unas de otras por transformaciones de igualdad. c) Si cada punto se designa por un par de enteros (X i , Y i ), razonar que la suma: |X i - X j | + | Y i - Y j | extendida a los seis pares AB, AC, AD, BC, BD, CD es constante. A D Solución El problema admite dos ejes de simetría coincidentes con las diagonales del cuadrado. Clasificaremos las soluciones posibles por la posición del punto B respecto del vértice A. Usaremos coordenadas enteras con origen en A. Las tres posiciones “fundamentales” (no deducibles unas de otras por las simetrías anteriores) son aquellas en las que B está en los puntos de coordenadas (0,1); (0,2) y (1,1) para cada una de ellas dibujamos us esquema con las posibles posiciones del punto C. Las posiciones “prohibidas” se dibujan en negro, la posición de B en gris y las de C i en blanco. Un criterio general para prohibir ubicaciones es localizar aquellos puntos que estan en la “mediatriz” de dos puntos ya situados. Como A y D son dados y fijos, la diagonal principal siempre contiene puntos “prohibidos” El esquema de la izquierda contiene 4 posiciones “originales” y cada una de ellas genera otras cuatro por aplicación de las dos simetrías, en total 16. El esquema del centro contiene 3 posiciones “originales” y cada una de ellas genera otras cuatro por aplicación de las dos simetrías, en total 12. El esquema de la derecha contiene 1 posición “original” que genera otras cuatro por aplicación de las dos simetrías, en total 4. Por tanto existen 32 posiciones posibles y 8 “originales”, esto contesta a los apartados a) y b). Para el apartado c) hay que suponer que los enteros asignados a cada punto son sus coordenadas en un origen cualquiera, nosotros supondremos que el origen está en A con lo que las coordenadas de A son (0,0) y las de D(3,0). los seis sumandos corresponden a las parajas AB, AC, AD, BC, BD y CD. El correspondiente a AD es constante y vale 3+3 = 6. Los correspondientes a AB y BD valen en conjunto siempre 6 ya que A está en fila inferior y columna izquierda y D en la fila superior y columna derecha. Por el mismo motivo los sumandos correspondientes a AC y CD valen entre los dos siempre 6. Sólo queda el sumando |X i - X j | + | Y i - Y j | correspondiente a BC que por simple comprobación en todos los casos “originales” vale siempre 3. La suma completa es entonces constante y vale 6 + 6 + 6 + 3 = 21. A D A D A D B B B C 1 C 2 C 3 C 4 C 1 C 2 C 3 C 1 C 1 6.- Una máquina de juego de un casino tiene una pantalla en la que se ofrece un esquema como el de la figura. Para comenzar el juego aparece una bola en el punto S. A cada impulso que recibe del jugador, esa bola se mueve hasta una de las letras inmediatas con la misma probabilidad para cada una de ellas. La partida termina al ocurrir el primero de los dos hechos siguientes: a) La bola vuelve a S y entonces el jugador pierde. b) La bola llega a G y entonces el jugador gana. Se pide la probabilidad de que el jugador gane y la duración media de las partidas. Solución (F. Bellot) Podemos representar el desarrollo del juego mediante un diagrama en árbol: La probabilidad de que el juego tenga longitud 2 es 1 3 La probabilidad de que el juego tenga longitud 4 es : 1 3 1 2 1 3 2 2 2 3 2 • • • • = La probabilidad de que el juego tenga longitud 6 es : 1 3 1 2 1 3 2 2 2 1 3 2 3 2 3 • • • • • • = , etc, en general la probabilidad de que el juego tenga longitud 2n es: 2 3 1 n n − Entonces, la duración media M de un juego es la suma de cada longitud por la probabilidad respectiva : M n n n n n n n = =       − = ∞ = ∞ ∑ ∑ 2 3 2 2 3 1 1 1 serie aritmético-geométrica que se suma por el mismo método que la geométrica: M M M M − = = +       − = ⇒ = = 2 3 3 2 3 2 3 1 2 3 2 2 3 6 2 • S G A B C D S C D A G C D B S C D A C D G B C D A S S 1 1/3 1/2 1/3 1/3 La probabilidad P de ganar será la suma de las probabilidades de ganar en 4 pasos más la de que gane en 6 pasos ...etc.: P = + + + = 1 3 2 3 2 3 1 3 2 3 2 4 .... OLIMPIADA MATEMÁTICA ESPAÑOLA Fase Nacional 1994 (Madrid) Primera sesión 1.- Demostrar que si entre los infinitos términos de una progresión aritmética de números enteros positivos hay un cuadrado perfecto, entonces infinitos términos de la progresión son cuadrados perfectos. 2.- Sea OXYZ un triedro trirectángulo de vértice O y aristas X, Y, Z. Sobre la arista Z se toma un punto fijo C, tal que OC = c. Sobre X e Y se toman respectivamente dos puntos variables P y Q de modo que la suma OP + OQ sea una constante dada k. Para cada par de puntos P y Q, los cuatro puntos O, C, P, Q están en una esfera, cuyo centro W se proyecta sobre el plano OXY. Razonar cuál es el lugar geométrico de esa proyección. Razonar también cuál es el lugar geométrico de W. 3.- Una oficina de Turismo va a realizar una encuesta sobre número de días soleados y número de días lluviosos que se dan en el año. Para ello recurre a seis regiones que le transmiten los datos de la siguiente tabla: Región Soleados o lluviosos Inclasificables A 336 29 B 321 44 C 335 30 D 343 22 E 329 36 F 330 35 La persona encargada de la encuesta no es imparcial y tiene esos datos más detallados. Se da cuenta de que, prescindiendo de una de las regiones, la observación da un número de días lluviosos que es la tercera parte del de días soleados. Razonar cuál es la región de la que prescindirá. Segunda sesión 4.- El ángulo A del triángulo isósceles ABC mide 2/5 de recto, siendo iguales sus ángulos B y C. La bisectriz de su ángulo C corta al lado opuesto en el punto D. Calcular las medidas de los ángulos del triángulo BCD. Expresar la medida a del lado BC en función de la medida b del lado AC, sin que en la expresión aparezcan razones trigonométricas. 5.- Con 21 fichas de damas, unas blancas y otras negras, se forma un rectángulo de 3x7. Demostrar que siempre hay cuatro fichas del mismo color situadas en los vértices de un rectángulo. 6.- Un polígono convexo de n lados se descompone en m triángulos, con los interiores disjuntos, de modo que cada lado de esos m triángulos lo es también de otro triángulo contíguo o del polígono dado. Probar que m + n es par. Conocidos n y m hallar el número de lados distintos que quedan en el interior del polígono y el número de vértices distintos que quedan en ese interior. Problemas de la XXX Olimpiada Madrid 1994 1.- Demostrar que si entre los infinitos términos de una progresión aritmética de números enteros positivos hay un cuadrado perfecto, entonces infinitos términos de la progresión son cuadrados perfectos. Solución Bastará probar que a partir de un cuadrado perfecto podemos construir otro. Sea la progresión: a 2 , a 2 + d, a 2 + 2d, ......,a 2 + kd...... Como (a + d) 2 = a 2 + 2ad + d 2 = a 2 + (2a + d)d, basta tomar k = 2a + d para obtener otro cuadrado en la progresión. 2.- Sea OXYZ un triedro trirectángulo de vértice O y aristas X, Y, Z. Sobre la arista Z se toma un punto fijo C, tal que OC = c. Sobre X e Y se toman respectivamente dos puntos variables P y Q de modo que la suma OP + OQ sea una constante dada k. Para cada par de puntos P y Q, los cuatro puntos O, C, P, Q están en una esfera, cuyo centro W se proyecta sobre el plano OXY. Razonar cuál es el lugar geométrico de esa proyección. Razonar también cuál es el lugar geométrico de W. Solución En la figura se muestran con trazo discontínuo las circunferencias que resultan de intersecar la esfera con los planos coordenados. Las proyecciones del centro W de la esfera sobre estos planos coinciden con los centros de estas circunferencias (denotados F, G y H en la figura) y al ser el triedro trirectángulo, F, G y H estan en los puntos medio de los segmentos PQ, QC y CP que son diámetros de sus circunferencias. Parametrizando con la distancia OP= λ tenemos trivialmente en la referencia OXYZ la siguientes coordenadas: P(λ,0,0); Q(0,k-λ,0); C(0,0,c); F k k c c k c λ λ λ λ λ λ 2 2 0 0 2 2 2 0 2 2 2 2 , , ; , , ; , , ; , , − | \ | ¹ | − | \ | ¹ | | \ | ¹ | − | \ | ¹ | G H W El lugar de F es la recta x y k + = 2 del plano XOY. El lugar de W es una recta paralela a la anterior situada en el plano z c = 2 , más concretamente es la intersección de los planos: x y k z c + = = ¦ ´ ¦ ¹ ¦ 2 2 X Y Z C P Q W H G F O 3.- Una oficina de Turismo va a realizar una encuesta sobre número de días soleados y número de días lluviosos que se dan en el año. Para ello recurre a seis regiones que le transmiten los datos de la siguiente tabla: Región Soleados o lluviosos Inclasificables A 336 29 B 321 44 C 335 30 D 343 22 E 329 36 F 330 35 La persona encargada de la encuesta no es imparcial y tiene esos datos más detallados. Se da cuenta de que, prescindiendo de una de las regiones, la observación da un número de días lluviosos que es la tercera parte del de días soleados. Razonar cuál es la región de la que prescindirá. Solución Al suprimir una región, la suma de días soleados o lluviosos de las restantes ha de ser múltiplo de 4. Esta suma vale para las seis regiones 1994 que dividido entre 4 da resto 2. El único dato de esta columna que da resto 2 al dividirlo entre 4 es 330 correspondiente a la región F. Suprimiendo esta región quedan entre las cinco restantes 416 días lluviosos y 3·416 = 1248 días soleados. 4.- El ángulo A del triángulo isósceles ABC mide 2/5 de recto, siendo iguales sus ángulos B y C. La bisectriz de su ángulo C corta al lado opuesto en el punto D. Calcular las medidas de los ángulos del triángulo BCD. Expresar la medida a del lado BC en función de la medida b del lado AC, sin que en la expresión aparezcan razones trigonométricas. Solución Con los datos del enunciado tenemos: en el triángulo ABC ∠ BAC = 36º; ∠ ABC = ∠ ACB = 72º en el triángulo CBD ∠ BCD = 36º; ∠ CDB = ∠ BDC = 72º en el triángulo ADC ∠ DAC = ∠ ACD = 72º; ∠ ADC = 108º por tanto ∆BCD y ∆ADC son isósceles y ademas ∆BCD es semejante al ∆ABC. Para los lados se tiene: DC = AD = a; BD = b - a. Expresando la proporcionalidad derivada de la semejanza anterior: b a a a b a b ab a ab b a b a b − = ⇔ = − ⇔ + − = ⇔ | \ | ¹ | + − = 2 2 2 2 2 0 1 0 y resolviendo queda ( ) a b a b = − ⇔ = − 5 1 2 5 1 2 es decir a es la sección áurea de b. A B D b-a a a C b 5.- Con 21 fichas de damas, unas blancas y otras negras, se forma un rectángulo de 3x7. Demostrar que siempre hay cuatro fichas del mismo color situadas en los vértices de un rectángulo. Solución Dispondremos el tablero en posición vertical, es decir, con 7 filas y 3 columnas. Asignaremos el color blanco a la cifra 0 y el negro a la cifra 1. De este modo cada fila representa un número escrito en base 2. En primer lugar es fácil ver que si en una fila se colocan todas las fichas del mismo color, por ejemplo el negro, necesariamente habrá un rectángulo ya que no podemos colocar en ninguna fila dos fichas negras y sólo podemos llenar un máximo de 5 filas en total sin formar rectángulo. Por otra parte si dos números son iguales sus filas forman rectángulo, luego todas las filas han de representar números distintos. Por la consideración anterior hemos de excluir los números 000 y 111. Con tres cifras en base dos existen 2 3 = 8 números distintos, quitando los anteriores quedan 6 para 7 filas por lo que necesariamente hemos de repetir y formar rectángulo. El problema tendría solución en un tablero de 3x6 tal como se muestra en la figura. 6.- Un polígono convexo de n lados se descompone en m triángulos, con los interiores disjuntos, de modo que cada lado de esos m triángulos lo es también de otro triángulo contíguo o del polígono dado. Probar que m + n es par. Conocidos n y m hallar el número de lados distintos que quedan en el interior del polígono y el número de vértices distintos que quedan en ese interior. Solución Como hay m triángulos, hay 3m lados; de ellos 3m - n son interiores, y como lado interior pertenece a dos triángulos, hay 3 2 m n − lados interiores distintos. En particular 3m - n es par, luego m y n tienen la misma paridad y m + n es par. Supongamos que el número de vértices v sólo depende de m y n. Razonemos por inducción sobre v. Si no hay ningún vértice interior (v = 0), uniendo un vértice del polígono con los otros, se divide en n - 2 = n + 2v - 2 triángulos. Supongamos que hay v vértices interiores y n + 2v - 2 triángulos. Al añadir un vértice hay dos posibilidades: a) El vértice está en el interior de un triángulo, entonces, para que se cumplan las condiciones del enunciado, debe unirse a cada uno de los tres vértices del triángulo que se divide en tres y el número de triángulos ahora es: n + 2v - 2 + 2 = n + 2(v + 1) - 2. b) El vértice está en un lado, entonces hay que unirlo con el vértice opuesto de cada uno de los dos triángulos que comparten ese lado, cada triángulo se descompone en dos y el número de triángulos es ahora: n + 2v - 2 + 2 = n + 2(v + 1) - 2. En conclusión: m = n + 2v - 2 ⇒ = − + v m n 2 2 1 2 3 4 5 6 OLIMPIADA MATEMÁTICA ESPAÑOLA Fase nacional 1995 (Castellón) Primera sesión 1.- Se consideran conjuntos A de cien números naturales distintos, que tengan la propiedad de que si a, b y c son elementos cualesquiera de A (iguales o distintos), existe un triángulo no obtusángulo cuyos lados miden a, b y c unidades. Se denomina S(A) a la suma de los perímetros considerados en la definición de A. Calcula el valor mínimo de S(A) 2.- Recortamos varios círculos de papel (no necesariamente iguales) y los extendemos sobre una mesa de modo que haya algunos solapados (con parte interior común), pero de tal forma que no haya ningún círculo dentro de otro. Prueba que es imposible ensamblar las piezas que resultan de recortar las partes no solapadas y componer con ellas círculos distintos. 3.- Por el baricentro G de un triángulo ABC se traza una recta que corta al lado AB en P y al lado AC en Q. Demuestra que: Segunda sesión 4.- Siendo p un número primo, halla las soluciones enteras de la ecuación: p.(x + y) = x.y 5.- Demuestra que en el caso de que las ecuaciones: x 3 + mx - n = 0 nx 3 - 2 m 2 x 2 - 5mnx - 2m 3 - n 2 = 0 (n no nulo) tengan una raíz común, la primera tendrá dos raíces iguales y determina entonces las raíces de las dos ecuaciones en función de n. 6.- En la figura, AB es un segmento fijo y C un punto variable dentro de él. Se construyen triángulos equiláteros de lados AC y CB, ACB' y CBA' en el mismo semiplano definido por AB, y otro de lado AB, ABC' en el semiplano opuesto. Demuestra: a) Las rectas AA', BB' y CC' son concurrentes. b) Si llamamos P al punto común a las tres rectas del apartado a), hallar el lugar geométrico de P cuando C varía en el segmento AB. lugar geométrico de P cuando C varía en el segmento AB. c) Los centros A'', B'' y C'' de los tres triángulos forman un triángulo equilátero. d) Los puntos A'', B'', C'' y P están sobre una circunferencia. Olimpiada Española 1995 1.- Se consideran conjuntos A de cien números naturales distintos, que tengan la propiedad de que si a, b y c son elementos cualesquiera de A (iguales o distintos), existe un triángulo no obtusángulo cuyos lados miden a, b y c unidades. Se denomina S(A) a la suma de los perímetros considerados en la definición de A. Calcula el valor mínimo de S(A). Solución: Si n es el menor de los elementos de A y m el mayor, al tener A cien elementos distintos, será m ≥ n + 99. Para que el triángulo isósceles de lados n, n, m sea no obtusángulo debe ocurrir que m 2 ≤ 2 n 2 ; si m es lo menor posible, m = n + 99 deberá ser (n + 99) 2 ≤ 2n 2 , o sea: ( ) n - 198n - 99 0 n 99 n 99 1 + 2 2 2 2 ≥ ⇔ ≥ + + ⇔ ≥ ⇔ ≥ 99 99 240 2 n . Si n < 240, es seguro que el conjunto no cumple la condición del enunciado pues m 2 ≥ (n+99) 2 ≥ 2n 2 y el triángulo de lados n, n, m no puede ser no obtuságulo. En particular la condición se cumple para el conjunto: A = {240, 241, 242, ...., 339} Cualquier otro conjunto que cumpla la condición, tendrá sus elementos respectivamente iguales o mayores que los de éste. Este es, por tanto el que da lugar al mínimo S(A). El número de triángulos que debe considerarse es el de variaciones ternarias con repetición de los elementos de A, que es 100 3 = 1000000, con lo que el número de lados en total será de 3000000; de ellos habrá 30000 de longitud 240, otros tantos de longitud 241, etc. Luego ( ) S A ( ) • ( .... ) • • = + + + + = + = 30000 240 241 242 339 30000 100 240 339 2 868500000 unidades. Este es el valor mínimo buscado. 2.- Recortamos varios círculos de papel (no necesariamente iguales) y los extendemos sobre una mesa de modo que haya algunos solapados (con parte interior común), pero de tal forma que no haya ningún círculo dentro de otro. Prueba que es imposible ensamblar las piezas que resultan de recortar las partes no solapadas y componer con ellas círculos distintos. Solución: La frontera de las piezas recortadas (que no sean círculos completos) está formada por arcos cóncavos y convexos (vistos desde fuera) que se cortan en puntos que llamaremos vértices. En un vértice pueden concurrir dos arcos cóncavos o uno cóncavo y otro convexo, pero nunca dos convexos ya que éstos únicamente provienen de la frontera de los círculos iniciales. Además, los ángulos que forman los arcos en cada vértice no son de 0º ni de 180º ya que excluimos las tangencias interiores. Supongamos que tenemos un círculo obtenido ensamblando piezas recortadas. Existe al menos un punto P de la frontera de dicho círculo en el que concurren tres o más arcos de la frontera de las pieza ensambladas (P es vértice de dos o más piezas). La tangente al círculo en P deja a todos los arcos en un mismo semiplano. Elegido un sentido de rotación en P a partir de la tangente, y avanzando en este sentido, el primer arco que encontramos es convexo y el último cóncavo. Por lo tanto es necesario que existan dos arcos consecutivos uno convexo y el otro cóncavo los cuales forman parte de la frontera de una de las piezas ensambladas. Como el arco que forman dichas piezas no puede ser ni 0º ni 180º, el punto P es un vértice de la pieza. Esto es contradictorio pues en ningún vértice pueden concurrir dos arcos convexos vistos desde fuera. Nota. Hay que entender en el enunciado.que quedan excluidas las tangencias interiores. De no ser así pueden encontrarse contraejemplos como el “despiece” que se muestra en la figura: 3.- Por el baricentro G de un triángulo ABC se traza una recta que corta al lado AB en P y al lado AC en Q. Demuestra que: PB PA QC QA • ≤ 1 4 Solución: Dupliquemos el triángulo trazando AD paralela a BC y CD paralela a BA como muestra la figura y tomemos la longitud del lado AB como unidad. Llamando M a la intersección de CD con la recta PQ y x = PB; 1-x = AP, tenemos: Por semejanza de ∆AQP y ∆QMC: QC QA MC AP MC x = = − 1 Por semejanza de ∆GPB y ∆GMD: PB MD GB GD = = 1 2 . Luego: MD = 2x y MC = 1 - 2x. Sustituyendo en el primer miembro de la relación del enunciado queda: ( ) PB PA QC QA x x x x x x • ( ) ( ) ≤ ⇔ − − ≤ ⇔ − + ≥ ⇔ − ≥ 1 4 1 2 1 1 4 9 6 1 0 3 1 0 2 2 2 Relación válida para cualquier x. La igualdad se alcanza para PB x MC = = ⇔ = 1 3 1 3 ⇔ PQ paralela al lado BC. P 2 P G A C D Q M B 4.- Halla las soluciones enteras de la ecuación: p·(x + y) = x·y siendo p un número primo. Solución: Ya que p es primo, p ≠ 0 y p ≠ 1. De la ecuación resulta que p divide a x o p divide a y. Como la ecuación es simétrica respecto de x e y, si (α, β) es solución, también lo será (β, α). Si p divide a x, x = p·a, (a∈Z) la ecuación se puede poner como: ( ) p pa y pay pa y ay y pa a + = ⇒ + = ⇒ = − ) 1 ya que a es entero además a y a - 1 son primos entre sí, luego a - 1 divide a p. Al ser p primo sólo hay cuatro posibilidades: a - 1 = ± 1 y a - 1 = ± p. Examinemos todos los casos. i) a - 1 = -1, entonces a = 0, x = 0, y = 0 ii) a - 1 = 1, entonces a = 2, x = 2p ⇒ = − = y p p 2 2 1 2 . iii) a - 1 = p, entonces a = p + 1, x = p(p + 1) ( ) ⇒ = + + − = + y p p p p 1 1 1 1 . iiii) a - 1 = -p, entonces a = 1 - p, x = p(1 - p) ( ) ⇒ = − − − = − y p p p p 1 1 1 1 . En resumen las soluciones son: (0, 0); (2p, 2p); (p(p+1), p+1); (p(1-p), p-1); y por la simetría añadimos (p+1, p(p+1)); (p-1, p(1-p)). 5.- Demuestra que en el caso de que las ecuaciones: x 3 + mx - n = 0 , nx 3 - 2 m 2 x 2 - 5mnx - 2m 3 - n 2 = 0 (n ≠ 0) tengan una raíz común, la primera tendrá dos raíces iguales y determina entonces las raíces de las dos ecuaciones en función de n. Solución: Sea α la raíz común de ambas ecuaciones. Entonces α 3 + mα = n (1) y sustituyendo en la segunda ecuación se obtiene, tras hacer operaciones: 6m α 4 + 8m 2 α 2 +2m 3 = 0 Supongamos m ≠ 0, entonces simplificando la relación anterior queda: 3 α 4 + 4m α 2 +m 2 = 0 (2) Resolviendo (2) respecto de m obtenemos m= −    α α 2 2 (i) -3 (ii) Analicemos cada caso. (i) Si m = -α 2 , sustituyendo en la primera ecuación y despejando n queda: n = α 3 - α 3 = 0 en contra de lo supuesto. Por tanto (i) queda descartado. (ii) Si m = -3α 3 , sustituyendo en la primera ecuación y despejando n queda: n = α 3 -3 α 3 = -2 α 3 y la primera ecuación queda: ( )( ) ( ) ( ) x x x x x x x 3 2 3 2 2 2 3 2 2 2 − + = − + − = − + α α α α α α α que, efectivamente, tiene la raíz α doble. de n = -2 α 3 obtenemos α = − n 2 3 . Entonces la segunda ecuación es de la forma ( ) − + + + = 2 9 15 25 0 3 3 2 2 3 α α α α x x x , y, dividiendo por (x - α) resulta ( )( ) − − + = 2 5 0 3 2 α α α x x , cuyas raíces son α y −5 α siendo doble la última. Si m = 0, las dos ecuaciones son iguales y sus tres raíces son las mismas pero la primera no tiene dos raíces iguales por lo que en el enunciado debería haberse añadido m ≠ 0. 6.- En la figura, AB es un segmento fijo y C un punto variable dentro de él. Se construyen triángulos equiláteros de lados AC y CB, ACB’ y CBA’ en el mismo semiplano definido por AB, y otro de lado AB, ABC’ en el semiplano opuesto. Demuestra: a) Las rectas AA’, BB’ y CC’ son concurrentes. b) Si llamamos P al punto común a las tres rectas del apartado a), hallar el lugar geométrico de P cuando C varía en el segmento AB. c) Los centros A’’, B’’ y C’’ de los tres triángulos forman un triángulo equilátero. d) Los puntos A’’, B’’, C’’ y P están sobre una circunferencia. Solución: a) Se traza la circunferencia circunscrita al triángulo ABC’ y se llama P a la intersección de CC’ con ella. Evidentemente (arco capaz) ∠APB=120º y PC’ es su bisectriz con lo que ∠APC=∠CPB=60º y P ha de estar en las circunferencias circunscritas a los triángulos ACB’ y BCA’. Por tanto las tres circunferencias se cortan en P. Como ∠CPB’=120º y ∠CPB=60º sumando queda: ∠BPB’=180º y P está alineado con BB’. De modo análogo se ve que P está alineado con AA’. b) Como P está definido por la intersección de la recta CC’ con la circunferencia circunscrita al triángulo ABC’ el lugar pedido es el arco APB de esa circunferencia. c) Los lados del triángulo son perpendiculares a las cuerdas PA , PB y PC que forman ángulos de 60º o 120º. por ello, entre sí forman ángulos iguales de 60º. B' A A' P C C' B A'' B'' C'' C' A B C A' B' P C'' A'' B'' d) Basta comprobar que los centros C’’, B’’, A’’ y P verifican el teorema de Tolomeo: PC A B PA B C PB A C PC PA PB AB AC CB ' ' ' ' ' ' ' ' ' ' ' ' ' ' ' ' ' ' ' ' ' ' ' ' = + ⇔ = + ⇔ = + siendo la última igualdad evidente por construcción. OLIMPIADA MATEMÁTICA ESPAÑOLA Fase nacional 1996 (Tarragona) Primera sesión 1.- Los números naturales a y b son tales que: es entero. Demostrar que el máximo común divisor de a y b no es mayor que 2.- Sea G el baricentro del triángulo ABC. Si se verifica demostrar que el triángulo es isósceles. 3.- Sean a, b ,c números reales. Se consideran las funciones . Sabiendo que demostrar que si -1 < = x < = 1, entonces: Segunda sesión 4.- Discutir la existencia de soluciones de la ecuación según los valores del parámetro real p, y resolverla siempre que sea posible. 5.- En Port Aventura hay 16 agentes secretos. Cada uno de ellos vigila a algunos de sus colegas. Se sabe que si el agente A vigila al agente B, entonces B no vigila a A. Además, 10 agentes cualesquiera pueden ser numerados de forma que el primero vigila al segundo, éste vigila al tercero,....., el último (décimo) vigila al primero. Demostrar que también se pueden numerar de este modo 11 agentes cualesquiera. 6.- La figura de la izquierda se compone de seis pentágonos regulares de lado 1m. Se dobla por las líneas de puntos hasta que coincidan las aristas no punteadas que confluyen en cada vértice. ¿Qué volumen de agua cabe en el recipiente formado?. Olimpiada Matemática. Fase Nacional. Tarragona 1996 1.- Los números naturales a y b son tales que: a b b a + + + 1 1 es entero. Demostrar que el máximo común divisor de a y b no es mayor que a b + Solución: Se tiene: a b b a a b a b ab + + + = + + + 1 1 2 2 . Sea d = m.c.d (a,b). Como ab es divisible por d 2 , entonces a b ab 2 2 + + es divisible por d 2 y también lo son a 2 + b 2 y a + b, y al ser a y b naturales, se tiene : a b d a b d + ≥ ⇔ + ≥ 2 2.- Sea G el baricentro del triángulo ABC. Si se verifica: AB + GC = AC + GB demostrar que el triángulo es isósceles. Solución: Primera solución. Teniendo en cuenta el teorema de la mediana, la relación del enunciado se escribe: c b a c b a b c − = + − − + −         2 3 2 4 2 4 2 2 2 2 2 2 , multiplicando y dividiendo por la expresión conjugada queda: ( ) ( ) c b c b m m c b m m c b c b c b − = − + ⇔ − + − +       = 2 3 3 4 2 0 2 2 Probaremos que el segundo factor es positivo, de donde se deduce la conclusión. Llamando B’ y C’ a los puntos medios de AC y Ab respectivamente, en los triángulos CC’A y BB’A tenemos por la desigualdad triangular: m b c m c b b c + > + > 2 2 ; . Sumando ambas desigualdades se obtiene el resultado. Segunda solución. Llamando A’, B’, C’ a los puntos medios de los lados BC, AC y AB respectivamente y dividiendo por dos la condición del enunciado podemos escribirla como: C' A C' G B A B G 2 2 2 2 + = + ' ' , es decir los puntos C’ y B’ están en una elipse de focos A y G. Llamando M al punto medio de C’B’ , M esta en la mediana AA’ y no es el centro de la elipse (punto medio del segmento AG), por tanto C’B’ ha de ser perpendicular a AA’, y entonces AA’ además de mediana es altura y el triángulo es isósceles. 3.- Sean a, b, c números reales. Se consideran las funciones: f x ax bx c g x cx bx a ( ) , ( ) = + + = + + 2 2 . Sabiendo que f f f ( ) , ( ) , ( ) , − ≤ ≤ ≤ 1 1 0 1 1 1 demostrar que si -1≤ x ≤ 1, entonces: f x y g x ( ) ( ) . ≤ ≤ 5 4 2 Solución: Podemos conseguir coeficientes A, B, c tales que se tenga idénticamente: f x Ax x Bx x C(x ( ) ( ) ( ) ) = + + − + − 1 1 1 2 . Particularizando para x = 1, -1 ,0 y resolviendo el sistema queda: f x f x x f x x f x x R ( ) ( ) ( ) ( ) ( ) ( )( ) = + + − − + − ∀ ∈ 1 2 1 1 2 1 0 1 2 De aquí se deduce: f x x x x x x ( ) ( ) ( ) ≤ + + − + − 1 2 1 1 2 1 1 2 ; como − ≤ ≤ + ≥ − ≥ − ≥ 1 1 1 0 1 0 1 0 2 x x x y x , , , resulta ( ) ( ) f x x x x x x x x x ( ) ≤ + + − + − = − + + = − −       ≤ 2 1 2 1 1 1 5 4 1 2 5 4 2 2 2 , por otra parte, para x ≠ 0, g x x f x ( ) =       2 1 . Entonces ( ) ( ) ( ) g x f x f x f x ( ) ( ) ( ) ( ) = + + − − + − 1 2 1 1 2 1 0 1 2 válido para − ≤ ≤ 1 1 x . Así pues g x x x x x ( ) ≤ + + − + − = − ≤ 1 2 1 2 1 2 2 2 2 A B C G C' B' A' M 4.- Discutir la existencia de soluciones de la ecuación x p x x 2 2 2 1 − + − = según los valores del parámetro real p, y resolverla siempre que sea posible. Solución: Si p < 0, entonces x p x 2 − > ; como 2 1 0 2 x − > , no existe solución. Por tanto p ≥ 0. Aislando un radical y elevando al cuadrado dos veces se llega a la ecuación: ( ) ( ) ( ) 8 2 4 4 8 2 2 2 − − = − − p x p de donde x p p , . Como x ∈ R, p < 2, así que ( ) x p p = − − 4 8 2 . Sustituyendo en la ecuación dada se obtiene ( ) ( ) ( ) ( ) 4 3 8 2 2 8 2 4 8 2 2 2 − − + − = − − p p p p p p como p > 0, |p| = p; y finalmente: 4 3 2 4 4 3 4 3 4 3 0 0 4 3 − + = − ⇒ − = − ⇒ − ≥ ⇔ ≤ ≤ p p p p p p p 5.- En Port Aventura hay 16 agentes secretos. Cada uno de ellos vigila a algunos de sus colegas. Se sabe que si el agente A vigila al agente B, entonces B no vigila a A. Además, 10 agentes cualesquiera pueden ser numerados de forma que el primero vigila al segundo, éste vigila al tercero,....., el último (décimo) vigila al primero. Demostrar que también se pueden numerar de este modo 11 agentes cualesquiera. Solución: Diremos que los agentes A y B son neutrales si A no vigila a B ni B vigila a A. Sean A 1 , A 2 ,........A n los agentes. Sean: a i el número de agentes que vigilan a Ai. b i el número de agentes que son vigilados por Ai. ci el número de agentes que son neutrales con Ai. Es claro que a b c a c b c i i i i i i i i + + = + ≤ + ≤ ∀ = 15 8 8 1 2 16 , , , , .... Notemos que si una cualquiera de las dos últimas desigualdades no se verificase, entonces no se podrían numerar 10 espías en la forma indicada. Combinando las relaciones anteriores obtenemos c i ≤ 1. Por tanto para cualquier espía el número de sus colegas neutrales es 0 ó 1. Razonemos por reducción al absurdo. Supongamos que hubiera un grupo de 11 espías que NO se pudiera numerar en la forma descrita. Sea B uno cualquiera de los espías de este grupo. Numeramos los otros 10 espías como C 1 , C 2 , ....C 10 de modo que C 1 vigila C 2 ....., C 10 vigila a C 1 . Supongamos que ninguno de los C i sea neutral respecto de B. Entonces si C 1 vigila a B, B no puede vigilar a C 2 , pues en tal caso C 1 , B, C 2 , ....C 10 formaría un grupo en las condiciones del problema, luego C 2 vigila B , etc . De este modo llegamos a la contradicción de que todos los espías del grupo vigilan a B. Por tanto cada uno de los 11 espías debe tener uno y solo uno del grupo neutral con él, lo cual es imposible. 6.- La figura de la izquierda se compone de seis pentágonos regulares de lado 1m. Se dobla por las líneas de puntos hasta que coincidan las aristas no punteadas que confluyen en cada vértice. ¿Qué volumen de agua cabe en el recipiente formado?. Solución: La figura formada por el agua es un tronco de pirámide pentagonal cuya base menor es el pentágono dado y cuya base mayor es otro pentágono regular que tiene por lado la diagonal del anterior paralela a la arista de la base como se muestra en la figura inferior derecha. Más abajo, se ha dibujado en forma invertida para una mejor comprensión del dibujo. (Figura central). Establezcamos primero algunas relaciones conocidas para un pentágono regular de lado 1. (Figura de la izquierda). Llamemos d a la diagonal. Por semejanza de los triángulos ABE y PCD tenemos: ( ) 1 1 1 1 0 1 5 2 1 2 d d d d d − = ⇔ − − = ⇒ = + = ϕ R r h H R-r 1 H-h A B C B A C 36º 1 O P A B C D E 72º ϕ es el llamado número áureo y representa la relación entre la diagonal y el lado de un pentágono regular. En nuestro caso es la relación de semejanza entre las bases del tronco de pirámide. Además : cos º 36 2 2 1 5 4 = = = + d ϕ y para el radio r : ( ) sen º sen º 36 1 2 1 2 36 1 4 2 2 = ⇔ = = − r r ϕ . Llamando V al volumen de la pirámide grande , v al de la pequeña, sabemos que V = ϕ 3 v ; y para el volumen del tronco de cono V t queda: V V v v v v ah t = − = − = − = − ϕ ϕ ϕ 3 3 3 1 1 3 1 ( ) ( ); siendo a el área del pentágono de lado 1. Sólo nos queda calcular a, h, sustituir y operar: El área a la calculamos sumado 5 triángulos isósceles de lados iguales r, r formando 72º a r r r r = = = = 5 2 72 5 2 2 36 36 5 2 36 5 4 2 2 sen º sen º cos º cos º ϕ . (hemos usado 2rsen36º = 1 de (2)). Para calcular h, por la semejanza de los triángulos de la figura central, tenemos: ( ) H R h r H h R r h r H h R r r R r r r = = − − ⇒ = − − = − − − = − − − = − − − − 1 1 1 1 1 1 1 4 1 2 2 2 2 2 ( ) ( ) ( ) ( ) ϕ ϕ ϕ ϕ ϕ ϕ Como ϕ verifica la ecuación (1): ϕ 2 = ϕ +1; tenemos para la expresión de h: ( ) ( ) ( ) h = − − − − = − − + − − − = − − + − − = − − 1 1 4 1 4 2 1 1 4 3 2 2 2 1 4 1 1 4 2 2 2 2 2 2 2 ( ) ϕ ϕ ϕ ϕ ϕ ϕ ϕ ϕ ϕ ϕ ϕ ϕ ϕ ϕ Sustituyendo las expresiones de a y h y poniendo ϕ 3 -1= (ϕ-1)(ϕ 2 + ϕ + 1); queda: V t = − − − − = + + − = + − = + − 1 3 5 4 4 1 1 4 5 12 1 4 5 6 1 3 5 6 2 1 3 2 3 2 2 2 ϕ ϕ ϕ ϕ ϕ ϕ ϕ ϕ ϕ ϕ ϕ ϕ ϕ ϕ ( ) ( ) ( ) ( ) y sustituyendo el valor de ϕ de (1), queda finalmente: V t = + − = + ≅ 5 3 2 5 5 5 15 7 5 12 2,554m 3 OLIMPIADA MATEMÁTICA ESPAÑOLA XXXIII Olimpiada Matemática Española Fase nacional 1997 (Valencia) Primera sesión 1.- Calcular la suma de los cuadrados de los cien primeros términos de una progresión aritmética, sabiendo que la suma de ellos vale -1, y que la suma de los términos de lugar par vale +1. 2.- Un cuadrado de lado 5 se divide en 25 cuadrados unidad por rectas paralelas a los lados. Sea A el conjunto de los 16 puntos interiores, que son vértices de los cuadrados unidad, pero que no están en los lados del cuadrado inicial. ¿Cuál es el mayor número de puntos de A que es posible elegir de manera que TRES cualesquiera de ellos NO sean vértices de un triángulo rectángulo isósceles?. 3.- Se consideran las parábolas y = x 2 + px + q que cortan a los ejes de coordenadas en tres puntos distintos por los que se traza una circunferencia. Demostrar que todas las circunferencias trazadas al variar p y q en R pasan por un punto fijo que se determinará. Segunda sesión 4.- Sea p un número primo. Determinar todos los enteros tales que es natural. 5.- Demostrar que en un cuadrilátero convexo de área unidad, la suma de las longitudes de todos los lados y diagonales no es menor que . 6.- Un coche tiene que dar una vuelta a un circuito circular. En el circuito hay n depósitos con cierta cantidad de gasolina. Entre todos los depósitos contienen la cantidad exacta que el coche necesita para dar una vuelta. El coche comienza con el depósito vacío. Demostrar que con independencia del número, posición y cantidad de combustible de cada depósito, siempre se puede elegir un punto de comienzo que le permita completar la vuelta. Notas: a) El consumo es uniforme y proporcional a la distancia recorrida. b) El tamaño del depósito es suficiente para albergar toda la gasolina necesaria para dar una vuelta. XXXIII Olimpiada Matemática Española Fase Nacional Valencia, Marzo 1997 Primera Sesión 1.- Calcular la suma de los cuadrados de los cien primeros términos de una progresión aritmética, sabiendo que la suma de ellos vale -1, y que la suma de los términos de lugar par vale +1. 2.- Un cuadrado de lado 5 se divide en 25 cuadrados unidad por rectas paralelas a los lados. Sea A el conjunto de los 16 puntos interiores, que son vértices de los cuadrados unidad, pero que no están en los lados del cuadrado inicial. ¿Cuál es el mayor número de puntos de A que es posible elegir de manera que TRES cualesquiera de ellos NO sean vértices de un triángulo rectángulo isósceles?. 3.- Se consideran las parábolas y = x 2 + px + q que cortan a los ejes de coordenadas en tres puntos distintos por los que se traza una circunferencia. Demostrar que todas las circunferencias trazadas al variar p y q en R pasan por un punto fijo que se determinará. Segunda Sesión 4.- Sea p un número primo. Determinar todos los enteros k∈Z tales que k kp 2 − es natural. 5.- Demostrar que en un cuadrilátero convexo de área unidad, la suma de las longitudes de todos los lados y diagonales no es menor que ( ) 2 2 2 + . 6.- Un coche tiene que dar una vuelta a un circuito circular. En el circuito hay n depósitos con cierta cantidad de gasolina. Entre todos los depósitos contienen la cantidad exacta que el coche necesita para dar una vuelta. El coche comienza con el depósito vacío. Demostrar que con independencia del número, posición y cantidad de combustible de cada depósito, siempre se puede elegir un punto de comienzo que le permita completar la vuelta. Notas: a) El consumo es uniforme y proporcional a la distancia recorrida. b) El tamaño del depósito es suficiente para albergar toda la gasolina necesaria para dar una vuelta. Soluciones 1.- Calcular la suma de los cuadrados de los cien primeros términos de una progresión aritmética, sabiendo que la suma de ellos vale -1, y que la suma de los términos de lugar par vale +1. Sea la progresión a, a + d, a + 2d, ......, a + 99d, entonces tenemos que hallar: S = a 2 + (a + d) 2 + (a + 2d) 2 +.....+ (a + 99d) 2 =100a 2 + 2ad (1 + 2 + ...+ 99) +d 2 (1 2 + 2 2 +....+ 99 2 ). Para calcular a y d resolvemos el sistema: ( ) ( ) a a d a d a d + + = − + + + = ¦ ´ ¹ 99 50 1 99 25 1 que operado y resuelto sale: a = -2,98; d = 0,06. El resto es fácil de calcular. Los paréntesis son progresiones de primer y segundo orden. 1 + 2 + ...+ 99 = 4950; 1 2 + 2 2 +....+ 99 2 = 328350. El resultado final es S = 299,98 2.- Un cuadrado de lado 5 se divide en 25 cuadrados unidad por rectas paralelas a los lados. Sea A el conjunto de los 16 puntos interiores, que son vértices de los cuadrados unidad, pero que no están en los lados del cuadrado inicial. ¿Cuál es el mayor número de puntos de A que es posible elegir de manera que TRES cualesquiera de ellos NO sean vértices de un triángulo rectángulo isósceles?. Numeremos los puntos como indica la figura 13 14 15 16 9 10 11 12 5 6 7 8 1 2 3 4 Por simple tanteo se obtiene un conjunto de seis puntos verificando la condición del enunciado, por ejemplo {1, 2, 3, 8, 12, 16}. Supongamos que hubiera un conjunto M de 7 puntos verificando la condición del enunciado. Notemos que si cuatro puntos forman un cuadrado, a lo sumo figurarán dos de ellos en M. Los puntos de los conjuntos {1, 4, 16, 13}, {2, 8, 15, 9}, {3, 12, 14, 5} forman cuadrados y su unión forma el “contorno exterior” de A, luego a los sumo 6 de los puntos elegidos deben estar en M y por tanto al menos un punto de M debe ser del conjunto “interior” de A: {6, 7, 10, 11}. Por la simetría de la figura supongamos que es el 7. Como {7, 16, 9} y {1, 7, 14} forman triángulos rectángulo isósceles, a lo sumo 2 de los puntos del conjunto {1, 9, 14, 16} deberán figurar en M. Además {5, 7, 13, 15} forman un cuadrado por tanto a lo sumo podremos elegir dos números entre {5, 13, 15} , de ello se deduce en M deben figurar al menos tres puntos de {2, 3, 4, 6, 8, 10, 11, 12} . Si descomponemos este conjunto en dos subconjuntos “cuadrados” y disjuntos : {3, 6, 11, 8} y {2, 4, 10, 12} forzosamente de uno de ellos habremos de tomar dos puntos y uno de otro. Si tomamos dos puntos del primero las únicas posibilidades son {3, 11} y {6, 8} ambas incompatibles con cualquier elección del punto restante en el segundo conjunto. Si los dos puntos se eligen del segundo las ínicas maneras son {2, 12} y {4, 10}, de nuevo incompatibles con cualquier elección del punto que falta en el primer conjunto. En resumen el número máximo de elementos es 6. 3.- Se consideran las parábolas y = x 2 + px + q que cortan a los ejes de coordenadas en tres puntos distintos por los que se traza una circunferencia. Demostrar que todas las circunferencias trazadas al variar p y q en R pasan por un punto fijo que se determinará. 1ª Solución (analítica) Sean α y β las raíces . Los tres puntos que definen la circunferencia son A(α, 0); B(β, 0); C(0, q). Verificando α + β = - p. y αβ = q. (1) La mediatriz de AB es la recta paralela al eje OY de ecuación x p = − 2 . Hallando la mediatriz de AC, cortando con la anterior y teniendo en cuenta (1) se obtiene para el centro las coordenadas − + | \ | ¹ | p q 2 1 2 , y para el radio ( ) r p q = + − 2 2 1 4 . La ecuación de la circunferencia es: ( ) x p y q p q + | \ | ¹ | + − + | \ | ¹ | = + − 2 1 2 1 4 2 2 2 2 , que una vez operada queda: ( ) x y px q y q 2 2 1 0 + + − + + = que se verifica para el punto (0, 1) con independencia de p y q como se comprueba por simple sustitución. Claramente el punto fijo se puede obtener a partir de tres circunferencias concretas. 2ª Solución (geométrica). Puesto que la parábola corta al eje de abscisas en dos puntos, se podrá escribir en la forma: y = (x - a) (x - b) y los puntos de intersección son A(a, 0); B(b, 0); C(0,ab) La inversión de polo el origen que transforma A en B, transforma C en U(0, 1) , así que los cuatro puntos A,B,C,U son concíclicos y todas las circunferencias pasan por el punto fijo U. 4.- Sea p un número primo. Determinar todos los enteros k∈Z tales que k kp 2 − es natural. Solución: Pongamos ( ) k kp n k pk n k p p n 2 2 2 2 2 0 4 2 1 − = ⇔ − − = ⇒ = ± + . El radicando ha de ser cuadrado perfecto, llamésmole a. Se tiene: p 2 + 4n 2 = a 2 ⇔ p 2 = (a+2n)(a-2n). Como p es primo y a + 2n ≥ a - 2n, sólo hay dos posibilidades: 1) a + 2n = p 2 y a - 2n =1 2) a + 2n = p y a - 2n = p En el caso 1) a p = + 2 1 2 ; n p = − 2 1 4 , lo que exige p ≠ 2 (n natural). En el caso 2) resulta a = p ; n = 0. Sustituyendo los valores de a en (1) y operando queda: Si p = 2 , entonces k = 2 o k = 0 Su p ≠ 2 entonces quedan los cuatro valores: k p k p k p k 1 2 2 2 3 4 1 2 1 2 0 = + | \ | ¹ | = − − | \ | ¹ | = = , , , 5.- Demostrar que en un cuadrilátero convexo de área unidad, la suma de las longitudes de todos los lados y diagonales no es menor que ( ) 2 2 2 + . Solución. 1ª Sea el cuadrilátero de lados a, b, c, d y diagonales p y q. Trazando la paralelas por cada vértice a la diagonal que no pasa por él se forma un paralelogramo de área 2 y lado p y q. Por el teorema isoperimétrico, de todos los paralelogramos de área 2, el cuadrado tiene perímetro mínimo que vale 4 2 , luego ( ) ( ) 2 4 2 2 2 1 p q p q + ≥ ⇔ + ≥ En cuanto al los lados por el mismo teorema para una cuadrado de área 1 el perímetro es 4 luego: a + b + c + d ≥ 4 (2) Sumando(1) y (2) se obtiene el resultado. Solución 2ª (Sin usar la propiedad isoperimétrica). Consiste en establecer directamente las desigualdades (1) y (2). Si α es el ángulo que forman las diagonales, tenemos: 1 2 2 2 = ≤ ⇔ ≥ pq pq pq senα pero (p + q) 2 = (p - q) 2 + 4pq ≥ 4pq ≥ 8. de donde p q + ≥ = 8 2 2 (1). Para los lados, si descomponemos el cuadrilátero en dos triángulos mediante la diagonal q, tenemos: 1 2 2 ≤ + ab cd Descomponiendo ahora en dos triángulos mediante la diagonal p resulta: 1 2 2 ≤ + bc da y de ambas desigualdades se obtiene: ab + bc + cd + da ≥ 4. Pero:(a + b + c + d) 2 = ((a + c) - (b + d)) 2 + 4 (a + c)(b + d) ≥ 4 (a + c)(b + d) ≥ 16, de donde a + b + c + d ≥ 4 (2) d a b c q p Basta sumar (1) y (2) para obtener lo pedido. 6.- Un coche tiene que dar una vuelta a un circuito circular. En el circuito hay n depósitos con cierta cantidad de gasolina. Entre todos los depósitos contienen la cantidad exacta que el coche necesita para dar una vuelta. El coche comienza con el depósito vacío. Demostrar que con independencia del número, posición y cantidad de combustible de cada depósito, siempre se puede elegir un punto de comienzo que le permita completar la vuelta. Notas: a) El consumo es uniforme y proporcional a la distancia recorrida. b) El tamaño del depósito es suficiente para albergar toda la gasolina necesaria para dar una vuelta. Solución 1 (Sergi Elizalde . Concursante) Sean c 1 , c 2 , .....,c n las cantidades de combustible en cada uno de los n depósitos y sean d 1 , d 2 ,....,d n las distancias a recorrer desde cada depósito hasta el siguiente. Hagamos el gráfico del consumo comenzando en un punto de aprovisionamiento cualquiera. Notemos que los tramos inclinado tienen todos la misma pendiente. Los tramos bajo el eje representan las situaciones imposibles. La pendiente de los tramos inclinados vale : − ∑ ∑ c d i i . La hipótesis de que el total de combustible es la cantidad exacta para dar la vuelta se traduce en que la gráfica comienza y termina en el eje OX. La función resultante (trazo continuo) tiene un mínimo, en la figura el punto 3. Basta comenzar en ese punto para asegurar que el recorrido es posible. En efecto, gráficamente equivale a trasladar el eje OX en sentido vertical hasta el punto más bajo con lo que aseguramos que ninguna zona queda bajo el eje. La nueva gráfica puede trazarse a partir del punto 3 siguiendo el mismo trazado hacia la derecha y trasladando la parte anterior (tramos 1-2 y 2-3) al punto final de la gráfica anterior (de puntos en la figura). Solución 2 (Mª A. López Chamorro. Miembro del Jurado). Se numeran los depósitos de 1 a n comenzando por uno cualquiera en sentido antihorario. Llamamos: a 1 , a 2 , ...,a n a la cantidad de gasolina de cada depósito. b 1 , b 2 , ...,b n a la cantidad de gasolina necesaria para ir del depósito a i al siguiente. d 1 = a 1 -b 1 , d 2 = a 2 - b 2 ,....., d n = a n - b n Diremos que un depósito es positivo o negativo según lo sea d i . Si d i = 0 , la ubicación del depósito i no influye en la ordenación del recorrido. Por ello podemos suponer sin pérdida de generalidad que d i ≠ 0 para todo i. Por otra parte, si hay varios depósitos consecutivos positivos o negativos, el tramo limitado por ellos se puede considerar como un único tramo positivo o negativo. Así, el problema se reduce a 1 2 3 1 2 4 5 6 7 tener un número par de depósitos alternativamente positivos o negativos. Agrupando los tramos por parejas, éstas resultarán positivas o negativas y volvemos a repetir el proceso. Así reducimos el caso a un número de depósitos n 1 < n/2. Como n < 2 k , a lo sumo en k - 1 etapas llegaremos a tener 2 depósitos, uno con más gasolina que otro, en cuyo caso empezando por el que tenga más combustible se puede completar el circuito. El caso de un sólo depósito es trivial. Se empieza y termina en ese único depósito. OLIMPIADA MATEMÁTICA ESPAÑOLA XXXIV Olimpiada Matemática Española Fase nacional 1998 (Tarazona) Primera sesión 1.- Un cuadrado ABCD de centro O y lado 1, gira un ángulo a en torno a O. Hallar el área común a ambos cuadrados. 2.- Hallar todos los números naturales de 4 cifras, escritos en base 10, que sean iguales al cubo de la suma de sus cifras. 3.- Se considera el triángulo ABC y su circunferencia circunscrita. Si D y E son puntos sobre el lado BC tales que AD y AE son, respectivamente, paralelas a las tangentes en C y en B a la circunferencia circunscrita, demostrar que: Segunda sesión 4.- Hallar las tangentes de los ángulos de un triángulo sabiendo que son números enteros positivos. 5.- Hallar todas las funciones estrictamente crecientes y tales que: f(n + f(n)) = 2 f(n) para n = 1, 2, 3, ... 6.- Determina los valores de n para los que es posible construir un cuadrado de n x n ensamblando piezas del tipo: Olimpiada Española 1998 1.- Un cuadrado ABCD de centro O y lado 1, gira un ángulo α en torno a O. Hallar el área común a ambos cuadrados. Solución 1. Por la simetría bastará considerar 0 < α < 90º, ya que la función es periódica con periodo de un cuarto de vuelta. El área pedida S(α) sale restando del área del cuadrado cuatro triángulos como el PA’M. Llamando x al cateto PA’ e y al cateto A’M, el área de cuatro triángulos vale 2xy. Como el lado B’A’ vale 1, tenemos: ( ) x y x y + = − + 1 1 2 2 relación que elevada al cuadrado y simplificada queda: ( ) 2 1 2 2 2 2 xy x y = − + pero x x y y x y = + = + 2 2 2 2 cos , sen α α , y sustituyendo en (1) resulta: ( ) x y x y 2 2 2 2 1 1 1 1 + + + = ⇔ + = + + cos sen sen cos α α α α sustituyendo en (2) y operando obtenemos: 2 1 2 1 1 1 xy = − + + = + − + + sen cos sen cos sen cos α α α α α α . Finalmente para el área pedida obtenemos: ( ) S α α α α α α α = − + − + + = + + 1 1 1 2 1 sen cos sen cos sen cos con 0 ≤ α ≤ 90º Solución 2. El área pedida consta de 8 triángulos como el sombreado en la figura OPM. Tomando como base b = MP, la altura es constante (de trazos en la figura) y vale ½. En el triángulo PA’M se tiene: MA’ = b cos α, PA’ = b sen α ; pero BM = MA’ y PA = PA’, además: BM + MP + PA = 1 ⇔ b cos α + b + b sen α = 1, de donde b = + + 1 1 sen cos α α y el área pedida es: O D A B C D' A' C' B' M P O D A B C D' A' C' B' M P S( ) sen cos sen cos α α α α α = + + = + + 8 1 2 1 2 1 1 2 1 con 0 ≤ α ≤ 90º 2.- Hallar todos los números naturales de 4 cifras, escritos en base 10, que sean iguales al cubo de la suma de sus cifras. Solución: Sea n un número verificando el enunciado, y s la suma de sus cifras. Como 1000 ≤ n ≤ 9999 y n = s 3 , resulta 11 ≤ s ≤ 21 (1) Si n = xyzt, tenemos: 1000x + 100y + 10z + t = s 3 (2) x + y + z + t = s restando queda: 999x + 99y + 9z = s 3 - s (3) cuyo segundo miembro ha de ser múltiplo de 9 (por serlo el primero) y, habida cuenta de que s 3 - s = (s - 1) s (s + 1) y por (1), sólo hay tres valores de s 3 - s que son múltiplos de 9: 16·17·18; 17·18·19 y 18·19·20 sustituimos en (3) y analizamos cada caso. 1º 999x + 99y + 9z = 16·17·18 ⇔ 111x + 11y + z = 544 resulta inmediatamente x = 4; y = 9; z = 1, valores que llevados a (2) con s = 17 se obtiene t = 3 y finalmente n = 4913 2º 999x + 99y + 9z = 17·18·19 ⇔ 111x + 11y + z = 646 de donde x = 5; y = 8; z = 3, valores que llevados a (2) con s = 18 se obtiene t = 2 y finalmente n = 5832 3º 999x + 99y + 9z = 18·19·20 ⇔ 111x + 11y + z = 760 resulta x = 6; y = 8; z = 6, valores que llevados a (2) con s = 19 resulta una contradicción. Resumiendo, las únicas soluciones son 4913 y 5832 3.- Se considera el triángulo ABC y su circunferencia circunscrita. Si D y E son puntos sobre el lado BC tales que AD y AE son, respectivamente, paralelas a las tangentes en C y en B a la circunferencia circunscrita, demostrar que: BE CD AB AC = 2 2 Solución: Los triángulos ∆ABC y ∆ADC son semejantes pues tienen los tres ángulos iguales ya que: ∠ADC = ∠BCM = ∠BAC (la primera igualdad por ser AC y CM paralelas y la segunda por ser ∠BCM ángulo semiinscrito) y el ángulo ∠ACD es común. Estableciendo la proporcionalidad entre sus lados, resulta: ( ) CD AC AC BC CD BC AC = ⇔ ⋅ = 2 1 De modo análogo los triángulos ∆ABC y ∆ABE son semejantes pues: ∠AEB = ∠EBM = ∠BAC y el ángulo ∠ABE es común. Estableciendo la proporcionalidad entre sus lados, resulta: ( ) BE AB AB BC BE BC AB = ⇔ ⋅ = 2 2 Dividiendo las igualdades (1) y (2) se obtiene el resultado. B A C E D M 4.- Hallar las tangentes de los ángulos de un triángulo sabiendo que son números enteros positivos. Solución. Sean α, β, γ los tres ángulos y supongamos α ≤ β ≤ γ . Si fuera γ π ≥ 2 , tendría que ser α π < 4 y entonces tg α no es entero. Si tg α > 1, entonces α ≥ arc tg 2 > arc tg 3 3 = π , imposible porque α + β + γ = π. Por tanto tg α = 1 y β + γ = 3 4 π , con lo que: ( ) tg tg tg tg tg β γ β γ β γ + = − = + − 1 1 relación que operada se convierte en: (tg β -1)(tg γ -1) =2 de donde, por ser enteros positivos, se sigue tg β = 2 y tg γ = 3. Existe una visualización “sin palabras” de la solución: arc tg 1 + arc tg 2 + arc tg 3 = π. α β γ 5.- Hallar todas las funciones f N N : → estrictamente crecientes y tales que: f(n + f(n)) = 2 f(n) para n = 1, 2, 3, ... Solución: Supongamos f(1) = b. Entonces, f(1 + b) = 2b, como f es estrictamente creciente, se tiene: b = f(1) < f(1 +1 ) < ….< f(1+b) = 2b = b + b. y resulta que f(1), f(2),….f(1+ b) son b + 1 naturales, distintos, el primero vale b y el último 2b, por tanto han de ser consecutivos. resulta entonces: f(1) = b, f(2) = 1 + b, f(3) = 2 + b,……, f(1 + b) = b + b. En general, para n > 1, si f(n) = c , f(n + c) = 2c = c + c y resulta que: c = f(n) < f(n + 1) <.…< f(n + c) = c + c y los números f(n), f(n + 1), ….., f(n + c) son consecutivos. Así pues, f(n) = n - 1 + f(1) 6.- Determina los valores de n para los que es posible construir un cuadrado de n × n ensamblando piezas del tipo: Solución: Evidentemente n 2 debe ser múltiplo de 4 y, por tanto n necesariamente es par. Si n = 4k podemos dividir cualquier cuadrado n × n en k 2 sub-cuadrados del tipo 4 × 4 cada uno de los cuales lo podemos rellenar en la forma señalada en la figura de la izquierda. Queda sólo considerar el caso n = 4k + 2. Veamos que en ese caso la repuesta es negativa. Supongamos que fuera posible. Si pintamos cada cuadradito alternativamente de blanco y negro como en un tablero de ajedrez, hay dos posibilidades para cada pieza: Sea a el número de piezas del tipo de las de la izquierda y b el número de piezas del tipo de las de la derecha. Tenemos: ( ) ( ) a b k k k k + = + = + = + + 4 2 4 2 1 4 4 1 2 2 2 luego a + b ha de ser impar. Por otra parte, como hay tantas casillas blancas como negras, se tiene: 3a + b = 3b + a ⇔ a = b, de donde a + b = 2a ha de ser par en contradicción con lo anterior. B N N N B B B N OLIMPIADA MATEMÁTICA ESPAÑOLA XXXV Olimpiada Matemática Española Fase nacional 1999 (Granada) Primera sesión 1.- Las rectas t y t’, tangentes a la parábola de ecuación y = x 2 en los puntos A y B, se cortan en el punto C. La mediana del triángulo ABC correspondiente al vértice C tiene longitud m. Determinar el área del triángulo ABC en función de m. 2.- Probar que existe una sucesión de enteros positivos a 1 , a 2 ,…, a n , … tal que a 1 2 + a 2 2 +…….+ a n 2 es un cuadrado perfecto para todo entero positivo n. 3.- Sobre un tablero en forma de triángulo equilátero como se indica en la figura; se juega un solitario. Sobre cada casilla se coloca una ficha. Cada ficha es blanca por un lado, y negra por el otro. Inicialmente, sólo una ficha, que está situada en un vértice, tiene la cara negra hacia arriba; el resto de las fichas tiene la cara blanca hacia arriba. En cada movimiento se retira sólo una ficha negra del tablero y se da la vuelta a cada una de las fichas que ocupan una casilla vecina. Casillas vecinas son las que están unidas por un segmento. Después de varios movimientos ¿será posible quitar todas las fichas del tablero? Segunda sesión 4.-. Una caja contiene 900 tarjetas, numeradas del 100 al 999. Se sacan al azar (sin reposición) tarjetas de la caja y se anota la suma de los dígitos de cada tarjeta extraída. ¿Cuál es la menor cantidad de tarjetas que se deben sacar, para garantizar que al menos tres de esas sumas sean iguales? 5.- El baricentro del triángulo ABC es G. Denotamos por las distancias desde G a los lados a, b y c respectivamente. Sea el radio de la circunferencia inscrita. Probar que :i) ii) 6.- Se divide el plano en un número finito de regiones N mediante tres familias de rectas paralelas. No hay tres rectas que pasen por un mismo punto. ¿Cuál es el mínimo número de rectas necesarias para que N>1999? XXXV OLIMPÍADA MATEMÁTICA ESPAÑOLA FASE NACIONAL Primera Sesión Granada, 12 de Marzo de 1999 Problema 1. Las rectas t y t’, tangentes a la parábola de ecuación y = x 2 en los puntos A y B, se cortan en el punto C. La mediana del triángulo ∆ABC correspondiente al vértice C tiene longitud m. Determinar el área del triángulo ∆ABC en función de m. Solución: Sean A(a, a 2 ) ; B(b, b 2 ) . Las ecuaciones de t y t' son: t: y = 2ax - a 2 , t': y = 2bx - b 2 y su intersección C es:       + ab , b a C 2 . La mediana CM está en la recta: 2 b a x + = , paralela al eje OY. Las coordenadas de M son:         + + 2 2 2 2 b a , b a . Tenemos: ( ) 2 2 b a CM m − = = y si h es la altura del triángulo ∆BMC resulta: 2 2 m a b h = − = Poniendo [ ] XYZ para denotar el área del triángulo de vértices X,Y,Z queda finalmente: [ ] [ ] 2 2 2 1 2 2 3 m m m BMC ABC = = = Problema 2. Probar que existe una sucesión de enteros positivos a 1 , a 2 ,…, a n , … tal que a 1 2 + a 2 2 +…….+ a n 2 es un cuadrado perfecto para todo entero positivo n. Solución: Lo haremos por inducción sobre n, para n = 2 basta tomar a 1 = 3; a 2 = 4 con 3 2 + 4 2 = 5 2 . Supongamos que a 1 2 + a 2 2 +…….+ a n 2 = k 2 . Veamos que podemos encontrar un entero positivo a n+1 tal que 2 2 1 2 p a k n = + + . En efecto, ( )( ) 1 1 2 1 2 2 − + + − + = − = n n n a p a p a p k . Pongamos 1 1 + + − = + = n n a p b ; a p a . X Y A B C M t t' Tenemos: 2 ; 2 1 b a a b a p n − = + = + ; 2 2 2 b a b a k − ⋅ + = . La última expresión exige que a y b son de la misma paridad. Distinguiremos dos casos 1.- a y b son pares, entonces k 2 = 4m . Tomado a = 2m; b = 2 queda: 1 4 1 1 4 1 2 1 2 − = − = + = + = + k m a ; k m p n 2.- a y b son impares, entonces k 2 = 2m + 1. Tomando a = 2m +1, b = 1 queda: 2 1 1 2 1 1 2 1 2 − = = + − = + = + k m a ; k m p n En ambos casos hemos encontrado a n+1 entero verificando el enunciado. Segunda Sesión Granada, 13 de Marzo de 1999 Problema 3. Sobre un tablero en forma de triángulo equilátero como se indica en la figura; se juega un solitario. Sobre cada casilla se coloca una ficha. Cada ficha es blanca por un lado, y negra por el otro. Inicialmente, sólo una ficha, que está situada en un vértice, tiene la cara negra hacia arriba; el resto de las fichas tiene la cara blanca hacia arriba. En cada movimiento se retira sólo una ficha negra del tablero y se da la vuelta a cada una de las fichas que ocupan una casilla vecina. Casillas vecinas son las que están unidas por un segmento. Después de varios movimientos ¿será posible quitar todas las fichas del tablero? Solución: En el tablero, hay casillas de tres tipos : vértice, lado, o interiores. Cada una de ellas tiene, respectivamente, dos, cuatro o seis casillas vecinas. Si pudiéramos retirar todas las fichas del tablero, habría un momento en que quedaría sobre él una única ficha negra. Esa ficha era inicialmente blanca, luego ha tenido que cambiar de color un número impar de veces. Pero esto es imposible, porque una ficha se vuelve cada vez que se retira una ficha vecina, y ninguna ficha tiene un número impar de casillas vecinas. Problema 4. Una caja contiene 900 tarjetas, numeradas del 100 al 999. Se sacan al azar (sin reposición) tarjetas de la caja y se anota la suma de los dígitos de cada tarjeta extraída. ¿Cuál es la menor cantidad de tarjetas que se deben sacar, para garantizar que al menos tres de esas sumas sean iguales? Solución: Hay 27 posibles resultados para la suma de dígitos (de 1 a 27). Las sumas 1 y 27 sólo se puede obtener de un modo (100 y 999) En el caso más desfavorable al sacar 52 (27 + 25) tarjetas todas repetirán suma dos veces y en la siguiente (extracción 53) una de ellas aparecerá por tercera vez. Por tanto el número pedido es 27 + 25 + 1 = 53. Problema 5. El baricentro del triángulo ∆ABC es G. Denotamos por c b g , g , a g las distancias desde G a los lados a, b y c respectivamente. Sea r el radio de la circunferencia inscrita. Probar que: i) 3 2 g , 3 2 g , 3 2 c b r r r g a ≥ ≥ ≥ ii) 3 ≥ + + r g g g c b a Solución 1 (del autor del problema): i) Es sabido que uniendo G con cada vértice, se forman tres triángulos BGC de base a y altura g a , AGC de base b y altura g b y AGB de base c y altura g c de la misma área. Por tanto, llamando S al área de ABC: a·g a = b·g b = c·g c = 2 3 S (1) Por otra parte sabemos que r·(a + b + c) = 2S (basta unir el incentro con los tres vértices y quedan tres triángulos de bases a, b, c y altura común r). Sustituyendo 2S en (1), y despejando queda: g r a b c a g r a b c b g r a b c c a b c = + + = + + = + + 3 3 3 ; ; (2) y por la desigualdad triangular (b + c ≥ a) , resulta : a b c a b c a + + = + + ≥ 1 2 , de donde g r a ≥ 2 3 y de modo análogo para g b y g c . b) De (2) , haciendo los inversos y sumando resulta: ( ) ( ) ( ) 1 1 1 3 3 3 3 g g g a r a b c b r a b c c r a b c r a b c + + = + + + + + + + + = finalmente, aplicando la desigualdad entre las medias aritmética y armónica: g g g g g g r r g g g r a b c a b c a b c + + ≥ + + = = ⇔ + + ≥ 3 3 1 1 1 3 3 3 Nota.- Sumando las tres desigualdades de a) sólo obtenemos g g g r a b c + + ≥ 2 Solución 2 (de Ramón José que mereció mención especial) i) Consideremos los puntos M A , H A , G A como indica la figura. Pondremos h A a la altura correspondiente a A, p el semiperímetro y S el área de ABC. g b g c g a A B C G a b c Los triángulos AM A H A y GM A G A son semejantes siendo la razón de semejanza 3 (propiedad del baricentro sobre cada mediana). Entonces h A = 3 g A (1) Por la desigualdad triangular: 1 2 2 ≤ ⇔ ≥ ⇔ ≥ ⇔ ≥ + p a a p a p a c b multiplicando por h A y teniendo en cuenta (1) queda: p S g p ah g A A A 3 2 3 ≥ ⇔ ≥ finalmente, como S = pr resulta r g A 3 2 ≥ . Análogamente obtendríamos las correspondientes desigualdades para g B y g C . ii) Usaremos la desigualdad 2 1 ≥ + x x que se deduce de la obvia ( ) 0 1 2 ≥ − x . (Consideraremos siempre x positivo). Tenemos entonces: 6 ≥       + +       + +       + c a a c b c c b a b b a Sumando 3, ordenando y operando resulta: 9 1 1 1 1 1 1 1 1 1 9 1 1 1 ≥       + + +       + + +       + + ⇔ ≥ + + + + + + + + c b a c c b a b c b a a b c a c c b a b c a b a sacando factor común, dividiendo por 3 y poniendo 2p = a + b + c, queda: 3 3 2 3 2 3 2 ≥ + + c p b p a p (2) Por otra parte, como C c B b A a h g ; h g ; h g = = = 3 3 3 , resulta c g b g a g S c b a 3 3 3 2 = = = Despejando 3a, 3b y 3c y sustituyendo en (2), queda: ( ) 3 ≥ + + S p g g g c b a Finalmente usando de nuevo S = pr, resulta 3 ≥ + + r g g g c b a Problema 6. Se divide el plano en un número finito de regiones N mediante tres familias de rectas paralelas. No hay tres rectas que pasen por un mismo punto. ¿Cuál es el mínimo número de rectas necesarias para que N>1999? Solución: g A h A A B C M A G G A H A Supongamos que hay x rectas en la primera familia, y en la segunda y z en la tercera. Las x rectas de la primera familia determinan x + 1 regiones. La primera recta de la segunda familia determina en el plano (x +1)·2 regiones, la segunda (x +1)·3..... la y -ésima determina (x + 1)(y + 1) regiones. La primera recta de la tercera familia es cortada por las x + y rectas existentes en x + y + 1 partes y cada una de estas partes divide a en dos a cada región existente de modo que el número de regiones se incrementa en x + y + 1 regiones. Cada recta de la tercera familia aumenta las regiones existentes en la misma cantidad; luego el número total de regiones N vale: ( )( ) ( ) 1 1 1 1 1 + + = + + + + + + = + + + + + = m n yz xz xy z y x y x z y x N con z y x n + + = y yz xz xy m + + = . Tenemos: ( ) ( ) ( ) ( ) 2 2 2 2 2 2 2 2 2 2 1 z y x y x x z z y z y x m + + ≤ − + − + − − + + = , entonces 3 3 2 2 2 2 2 2 n m m m z y x n ≤ ⇔ ≥ − + + = y 1 3 1 2 + + ≤ + + = n n m n N . Para n = 76, 2002 1 3 2 2 > + + n n . Así, si z y x n + + = = 76 con x = 26, y = 25, z = 25, resulta: m = 1925 y N = 2002. OLIMPIADA MATEMÁTICA ESPAÑOLA XXXVI Olimpiada Matemática Española Fase nacional 2000 (Palma de Mallorca) Primera sesión 1.- Sean los polinomios: P(x) = x 4 + ax 3 + bx 2 + cx + 1; Q(x) = x 4 + cx 3 + bx 2 + ax + 1. Halla las condiciones que deben cumplir los parámetros reales a, b y c (a distinto de c) para que P(x) y Q(x) tengan dos raíces comunes y resuelve en ese caso las ecuaciones P(x) = 0; Q(x) = 0. 2.- La figura muestra un plano con calles que delimitan 12 manzanas cuadradas. Una persona P va desde A hasta B y otra Q desde B hasta A. Ambas parten a la vez siguiendo caminos de longitud mínima con la misma velocidad constante. En cada punto con dos posibles direcciones a tomar, ambas tienen la misma probabilidad. Halla la probabilidad de que se crucen. 3.- Dos circunferencias secantes C 1 y C 2 de radios r 1 y r 2 se cortan en los puntos A y B. Por B se traza una recta variable que corta de nuevo a C 1 y C 2 en dos puntos que llamaremos P r y Q r respectivamente. Demuestra la siguiente propiedad: Existe un punto M, que depende sólo de C 1 y C 2 , tal que la mediatriz del segmento P r Q r pasa por M. Segunda sesión 4.- Encuentra el mayor número entero N que cumpla las siguientes condiciones : a) tiene sus tres cifras iguales. b) es suma de números naturales consecutivos comenzando en 1, es decir, existe un natural n tal que: Nota: es la parte entera de x. 5.- Tomemos cuatro puntos situados en el interior o el borde de un cuadrado de lado 1. Demuestra que al menos dos de ellos están a distancia menor o igual que 1. 6.- Demuestra que no existe ninguna función que cumpla: f(f(n)) = n + 1. Olimpiada Española 2000 Problema 1. Sean los polinomios: P(x) = x 4 + ax 3 + bx 2 + cx + 1; Q(x) = x 4 + cx 3 + bx 2 + ax + 1. Halla las condiciones que deben cumplir los parámetros reales a, b y c (a ≠ c) para que P(x) y Q(x) tengan dos raíces comunes y resuelve en ese caso las ecuaciones P(x) = 0; Q(x) = 0. Solución de Virginia García Madurga de Zaragoza. Las raíces comunes a ambos polinomios serán raíces de la diferencia: P(x) - Q(x) = (a -c) x 3 + (c - a) x Resolvemos la ecuación P(x) - Q(x) = 0, sacando primero x factor común: ( ) ( ) [ ] 0 a c x c a x 2 = − + − Las tres raíces son: 0, 1 y -1, entre ellas tienen que estar las raíces comunes Como 0 no es raíz ni de P(x) ni de Q(x), las dos raíces comunes tiene que ser 1 y -1. Sustituyendo estos valores en P(x) y Q(x) obtenemos el sistema: ¹ ´ ¦ = − + − = + + + 0 c b a 2 0 c b a 2 que nos da las condiciones: b = -2 a = -c Los polinomios quedan en la forma: P(x) = x 4 + ax 3 - 2x 2 - ax + 1 Q(x) = x 4 - ax 3 - 2x 2 + ax + 1 Para resolver las ecuaciones P(x) = 0, Q(x) = 0, separamos por Ruffini las raíces conocida 1 y -1 y quedan las ecuaciones en la forma: P(x) = (x + 1)(x - 1) (x 2 + ax - 1) = 0 Q(x) =(x + 1)(x - 1) (x 2 - ax - 1) = 0 Resolviendo las ecuaciones de segundo grado queda finalmente: Soluciones de P(x) = 0: x = 1; x = -1; 2 4 a a x ; 2 4 a a x 2 2 + − − = + + − = Soluciones de Q(x) = 0: x = 1; x = -1; 2 4 a a x ; 2 4 a a x 2 2 + − = + + = Problema 2. La figura muestra un plano con calles que delimitan 12 manzanas cuadradas. Una persona P va desde A hasta B y otra Q desde B hasta A. Ambas parten a la vez siguiendo caminos de longitud mínima con la misma velocidad constante. En cada punto con dos posibles direcciones a tomar, ambas tienen la misma probabilidad. Halla la probabilidad de que se crucen. Solución de Fernando Cruz Robledillo (Madrid 2). Definamos un sistema de coordenadas con origen en A y unidad el lado de un cuadrado. Como P y Q recorren caminos de longitud mínima, P sólo puede ir a la derecha o arriba y Q a la izquierda o abajo. Todos los caminos tienen longitud 7, P y Q sólo se podrán encontrar entre el 3º y el 4º movimiento, se han marcado en rojo todas las posibles posiciones de P tras el tercer movimiento y en verde las de Q. Caso 1. P llega a (0, 3). La probabilidad de que P llegue a (0, 3) es: 8 1 2 1 2 1 2 1 = Sólo se puede cruzar con Q si éste está en (1, 3) lo que sucede también con probabilidad 8 1 2 1 2 1 2 1 = P está obligado a pasar a (1, 3) pero Q pasa a (0, 3) con probabilidad 2 1 . La probabilidad de que se crucen entre (0, 3) y (1, 3) es: 7 2 1 2 1 8 1 8 1 = Caso 2. P llega a (1, 2). La probabilidad de que P llegue a (1, 2) es 8 3 2 1 3 3 = | ¹ | \ | (hay tres modos de llegar (1, 2)). Sólo se puede cruzar con Q si éste está en (1, 3) o en (2, 2). Distingamos ambos casos: a) Q llega a (1, 3) con probabilidad 8 1 , entonces se cruzarán entre (1, 2) y (1, 3) si P se mueve hacia (1, 3) y Q hacia (1, 2) ambos movimientos con probabilidad 2 1 . La probabilidad de cruzarse es 8 2 3 2 1 2 1 8 1 8 3 = b) Q llega a (2, 2) con probabilidad 8 3 , entonces se cruzarán entre (1, 2) y (2, 2) si P se mueve hacia (2, 2) y Q hacia (1, 2) ambos movimientos con probabilidad 2 1 . La probabilidad de cruzarse es 8 2 9 2 1 2 1 8 3 8 3 = Caso 3. P llega a (2, 1). A B A B Procediendo de modo análogo, la probabilidad de cruzarse entre los puntos (2, 1) y (2, 2) es: 8 2 9 y la de cruzarse entre (2, 1) y ( 3, 1) es 8 2 9 . Caso 4. P llega a (3, 0). La probabilidad de cruzarse entre (3, 0) y (3, 1), es 8 2 3 y la de cruzarse entre (3, 0) y (4, 0) es 7 2 1 . La probabilidad pedida es la suma de todos los caso, resulta: 256 37 2 1 2 3 2 9 2 9 2 9 2 3 2 1 7 8 8 8 8 8 7 = + + + + + + Problema 3. Dos circunferencias secantes C 1 y C 2 de radios r 1 y r 2 se cortan en los puntos A y B. Por B se traza una recta variable que corta de nuevo a C 1 y C 2 en dos puntos que llamaremos P r y Q r respectivamente. Demuestra la siguiente propiedad: Existe un punto M, que depende sólo de C 1 y C 2 , tal que la mediatriz del segmento P r Q r pasa por M. Solución de Luis Emilio García Martínez (Valencia U. Politécnica): Sea O el punto medio del segmento M 1 M 2 . demostraré que todas las mediatrices de los segmentos P r Q r pasan por el simétrico de B respecto de O. Sean ε = ∠P r BM 1 ; γ = ∠M 1 BM 2 , Entonces: ∠M 2 BQ r = 180º - (γ + ε) y como el triángulo M 2 BQ r es isósceles, ∠BM 2 Q r = 180º - 2 ∠M 2 BQ r = -180º + 2 (γ + ε) y por tanto, ∠MM 2 Q r = 180º - γ + ∠BM 2 Q r = 180º - γ - 180º + 2 (γ + ε) = γ + 2 ε De modo análogo, por ser el triángulo P r M 1 B isósceles, se tiene: ∠ P r M 1 B = 180º - 2ε y ∠P r M 1 M = 360º - (∠P r M 1 B + 180º - γ) = 360º - 180º +2ε -180º + γ = 2ε + γ Resulta que para cualquier posición de la recta variable los triángulos MM 1 Pr y MM 2 Qr son iguales y por tanto MP r = MQ r y M está en la mediatriz de P r Q r . Como M no depende de la recta variable queda probada la propiedad del enunciado. Problema 4. Encuentra el mayor número entero N que cumpla las siguientes condiciones : a) | ¹ | \ | 3 N E tiene sus tres cifras iguales. C 1 C 2 M 1 M 2 A B M O Q r P r ε ε γ b) | ¹ | \ | 3 N E es suma de números naturales consecutivos comenzando en 1, es decir, existe un natural n tal que | ¹ | \ | 3 N E = 1 + 2 + ....+ (n-1) + n . Nota: E(x) es la parte entera de x. Solución de Roberto Alonso Pérez del País Vasco. Condición a): 9 k 1 ; N k ; k · 111 3 N E z ≤ ≤ ∈ ∀ = | ¹ | \ | = Condición b): ( ) 2 z 8 1 1 n 0 z 2 n n 2 1 n n z n ... 3 2 1 3 N E z 2 + + − = ⇒ = − + ⇒ + = ⇒ + + + + = | ¹ | \ | = (la otra raíz es negativa). Juntando las dos condiciones, queda: 2 k · 111 · 8 1 1 n + + − = Como n es natural, el radicando ha ser cuadrado perfecto lo que ocurre sólo para k = 6 que sustituido en la expresión anterior resulta n = 36. Recuperando la condición a): 1998 N 2001 666 3 N 667 666 6 · 111 3 N E z > > ⇒ > > ⇒ = = | ¹ | \ | = Por tanto el mayor N que cumple a) y B es N = 2000 Problema 5. Tomemos cuatro puntos situados en el interior o el borde de un cuadrado de lado 1. Demuestra que al menos dos de ellos están a distancia menor o igual que 1. Solución de Manuel Pérez Molina del Alicante. Vamos a demostrarlo por reducción al absurdo. Supongamos que distribuimos 4 puntos en el cuadrado de manera que cada una de las seis distancias se mayor que 1. Entonces hay dos posibilidades: a) Los cuatro puntos forman un cuadrilátero convexo. b) Los cuatro puntos forman un cuadrilátero no convexo. Veamos ambos casos: a) sean α, β, γ, δ los ángulos del cuadrilátero convexo. Sabemos que α + β + γ + δ = 360º. Además cualquier pareja de puntos del interior (o frontera) del cuadrado están a una distancia d ≤ 2 ya que el diámetro de dicho cuadrado es 2 . De la condición α + β + γ + δ = 360º, se deduce que necesariamente uno de los ángulos ha de ser mayor o igual que 90º, digamos por ejemplo α ≥ 90º. Tenemos (ver figura): 1 P P j i > , i ≠ j luego P 1 P 2 P 3 P 4 α β γ δ α ⋅ − + = cos P P P P 2 P P P P P P 3 2 2 1 2 3 2 2 2 1 2 3 1 como el cuadrilátero es convexo, 90º ≤ α ≤ 180º y por tanto cos α ≤ 0 y en consecuencia: 2 P P 2 P P P P P P 3 1 2 3 2 2 2 1 2 3 1 > ⇒ > + ≥ lo que es imposible. b) Si se forma un cuadrilátero no convexo podemos elegir tres de los cuatro puntos formando un triángulo de modo que el cuarto punto sea interior. Supongamos que el punto interior es P 4 . Cada lado de dicho triángulo es menor o igual que 2 (diámetro del cuadrado) y por tanto estará contenido en un triángulo equilátero de lado 2 , y circunradio 1 3 2 3 2 2 3 2 < = . Si su centro es C, P 4 estará en el interior de uno de los tres triángulos que resultan de unir C con cada vértice y la distancia de P 4 a uno de los vértices será menor o igual que el circunradio, es decir menor que 3 2 y por tanto menor que 1. hemos encontrado un par de puntos a distancia menor o igual que 1. Por último si tres puntos están alineados se reduce al caso b) y si los cuatro puntos están alineados llamando x 1 , x 2 , x 3 a las distancias entre puntos consecutivos, tenemos: 2 x x x 3 2 1 ≤ + + y por el principio del palomar, uno de ellos, digamos x 1 , cumple: 1 3 2 x 1 < ≤ . Problema 6. Demuestra que no existe ninguna función f : N → N que cumpla: f(f(n)) = n + 1. Solución de Alberto Suárez Real de Oviedo. Supongamos que exista ( ) ( ) 1 n n f f | N N : f + = → . Se tiene que f(0) = a ∈ N. Por el enunciado: ( ) ( ) ; 1 0 f f = ( ) ( ) ( ) 1 a f 0 f f = = del mismo modo, f(1) = a + 1, f(a + 1) = 2, f(2) = a + 2,........ Supongamos que f(n - 1) = a + n - 1, entonces f( a + n -1) = a + n luego hemos probado por inducción que ( ) ( ) ( ) n a 2 n a f n f + = + = entonces, N 2 1 a 1 n n a 2 ∉ = ⇒ + = + hemos llegado a una contradicción y la condición supuesta es falsa con lo que queda demostrado la inexistencia de la función f. 2 C OLIMPIADA MATEMÁTICA ESPAÑOLA XXXVII Olimpiada Matemática Española Fase nacional 2001 (Murcia) Primera sesión 1.- Probar que la gráfica del polinomio P(x) es simétrica respecto del punto A(a, b) si y sólo si existe un polinomio Q(x) tal que: P(x) = b + (x -a)Q((x-a) 2 ) 2.- Sea P un punto en el interior del triángulo ABC, de modo que el triángulo ABP verifica: AP =BP Sobre cada uno de los otros dos lados de ABC se construyen exteriormente triángulos BQC y CRA, ambos semejantes al triángulo ABP cumpliendo: BQ = QC y CR = RA Probar que los puntos P, Q, C y R o están alineados o son los vértices de un paralelogramo. 3.- Se tienen cinco segmentos de longitudes a 1 , a 2 , a 3 , a 4 y a 5 tales que con tres cualesquiera de ellos es posible construir un triángulo. Demostrar que al menos uno de esos triángulos tiene todos sus ángulos agudos. Segunda sesión 4.- Los números enteros desde 1 hasta 9 se distribuyen en las casillas de una tabla 3x3. Después se suman seis números de tres cifras: los tres que se leen en filas de izquierda a derecha y los tres que se leen en columnas de arriba abajo. ¿Hay alguna distribución para la cual el valor de esa suma sea 2001? 5.- ABCD es un cuadrilátero inscrito en una circunferencia de radio 1 de modo que AB es un diámetro y el cuadrilátero admite circunferencia inscrita. Probar que: . Probar que: . 6.- Determinar la función f : N ® N (siendo N = {1,2,3,...} el conjunto de los números naturales) que cumple, para cualesquiera s, n Î N, las siguientes condiciones: f (1) = f (2 s ) = 1 y si n < 2 s , entonces f (2 s + n) = f (n) + 1. Calcular el valor máximo de f (n) cuando n£ 2001. Hallar el menor número natural n tal que f (n) = 2001. 1 Olimpiada Española 2001 Primera sesión 1.- Prueba que la gráfica del polinomio P es simétrica respecto del punto ) , ( b a A sí y sólo sí existe un polinomio Q tal que: ), ) (( ) ( ) ( 2 a x Q a x b x P − − + = para todo . R x ∈ Solución: Supongamos primero que exista el polinomio P que cumple las condiciones requeridas. Sea h a x = − ó . h a x + = Entonces :    + = + − = − ) ( ) ( ) ( ) ( 2 2 h hQ b h a P h hQ b h a P y , 2 ) ( ) ( b h a P h a P = + + − para todo . R h∈ Lo que significa que la gráfica de P es simétrica respecto del punto ). , ( b a A Sea ). ( ) ( ) ( , h R h a P x P h a x = + = + = La condición b h a P h a P = + + − 2 ) ( ) ( es equivalente a , 2 ) ( ) ( h h R h R = + − porque ). ( ) ( h R h a P − = − Para , ... ) ( 1 0 n n h a h a a h R + + + = la condición anterior se escribe de la forma: b h a h a a a h a h a a n n n n n 2 ) 1 ( ... ... 2 2 1 0 1 0 = − + − + − + + + + es decir , ... 2 2 0 b h a h a a m m = + + + para cada . R h∈ n n m = par, n n m 1 − = impar. Se deduce que . , 0 .... 0 4 2 b a a a a m = = = = = Por tanto ahora se tiene que ... ) ( 3 3 1 + + + = h a h a b h R y así existe un polinomio Q tal que ), ( ) ( 2 h hQ b h R + = para algún polinomio . Q Por último ). ) (( ) ( ) ( ) ( 2 a x Q a x b h R x P − − + = = 2.- Sea P un punto, en el interior del triángulo ABC, de modo que el triángulo ABP es isósceles. Sobre cada uno de los otros dos lados de ABC se construyen exteriormente triángulos BCQ y CAR, ambos semejantes al triángulo ABP. Probar que los puntos P, Q, C y R o están alineados o son los vértices de un paralelogramo. Solución: Los triángulos ABC y PBQ son semejantes pues tienen un ángulo igual ∠ABC = ∠PBQ y los lados que lo forman proporcionales: BQ BP a c = De modo análogo, ABC es semejante a APR , por tanto PBQ y APR son semejantes (y al ser PB = PA son iguales). En particular: ∠ARP = ∠ACB y ∠BQP = ∠ACB Llamando α = ∠BAP = ∠ABP, resulta: ∠QPR = 360º − (180 – 2α) − (A + B) = 180º + 2α - (180º − ∠ACB ) = 2α + ∠ACB ∠QCR = ∠ACB + 2α ∠PRC = 180º − 2α −∠ARP = 180º - 2α − ∠ACB ∠PQC =180º − 2α −∠BQP = 180º - 2α − ∠ACB Las cuatro igualdades establecen que los dos pares de ángulos opuestos del cuadrilátero PQCR son iguales y es un paralelogramo. La alineación es un caso particular y se producirá cuando ∠ACB + 2α = 180º, es decir cuando 2 º 180 ACB −∠ = α . c a b A B C P Q R 2 3.- Están dados 5 segmentos de longitudes a 1 , a 2 , a 3 , a 4 y a 5 tales que con tres cualesquiera de ellos es posible construir un triángulo. Demuestra que al menos uno de esos triángulos tiene todos sus ángulos agudos. Solución: Supongamos que 5 4 3 2 1 0 a a a a a ≤ ≤ ≤ ≤ < . Si ningún triángulo es acutángulo, tendríamos: (3) a (2) a (1) 2 5 2 4 2 3 2 4 2 3 2 2 2 3 2 2 2 1 a a a a a a a ≤ + ≤ + ≤ + Pero (desigualdad triangular): a 5 <a 1 + a 2 , luego 2 1 2 2 2 1 2 5 2 a a a a a + + < (4) Sumando las desigualdades (1),(2),(3) y (4) tenemos: 2 1 2 2 2 1 2 5 2 4 2 3 2 5 2 4 2 3 2 2 2 1 2 2 2 a a a a a a a a a a a a + + + + + < + + + + es decir, 2 1 2 3 2 2 2 a a a a < + Como 3 2 a a ≤ , resulta 2 1 2 3 2 2 2 2 2 2 a a a a a < + ≤ , y por tanto a 2 < a 1 , en contradicción con la ordenación inicial. 3 Segunda sesión 4.- Los números enteros desde 1 hasta 9 se distribuyen en las casillas de una tabla 3x3. Después se suman seis números de tres cifras: los tres que se leen en filas y los tres que se leen en columnas. ¿Hay alguna distribución para la cual el valor de esa suma sea 2001? Solución Consideremos la distribución: a b c d e f g h i Resulta: S = abc + def + ghi + adg + beh + cfi = 100 (a + c + f + b + a + d + g) + 10(d + e + f + b + e + h) + (g + h + i + c + f + i) = 200 a + 110b + 101c + 110d + 20e + 11f + 101g + 11h + 2i Módulo 9 tenemos: S = 2(a + b + c+....h + i) = 2.45 = 0 Como 2001 no es múltiplo de 9, no habrá ninguna distribución para la que la suma indicada tome el valor 2001. 5.- ABCD es un cuadrilátero inscrito en una circunferencia de radio 1 de modo que AB es un diámetro y el cuadrilátero admite circunferencia inscrita. Probar que: 4 5 2 − ≤ CD Solución: Sea O el centro de la semicircunferencia. Pongamos a = BC; b = AD; p = CD; 2α = ∠BOD; 2β = ∠AOD; 2γ = ∠COD. La condición necesaria y suficiente para que ABCD admita una circunferencia inscrita es: p + 2 = a + b (1) Como 2α + 2β + 2γ = 180º, entonces β = 90 − ( α + β) y además: γ α − γ α = γ + α = β = γ = α = sen sen sen b sen p sen a 2 cos cos 2 ) cos( 2 2 ; 2 ; 2 Vamos a expresar la condición (1) en función del ángulo α y el dato p que determina por completo el cuadrilátero. 2 4 4 1 cos 2 2 p p − = − = γ , de donde: α α psen p b − − = cos 4 2 sustituyendo en (1), queda: a b p O B A C D α β 2γ 4 α − α − + α = + psen p sen p cos 4 2 2 2 o lo que es lo mismo: ( ) 2 2 cos 4 2 + = − + − p sen p p α α (2) Por tanto, existirá circunferencia inscrita para los valores de p que hagan compatible la ecuación (2) en la incógnita α. Puede expresarse el seno en función del coseno y estudiar el discriminante de la ecuación de segundo grado que se obtiene, pero es más rápido interpretar la ecuación (2) como el producto escalar de los vectores ( ) α α sen u , cos r de módulo 1 y ( ) p p v − − 2 , 4 2 r . La condición (2) queda: 2 cos + = p v δ r (3) siendo δ el ángulo formado por los vectores u r y v r . Para que (3) sea compatible debe cumplirse ( ) 2 2 2 4 2 p p v p − + − = ≤ + r , elevando al cuadrado y operando queda: 0 4 8 2 ≤ − + p p Las raíces de la ecuación son 4 5 2 − ± = p . Como p es positivo la condición final es: 4 5 2 0 − ≤ ≤ p 6.- Determinar la función f : N → N (siendo N = {1,2,3,...} el conjunto de los números naturales) que cumple, para cualesquiera s, n ∈ N, las dos siguientes condiciones: a) f (1) = 1, f (2 s ) = 1. b) Si n < 2 s , entonces f (2 s + n) = f (n) + 1. Calcular el valor máximo de f (n) cuando n ≤ 2001. Hallar el menor número natural n tal que f (n) = 2001. Solución Para cada número natural n definimos f (n) como la suma de las cifras de la expresión de n escrito en base 2. Está claro que esta función f cumple las condiciones a) y b). Además, es la única función que las cumple, porque el valor de f (n) viene determinado por las condiciones a) y b). Probamos esa afirmación por inducción sobre n. Si n = 1 o n = 2 s , f (n) = 1. Supongamos n > 1, n ≠ 2 s y que es conocido f (m) para todo m < n; se puede escribir n = 2 s + m con m < 2 s tomando 2 s la mayor potencia de 2 que es menor que n; entonces f (n) = f (m) + 1. Ahora, es fácil resolver las dos cuestiones que nos plantean: En el primer caso, se trata de ver cuántos unos puede tener como máximo un número menor o igual que 2001 escrito en base 2. Ese número, escrito en base 2, es, obviamente, 1111111111, que corresponde a n = 1023 = 2 10 - 1. Es f (n) = 10. En el segundo caso, razonando de manera análoga, se observa que la respuesta es n = 2 2001 -1. XXXVIII Olimpiada Matemática Española Fase nacional 2002 (La Rioja) Primera sesión (5 de abril) 1.- Hallar todos los polinomios P(t) de una variable, que cumplen: P(x 2 - y 2 ) = P(x + y)·P(x – y) para todos los números reales x e y. 2.-En un triángulo ABC, A’ es el pie de la altura relativa al vértice A y H el ortocentro. a)Dado un número real positivo k tal que , encontrar la relación entre los ángulos B y C en función de k. b) Si B y C son fijos, hallar el lugar geométrico del vértice A para cada valor de k. 3.-La función g se define sobre los números naturales y satisface las condiciones: ·g(2) = 1 ·g(2n) = g(n) ·g(2n + 1) = g(2n) + 1 Sea n un número natural tal que 1 £ n £ 2002. Calcula el valor máximo M de g(n). Calcula también cuántos valores de n satisfacen g(n) = M. Segunda sesión (6 de abril) 4.-Sea n un número natural y m el que resulta al escribir en orden inverso las cifras de n. Determinar, si 4.-Sea n un número natural y m el que resulta al escribir en orden inverso las cifras de n. Determinar, si existen, los números de tres cifras que cumplen 2m + S = n, siendo S la suma de las cifras de n. 5.-Se consideran 2002 segmentos en el plano tales que la suma de sus longitudes es la unidad. Probar que existe una recta r tal que la suma de las longitudes de las proyecciones de los 2002 segmentos dados sobre r es menor que . 6.-En un polígono regular H de 6n + 1 lados (n entero positivo), R vértices se pintan de rojo y el resto de azul. Demostrar que el número de triángulos isósceles que tienen sus tres vértices del mismo color no depende del modo de distribuir los colores en los vértices de H. Soluciones Olimpiada Española 2002 Problema 1 Hallar todos los polinomios P(t) de una variable, que cumplen: 2 2 ( ) ( ) · ( ) P x y P x y P x y − = + − para todos los números reales x e y. La ecuación funcional dada ) ( ) ( ) ( 2 2 y x P y x P y x P − + = − (*) es equivalente a la ecuación funcional ) ( ) ( ) ( v P u P uv P = (**) con el cambio de variables y x u + = y , y x v − = para todos . , R v u ∈ Poniendo 0 = = v u en (**) se obtiene , )) 0 ( ( ) 0 ( 2 P P = de donde 1 ) 0 ( = P ó . 0 ) 0 ( = P Sea , 1 ) 0 ( = P haciendo 0 = v en (*) se deduce que ) 0 ( ) ( ) 0 ( P u P P = para todo , R u ∈ es decir . 1 ) ( ≡ u P Sea ahora . 0 ) 0 ( = P Entonces ), ( ) ( u uQ u P = siendo ) (u Q un polinomio de grado una unidad inferior al grado de ). (u P Fácilmente se comprueba que ) (u Q satisface la ecuación funcional (**). Por tanto n u u P = ) ( con . N n ∈ Recíprocamente se comprueba sin dificultad que 1 ) ( ≡ x P y n x x P = ) ( con N n ∈ satisfacen la ecuación funcional inicial (*). También puede hacerse sin el cambio de variable haciendo x = y = 0 se llega a 2 (0) ( (0)) P P = . Además está la solución trivial P(x) ≡ 0. Problema 2 En un triángulo ABC, A’ es el pie de la altura relativa al vértice A y H el ortocentro. a) Dado un número real positivo k tal que ' ' AA k HA = , encontrar la relación entre los ángulos B y C en función de k. b) Si B y C son fijos, hallar el lugar geométrico del vértice A para cada valor de k. a) Tenemos: ' cos BA c B = ; ' tg ' ctg ' HA HBA C BA = = , ' sen AA c B = . De donde: ' sen tg ·tg ' cos ctg AA c B k B C k HA c B C = = ⇔ = (1) b) Poniendo a = BC, tomando unos ejes con origen en el punto medio de BC y eje OX sobre el lado BC, resulta , 0 ; , 0 2 2 a a B C     −         y llamando A(x, y), la condición (1) se escribe: 2 2 2 · 4 2 2 y y a k y k x a a x x   = ⇔ = −     − + que, una vez operada resulta: 2 2 2 2 1 4 4 x y a ka + = (2) A B C A' B' C' H ecuación de una elipse en la que distinguimos dos casos: Si k < 1, elipse con eje mayor sobre OX, semidistancia focal = 1 2 a k − y semieje mayor = 2 a Si k >1, elipse con eje mayor sobre OY, semidistancia focal = 1 2 a k − y semieje mayor = 2 a n . Problema 3 La función g se define sobre los números naturales y satisface las condiciones: • g(2) = 1 • g(2n) = g(n) • g(2n + 1) = g(2n) + 1 Sea n un número natural tal que 1 ≤ n ≤ 2002. Calcula el valor máximo M de g(n). Calcula también cuántos valores de n satisfacen g(n) = M. Para cualquier natural n, consideramos su representación binaria, ( ) 1 1 1 0 1 0 2 2 2 2 k k k k k n a a a a a a a − − = + + + + = K K , donde a j = 0 o 1. Probaremos por inducción que ( ) 0 k j j g n a = = ∑ por inducción sobre k: Para k = 0 es cierto: ( ) ( ) ( ) 2 1 1 1 g g = = . Supuesto cierto para k, hay dos casos para k + 1: ( ) ( ) ( ) ( ) ( ) ( ) ( ) ( ) 1 0 1 2 0 2 0 1 0 1 2 0 2 0 0 2· , 1 1 2· 1 k k k j j k k k j j g a a a g a a a a g a a a g a a a a = = = = = + = + ∑ ∑ K K K K donde se han aplicado las propiedades de g y la hipótesis inductiva. Entonces g(n) es el número de unos de n escrito en base 2. Como 2 11 = 2048 > 2002 > 1024 = 2 10 , resulta M = 10. Hay cinco soluciones de g(n) = 10: 1023, 1535, 1791, 1919 y 1983. B C F F' A H A' B C F F' A H A' Problema 4 Sea n un número natural y m el que resulta al escribir en orden inverso las cifras de n. Determinar, si existen, los números de tres cifras que cumplen 2m + S = n, siendo S la suma de las cifras de n. n = abc = c + 10b + 100a; m = cba = 100c + 10b + a 2 m + S = n nos da: 200c + 20b + 2a + (a + b + c) = 100a + 10b + c, es decir 200c + 11b – 97a = 0. Por lo tanto, 200c – 97a es múltiplo de 11. Módulo 11: 2(c + a) es 0, y como mcd (2,11) = 1, resulta que a + c es congruente con 0 módulo 11. Módulo 9: 2(c + a + b) congruente con 0, y c + a + b congruente con 0. Por la primera congruencia, c + a = 0, o bien c + a = 11. Si c + a = 0, entonces a = c = 0 y no hay solución por ser números de tres cifras. Si c + a = 11, entonces b = 7. Por lo tanto, 200c – 97a es múltiplo de 7. Trabajando módulo 7: 4c + a es congruente con 0 módulo 7, es decir; 4c + a = 0, 7, 14, 21, 28, 35, 42. Como a + c = 11, tenemos que 3c debe tomar uno de los valores -11, -4, 3, 10, 17, 24, o 31 y ser múltiplo de 3. Luego c = 1 o c = 8. Si c = 1, entonces a = 10, imposible. Si c = 8, a = 3. Pero n = 378 no es solución y no existen números con las condiciones pedidas. Problema 5 Se consideran 2002 segmentos en el plano tales que la suma de sus longitudes es la unidad. Probar que existe una recta r tal que la suma de las longitudes de las proyecciones de los 2002 segmentos dados sobre r es menor que 3 2 . Cada segmento determina dos vectores de igual módulo y sentido opuesto. Consideramos los 2 x 2002 = 4004 vectores así obtenidos y los ordenamos por sus direcciones entre 0 y 2π respecto de un sistema de referencia ortonormal arbitrario. Construimos ahora un polígono convexo de 4004 lados “uniendo” los vectores uno a continuación del otro, a partir de uno cualquiera dado. Claramente el perímetro de este polígono es 2 Además es un polígono centrado y simétrico, respecto de un punto O (la prueba de esta observación es sencilla y es necesario hacerla). Tomamos entonces uno de los lados más próximos a O; sea d el segmento perpendicular a ese lado y a su opuesto que pasa por el centro O. La proyección del polígono sobre la recta que contiene a este segmento es d y por tanto la suma de las proyecciones sobre la recta anterior es también d. Por otra parte la circunferencia de centro O y radio 2 d está totalmente contenida en el interior del polígono y entonces su circunferencia es menor que el perímetro del polígono. Es decir: 2 < π d y . 3 2 2 < < π d Falta considerar el caso trivial de que todos los segmentos tengan la misma dirección en cuyo caso ni hay polígono pero tomando la recta perpendicular a la dirección común sale d = 0. Problema 6 En un polígono regular H de 6n + 1 lados (n entero positivo), r vértices se pintan de rojo y el resto de azul. Demostrar que el número de triángulos isósceles que tienen sus tres vértices del mismo color no depende del modo de distribuir los colores en los vértices de H. Debido a que el número de lados del polígono H deja de resto uno al dividirse entre seis, cada diagonal y cada lado del mismo pertenece sólo (exactamente) a tres triángulos isósceles distintos (la demostración es sencilla y se debe hacer). Denotamos por AA, AR y RR los números de segmentos que son lados y diagonales cuyos extremos respectivamente están coloreados ambos de azul, de azul y de rojo o ambos de rojo. Análogamente denotamos por AAA, AAR, ARR y RRR el número de triángulos isósceles cuyos vértices son los tres azules, dos azules y uno rojo, uno azul y el otro rojo o los tres rojos y ninguno azul, respectivamente. Entonces , 3 3 AAR AAA AA + × = × porque cada diagonal o lado de H pertenece a tres triángulos isósceles y los triángulos isósceles con tres vértices azules tienen tres lados con sus dos extremos azules. Los triángulos isósceles con dos vértices azules tienen sólo un lado con sus extremos de color azul y los triángulos isósceles con menos de dos vértices azules no tiene ningún lado con los extremos del mismo color azul. Análogamente establecemos: ARR AAR RA × + × = × 2 2 3 y . 3 3 RRR ARR RR × + = × (se deben probar estas dos nuevas relaciones). Las tres relaciones obtenidas conducen a que: 1 2 AAA RRR RR AA RA + = + − × = , 2 1 ) 1 ( 2 1 ) 1 ( 2 1 A R A A R R × × − − × × + − × × donde A es el número de vértices azules, . 1 6 R n A − + = Esto completa la prueba. Se observa que el resultado es también cierto si el polígono H tiene 5 6 + n lados. XXXIX Olimpiada Matemática Española Fase nacional 2003 (Canarias) Primera sesión (3 de marzo) 1.- Probar que para cualquier primo p distinto de 2 y 5 existe un múltiplo de p cuyas cifras son todas nueves. Por ejemplo si p = 13, 999999 = 13·76923 2.-¿Existe algún conjunto finito de números reales M que contenga al menos dos elementos distintos y que cumpla la propiedad de que para dos números a, b cualesquiera de M, el número 2a - b 2 sea también un elemento de M? 3.- Las alturas del triángulo ABC se cortan en el punto H. Se sabe que AB = CH. Determinar el valor del ángulo BCA Segunda sesión (4 de marzo) 4.-Sea x un número real tal que x 3 + 2x 2 +10x = 20. Demostrar que tanto x como x 2 son irracionales. 5.- ¿Cuáles son las posibles áreas de un hexágono con todos los ángulos iguales y cuyos lados miden 1, 2, 3, 4, 5 y 6 en algún orden? 6.- Ensartamos 2n bolas blancas y 2n bolas negras formando una cadena abierta. Demuestra que, se haga en el orden que se haga, siempre es posible cortar un segmento de cadena exactamente con n bolas blancas y n bolas negras. Olimpiada Española 2003 Problema 1 Probar que para cualquier primo p distinto de 2 y 5 existe un múltiplo de p cuyas cifras son todas nueves. Por ejemplo si p = 13, 999999 = 13·76923 Solución de Luis Hernández Corbato de Madrid. Sea a i el número compuesto por i nueves 99 9 i i a = 678 K . Supongamos que p ∃ tal que | i p a i ∀ ∈ / para probar por contradicción el enunciado. Considérense en dicho caso los números { } 1 2 , , p a a a K , en este conjunto sabemos que no hay ningún ( ) 0 i a p ≡ (por hipótesis) . Por tanto al haber p números y sólo p – 1 restos posibles módulo p, se sabe que existen m, n tales que ( ) 0 m n a a p − ≡ . Suponemos sin pérdida de generalidad que m > n y: | 99 9 99 9 99 900 0 ·10 m n m n n n m n m n p a a a − − − = − = = 678 678 678678 K K K K Como 2 y 5 | 10 2 ·5 | n n n m n p p p p a − ≠ ≠ ⇒ = ⇒ / y como a m – n pertenece al conjunto escogido por ser m – n < n y m – n ≥ 1 se ha llegado a una contradicción. Por ende: tal que | i i p a p a ∀ ∃ y el enunciado queda probado. Problema 2 ¿Existe algún conjunto finito de números reales M que contenga al menos dos elementos distintos y que cumpla la propiedad de que para dos números a, b cualesquiera de M, el número 2a - b 2 sea también un elemento de M? Solución de Víctor González Alonso de Burgos. Como M es finito, necesariamente estará acotado. Pongamos [ ] , M x y ⊂ , con x = Mín M e y = Máx M. Supongamos x ≤ 0: Tenemos x ≤ 0 ⇒ 2x ≤ x ⇒ 2x – k 2 < x (k cualquier número de M). Esto contradice que x sea el mínimo de M. Por tanto x > 0 y 0 < x < y. En cualquier casi debe ser: (1) x ≤ 2x – y 2 ≤ y y además (2) x ≤ 2y – y 2 ≤ y . De (1) se desprende que : x ≤ 2x – y 2 ⇒ 0 ≤ x – y 2 ⇒ y 2 ≤ x <y ; que sólo se cumple si y ∈ (0, 1). De (2) obtenemos que: 2y – y 2 ≤ y ⇒ y – y 2 ≤ 0 ⇒ y ≤ y 2 ; y esto sólo es cierto si [1, ) y ∈ +∞ . Como (1) y (2) deben cumplirse a la vez, no existe ningún y ∈ R que pueda ser máximo de M por lo que no estaría acotado y no sería finito. Problema 3 Las alturas del triángulo ABC se cortan en el punto H. Se sabe que AB = CH. Determinar el valor del ángulo ∠BCA. Solución de Ibón Arregui Bilbao del País Vasco. Ángulo C < 90º. A C B A' C' H α 90 − α α Llamaremos A’ al punto en que la altura de A corta al lado BC del triángulo ABC, y C’ al punto donde la altura de C corta al lado AB del triángulo ABC. El ángulo ∠CHA’ es igual al ángulo ∠AHC’. En el triángulo CA’H, el ángulo ∠CA’H es recto, por tanto el ángulo ∠HCA’ es 90º - α . En el triángulo AHC’ el ángulo ∠HC’A es recto, por tanto el ángulo ∠HAC’ es 90º - α . El ángulo ∠HAC’ es igual al ángulo ∠A’AB del triángulo A’AB que es rectángulo por tanto el ángulo ∠A’BA es α. De aquí concluimos que los triángulos CHA’ y A’AB son semejantes, y como CH = AB , son triángulos iguales de donde obtenemos que AA’ = CA’, por tanto el valor de tg C = 1, y C = 45º. Ángulo C > 90º. Procediendo de modo análogo el ángulo ∠A’CH es igual al ángulo ∠C’CB . En el triángulo C’CB el ángulo ∠CA’H es recto, por tanto el ángulo ∠A’HC es 90º - α y en el triángulo CC’B el ángulo ∠CC’B es recto y por tanto ∠C’BC es 90º - α. El triángulo AA’B es rectángulo en A’ y por ello ∠BAA’ es α. Entonces los triángulos AA’B y A’CH son semejantes y tienen la hipotenusa igual, luego son iguales y deducimos AA’ = A’C , entonces la tangente de C vale –1 y C = 135º. Finalmente, si fuese C = 90º, C coincide con H y CH = 0. Como AB ≠ 0, este valor de C no es válido. Problema 4 Sea x un número real tal que x 3 + 2x 2 +10x = 20. Demostrar que tanto x como x 2 son irracionales. Solución: Primero veamos que x no puede ser entero. Esto puede hacerse teniendo en cuenta que si lo fuese, sería un divisor de 20, y basta probar los 8 divisores para comprobar que ninguno verifica la ecuación. Otro modo de verlo es comprobar que f(x) = x 3 + 2x 2 +10x – 20 es estrictamente creciente (su derivada es positiva para todo x) y además f(1) = 13 y f(2) = 36. Luego no hay raíces enteras. Veamos que x no puede ser racional por reducción al absurdo. Supongamos que x = p/q con q ≥ 1 y p/q irreducible. Entonces ( ) 3 3 2 2 2 2 20 10 2 20 10 2 p q q p qp q q qp p = − − = − − Si q fuera estrictamente mayor que 1, la igualdad anterior estaría en contradicción con la hipótesis de que p/q es irreducible. Por tanto q = 1, x sería entero lo que es imposible. Luego x es irracional. Para la irracionalidad de x 2 basta ver que ( ) 2 2 2 2 20 2 10 20 2 10 x x x x x x − + = − ⇒ = + , y si x 2 fuese racional, también los sería x en contra de lo probado. Problema 5 ¿Cuáles son las posibles áreas de un hexágono con todos los ángulos iguales y cuyos lados miden 1, 2, 3, 4, 5 y 6 en algún orden? Solución: A C B C' A' B' H α α α 90 − α 90 − α La idea es prolongar los lados para formar un triángulo equilátero. a + b + c + d + e + f = 21 l = a + b + c = c + d + e = e + f + a 3l =21 + a + c + e, por tanto l = 7 + (a + c + e) / 3 El valor más pequeño de a + c + e es 6 y el más grande 15 así que 9 ≤ l ≤ 12 Si a + c + e = 6, entonces son: (a, c, e) = (1, 2, 3) y (b, c, d) = (4, 5, 6) Si a + c + e = 9 el único caso posible es: (a ,c, e) = (1, 3, 5) y (b, c, d) = (2, 4, 6) Si a + c + e = 12 el único caso posible es (a ,c, e) = (2, 4, 6) Si a + c + e = 15 el único posible es (4, 5, 6). Como el área del triángulo de lado l es 2 3 4 l y la del hexágono es ( ) 2 2 2 2 3 4 l a c e é ù - + + ê ú ë û , las áreas posibles son: Si a + c + e = 6, entonces l = 9 y el área 4 3 67 Si a + c + e = 9, entonces l = 10 y el área 4 3 65 Si a + c + e = 4, entonces l =11 y el área 4 3 65 Si a + c + e = 5, entonces l = 12 y el área 4 3 67 Problema 6 Ensartamos 2n bolas blancas y 2n bolas formando una cadena abierta. Demuestra que, se haga en el orden que se haga, siempre es posible cortar un segmento de cadena exactamente con n bolas blancas y n bolas negras. Solución de Mohammed Blanca Ruiz de Valencia. Tenemos la cadena con el total de 4n bolas, 2n blancas y 2n negras. Cogemos un grupo de un extremos con 2n bolas, este grupo tendrá x bolas negras e y bolas blancas, de forma que la diferencia es x – y = 2k para { } ,1 , , 0, 1, k n n n n ∈ − − − K K . Vamos moviéndonos de una en una posición hacia el extremo contrario, en cada movimiento la diferencia varía en 2 o no varía, es decir k aumente en 1, disminuye en 1 o no cambia. La diferencia varía en 2 si la bola que se deja y que se coge son de distinto color y no se mantiene si son del mismo color. La posición final, es decir en el otro extremo, tendrá las bolas al revés, x bolas blancas e y bolas negras con lo que la diferencia (blancas - negras) será ahora y – x = –2k, para el mismo k. Es decir que k pasa de una posición a su opuesta con el mismo valor absoluto. Como k sólo puede variar de 1 en 1 tiene que pasar por el cero ya que no se lo puede saltar. En el momento en que k = 0, x = y = n, c.q.d. Siempre se podrá cortar un segmento de longitud 2n con n bolas blancas y n bolas negras. e e a a c c f b d XL Olimpiada Matemática Española Fase nacional 2004 (Ciudad Real) Primera sesión (26 de marzo) 1.- Tenemos un conjunto de 221 números reales cuya suma es 110721. Los disponemos formando una tabla rectangular de modo que todas las filas y la primera y última columnas son progresiones aritméticas de más de un elemento. Probar que la suma de los elementos de las cuatro esquinas vale 2004 2.- ABCD es un cuadrilátero cualquiera, P y Q los puntos medios de las diagonales BD y AC respectivamente. Las paralelas por P y Q a la otra diagonal se cortan en O. Si unimos O con las cuatro puntos medios de los lados X, Y, Z y T se forman cuatro cuadriláteros, OXBY, OYCZ, OZDT y OTAX. Probar que los cuatro cuadriláteros tienen la misma área. · · 3.- Se representa por Z el conjunto de todos los enteros. Hallar todas las funciones f: Z → Z tales que para cualesquiera x, y enteros se verifica: f(x + f(y)) = f(x) – y Segunda sesión (27 de marzo) 4.- ¿Existe alguna potencia de 2 que al escribirla en el sistema decimal tenga todos sus dígitos distintos de cero y sea posible reordenar los mismos para formar con ellos otra potencia de 2?. Justificar la respuesta. 5.- Demostrar que la condición necesaria y suficiente para que, en el triángulo ABC, la mediana desde B sea dividida en tras partes iguales por la circunferencia inscrita en el triángulo, es 6.- Colocamos, formando una circunferencia, 2004 fichas bicolores: blancas por una cara y negras por la otra. Un movimiento consiste en elegir una ficha con la cara negra hacia arriba, y dar la vuelta a tres fichas: la elegida, la de su derecha y la de su izquierda. Supongamos que inicialmente hay una sola ficha con la cara negra hacia arriba. ¿Será posible, repitiendo el movimiento descrito, conseguir que todas las fichas tengan la cara blanca hacia arriba? ¿Y si tuviéramos 2003 fichas, entre las cuales exactamente una tiene al comienzo la cara negra hacia arriba? Olimpiada Española 2004 Problema 1 Tenemos un conjunto de 221 números reales e cuya suma es 110721. Los disponemos formando un rectángulo de modo que todas las filas y la primera y última columna son progresiones aritméticas de más de un elemento. Probar que la suma de los elementos de las cuatro esquinas vale 2004 Solución Denotaremos por j i a al elemento de la fila i-ésima y columna j-ésima del rectángulo Pongamos n para el número de filas, m para el de columnas y S para la suma de los n·m elementos. Con notación matricial queda: 1 2 1 1 1 1 2 2 2 2 1 2 m m m n n n a a a a a a M a a a | | | | = | | | \ ¹ L L L L L L L Sumando por filas y llamando S k a la suma de la fila k, resulta: 1 1 1 1 1 2 2 2 1 · 2 · 2 ....................... · 2 m m m n n n a a S m a a S m a a S m + = + = + = y sumando miembro a miembro queda: ( ) ( ) ( ) 1 1 1 1 1 1 2 1 2 1 2 1 1 · 2 4 m m m m m n n n n n m n m S S S S a a a a a a a a a a = + + + = + + + + + + + = + + + K K K 1 1 1 1 4 4·110721 2004 · 221 m m n n S a a a a n m + + + = = = Problema 2 ABCD es un cuadrilátero cualquiera, P y Q los puntos medios de las diagonales BD y AC respectivamente. Las paralelas por P y Q a la otro diagonal se cortan en O. Si unimos O con las cuatro puntos medios de los lados X, Y, Z y T se forman cuatro cuadriláteros, OXBY, OYCZ, OZDT y OTAX. Probar que los cuatro cuadriláteros tienen la misma área. Solución 1 (“oficial”). Bastará probar que el área de cada cuadrilátero es la cuarta parte del área total. La quebrada APC divide al cuadrilátero en dos partes de igual área pues AP es la mediana de ABD y PC lo es de CBD. La quebrada TPZ divide al cuadrilátero APCB (sombreado) en dos partes de igual área pues PT es mediana de APD y PZ es mediana de CPD. Tenemos ya probado que el área del cuadrilátero TPZD es la cuarta parte del área del cuadrilátero inicial. Finalmente TZ es paralela a OP por serlo ambas a AC; luego los triángulos TPZ y TOZ tienen la misma área y lo mismo les ocurre a los cuadriláteros TPZD y TOZD. Del mismo modo se probaría para los otros tres cuadriláteros. Solución 2 (de la concursante Elisa García Lorenzo) La fórmula de la superficie del cuadrilátero es: · · 2 AC BD sen S α = Además 2 AC ZT XY = = al ser ZT la paralela media del triángulo ACD y XY la paralela media del triángulo ABC. Igualmente: 2 BD XT ZY = = Para probar el enunciado bastará probar que: B A D C Q P O T X Y Z B A D C Q P O T X Y Z β β α α A B C D T Z Y X Q P T ' Z ' O · · 4 2 2 · · 2 · · 2 AC BD XT AO sen sen AC BD sen XT AO sen ACsen AOsen AQsen AOsen AQ AO sen sen α β α β α β α β β α = = = = = que es el teorema del seno en el triángulo AQO. Queda probado el enunciado por extensión de la demostración a los 4 cuadriláteros pequeños que resultan ser una cuarta parte de grande. Solución 3 (de Marco Castrillón López). Al ser OP paralela a AC, los triángulos OXY, PXY tienen la misma base e igual altura y por tanto la misma área. De ahí que los cuadriláteros OXBY, PXBY también tienen la misma área, pero el área de PXBY (en amarillo en la figura) es la cuarta parte del cuadrilátero inicial al ser semejantes con razón 2 del grande al pequeño. A B C D T Y X Z P Q O Problema 3 Se representa por Z el conjunto de todos los enteros. Hallar todas las funciones : , f Z Z → tales que para cualesquiera , x y enteros se verifica: ( ( )) ( ) . f x f y f x y + = − Solución: Primeramente observemos que ( ( )) ( ) . f x nf y f x ny + = − Para 0 n = es obvio, y por inducción suponiendo que para cada entero 1 n ≥ ( ( 1) ( )) ( ) ( 1) , f x n f y f x n y + − = − − entonces: ( ( )) ( ( 1) ( ) ( )) ( ( 1) ( )) ( ) ( 1) ( ) . f x n f y f x n f y f y f x n f y y f x n y y f x ny + = + − + = + − − = = − − − = − Análogamente se prueba para cada entero 1. n ≤ − Por tanto (1 (1) (1)) 0. f f f + ⋅ = Poniendo 2 1 (1) (1) 1 (1) 0, k f f f = + ⋅ = + > se tiene ( ) ( ( )) ( ) , f x f x f k f x k = + = − que es una contradicción. Deducimos que no existen funciones que satisfagan la condición requerida. Problema 4 ¿Existe alguna potencia de 2 que al escribirla en el sistema decimal tenga todos sus dígitos distintos de cero y sea posible reordenar los mismos para formar con ellos otra potencia de 2 ?. Justificar la respuesta Solución 1 (“oficial”): Supongamos que exista tal potencia de 2, es decir, que haya dos potencias de 2 cuyas expresiones decimales sólo difieran en el orden de colocación de los dígitos. Claramente ninguna de las dos potencias es divisible por 3, y ambas dejan el mismo resto cuando se dividen por 9. Esto último se debe a que el resto de un número al dividirse por 9 es congruente, módulo 9, con la suma de sus dígitos. Por otra parte la mayor de ambas potencias se obtiene de la menor multiplicando ésta por 2, 4 u 8 (de otra manera no tendrían ambas el mismo número de dígitos). Sin embargo al multiplicar la menor potencia de las dos por 2, 4 u 8, cambia el resto cuando se divide por 9. Los restos de las sucesivas potencias de 2 al dividirse por 9 forman una sucesión periódica. Efectivamente, los restos de: 2, 4,8,16, 32, 64,128, 256, 512,1024, 2048, 4096,..., son: 2, 4, 8, 7, 5,1, 2, 4, 8, 7, 5,1,... Esta sucesión tiene periodo 6, porque para todo n entero positivo 6 6 0 2 2 2 (2 2 ) 63 2 , n n n n + − = − = ⋅ y este número es divisible por 9, por lo que ambas potencias dejan el mismo resto. No es posible por tanto, reordenar los dígitos de una potencia de 2 para obtener otra potencia distinta de 2. Solución 2 (del concursante Lander Ramos Garrido). No existe ningún número que cumpla las condiciones del enunciado. En primer lugar, ambos deben tener las mismas cifras lo que implica que el número de cifras sea el mismo, así que el cociente entre ambos no debe ser mayor que 8, porque si fuera 16 se alteraría el número de cifras. Otra condición que han de cumplir es, obviamente, que la suma de sus cifras sea la misma. Como duplicar un número implica que la nueva suma de sus cifras sea el doble de la antigua menos 9x, donde x es el número de llevadas ya que a cada llevada restas 10 a un número y sumas 1 al siguiente. Para que la suma fuera igual tiene que cumplirse 2 9 0 y x − = siendo y la suma de las cifras antiguas. Entonces y debe ser múltiplo de 9 para que se cumpla la ecuación anterior ya que en caso contrario habría “medias llevadas”, absurdo. Si y es múltiplo de 9, según el criterio de divisibilidad, el número también debería ser múltiplo de 9, pero como estamos tratando potencias de 2, no habrá ningún número que cumpla esas características. Para 4 y 8 el proceso es parecido, las fórmulas serían: Para 4: (z son las llevadas en la segunda duplicación). ( ) ( ) 2 2 9 9 0 4 18 9 0 4 9 2 x y x x y z x y z − − = ⇔ − − = ⇔ = + Tampoco podría ser ya que 2y + z es un natural. Para 8: (a son las llevadas en la tercera duplicación). ( ) ( ) 2 4 9 2 9 0 8 9 4 8 x y z a x y z a − + − = ⇔ = + + . Y tampoco podría ser. Problema 5 Demostrar que la condición necesaria y suficiente para que, en el triángulo ABC, la mediana desde B sea dividida en tras partes iguales por la circunferencia inscrita en el triángulo, es 5 10 13 a b c = = Solución. a) la condición es necesaria. Sea ABC un triángulo tal que la mediana BK (K punto medio de AC) corte a la circunferencia inscrita en dos puntos, M y N, tales que BM MN NK x = = = Sea T el punto de tangencia del círculo inscrito con el lado BC. Las siguientes relaciones se verifican en cualquier triángulo: 2 2 2 2 2 2 2 4 a c b BT a c b BK + − = + − = (La primera se deduce sin más de BT + CT = a, BT – CT = c – b; la segunda –fórmula de Apolonio o de la mediana- se puede también obtener completando el triángulo ABC hasta obtener un paralelogramo ABCD). Entonces resulta 2 2 2 2 2 2 36 a c b x + − = (1) La potencia del vértice B respecto del círculo inscrito se puede escribir de dos maneras: 2 · BT BM BN = , con lo cual ( ) 2 2 8 a c b x + − = (2) Como, evidentemente, en el triángulo del problema, los puntos B y K están igualmente alejados del centro del círculo inscrito, resulta BC = KC, de donde 2 b a = Sustituyendo esta última igualdad en (1) y (2), obtenemos ( ) 2 2 2 2 2 18 , 8 c a x c a x − = − = y ya que 0, 0, c a x − ≠ ≠ resulta 9 13 , de donde 4 5 c a c c a a + = = − Por lo tanto, c a b B A C I T K M N 5 10 13 a b c = = . b) la condición es suficiente. No hay pérdida de la generalidad en suponer que a = 5, b = 10, c = 13. Sustituyendo los valores de los lados en las fórmulas utilizadas en la parte a), resulta 2 6 2, 16 · BK BT BM BN = = = y en el inradio 6 14 S r p = = (calculando S por la fórmula de Herón). El triángulo BCK es isósceles, así que la bisectriz del ángulo C es también altura. Sea H CI BK = ∩ ; consideremos el triángulo rectángulo BIT; entonces 2 2 2 2 2 47 4 14 BI r × = + = por otra parte, en BIH, 2 4 7 HI = , y finalmente en IHM, 2 2 2 2. HM r HI = − = Como H es el punto medio de MN, resulta 2 2 MN = , luego la mediana BK queda, en efecto, dividida en tres partes iguales por el círculo inscrito. c a b B A C I T K M N H Problema 6 Colocamos, formando una circunferencia, 2004 fichas bicolores: blancas por una cara y negras por la otra. Un movimiento consiste en elegir una ficha negra, y dar la vuelta a tres fichas: la elegida, la de su derecha y la de su izquierda. Supongamos que inicialmente hay una sola ficha con la cara negra hacia arriba. ¿Será posible, repitiendo el movimiento descrito, conseguir que todas las fichas tengan la cara blanca hacia arriba? ¿Y si tuviéramos 2003 fichas, entre las cuales exactamente una tiene al comienzo la cara negra hacia arriba? Solución Numeremos las fichas desde 1 hasta 2004: la 1 es negra y las restantes son blancas. Cada ficha inicialmente blanca debe ser “tocada” un número par de veces, para que al final del proceso siga teniendo la cara blanca hacia arriba. Cada movimiento posible cambia el número de fichas negras en un número impar: BNB pasa a NBN : el número de fichas negras aumenta en 1 NNB pasa a BBN : el número de fichas negras disminuye en 1 BNN pasa a NBB: el número de fichas negras disminuye en 1 NNN pasa a BBB: el número de fichas negras disminuye en 3 Como inicialmente hay exactamente una ficha negra, el número total de movimientos para tener las 2004 fichas con la cara blanca hacia arriba debe se impar. Designamos por x i el número de movimientos realizados eligiendo la ficha i (que debe ser negra). La ficha que ocupa el lugar i cambia de color en los movimientos en que la elegimos a ella (x i ), a la de su izquierda (x i-1 ) o a la de su derecha (x i+1 ). Por lo tanto, (x i-1 + x i + x i + 1) es el número de veces que hemos dado la vuelta a la ficha que ocupa el lugar i (2004+1 se identifica con 1, y 2003 +2 se identifica con 1) El número total de movimientos será: ( ) ( ) 1 2 3 4 2002 2003 2004 N x x x x x x x = + + + + + + + K Como 2004 es múltiplo de 3, N es la suma del número de veces que hemos dado la vuelta a las fichas en los lugares 2, 5 ... 3k+2, ... 2003, todas ellas blancas al principio: así que N, suma de números pares, debería ser par: contradicción, pues N es impar. Por lo tanto, no será posible conseguir que las 2004 fichas tengan la cara blanca hacia arriba. Con 2003 fichas si es posible: iniciando el movimiento sobre la ficha 1, (única negra al principio), y repitiéndolo sobre las fichas que ocupan los lugares 2 .....2001,2002 llegaríamos a la configuración NNN NNN .... NNN BB Eligiendo ahora las fichas que ocupan los lugares 2, 5 ... 3k+2.... 2000 tendríamos: BBB BBB .......... BBB BB en la que todas las fichas tendrían la cara blanca hacia arriba. XLI Olimpiada Matemática Española Fase nacional 2005 (Santiago de Compostela) Primera sesión (21 de marzo) 1.- Sean a y b enteros. Demostrar que la ecuación admite a lo sumo una solución entera. 2.- ¿Es posible colorear los puntos del plano cartesiano Oxy de coordenadas enteras con tres colores, de tal modo que cada color aparezca infinitas veces en infinitas rectas paralelas al eje Ox y tres puntos cualesquiera, cada uno de distinto color, no estén alineados? Justificar la contestación. 3.- Diremos que un triángulo es multiplicativo si el producto de las longitudes de dos de sus lados es igual a la longitud del tercer lado. Sea ABC...XYZ un polígono regular de n lados con todos sus lados de longitud 1. Las n – 3 diagonales que salen del vértice A dividen al triángulo ZAB en n – 2 triángulos más pequeños. Probar que cada uno de esos triángulos es multiplicativo. Segunda sesión (22 de marzo) 4.- Probar que para todo entero positivo n, la expresión decimal de es periódica mixta. 5.- Sean números reales cualesquiera. Probar que: mín 6.- En un triángulo de lados a, b, c el lado a es la media aritmética de b y c. Probar: a) 0º ≤ A ≤ 60º. b) La altura relativa al lado a es tres veces el inradio r. c) La distancia del circuncentro al lado a es R – r , siendo R el circunradio. -1- Olimpiada Española 2005 Problema 1 Sean a y b enteros. Demostrar que la ecuación ( )( )( ) 3 1 0 x a x b x − − − + = admite a lo sumo una solución entera. Solución. Sea el entero p una raíz, entonces: ( )( )( ) 3 1 x a x b x − − − + se anula para x = p, es decir ( )( )( ) 3 1 p a p b p − − − = − Distingamos varios casos 1.- ( ) 3 1 4 p p − = ⇒ = entonces para los otros factores tenemos dos posibilidades: ( ) ( ) 1 1 5 1 1 3 p a a p p b b p − = − ⇒ = + = ¹ ¦ ` − = ⇒ = − = ¦ ) sustituyendo queda la ecuación ( ) ( ) 2 3 2 3 5 1 11 39 44 0 x x x x x − − + = − + − = y una vez separada la raíz 4 resulta la ecuación 2 7 11 0 x x − + = que no tiene raíces enteras. ( ) ( ) 1 1 3 1 1 5 p a a p p b b p − = ⇒ = − = ¹ ¦ ` − = − ⇒ = + = ¦ ) idéntico al anterior. 2.- ( ) 3 1 2 p p − = − ⇒ = entonces para los otros factores tenemos dos posibilidades: ( ) ( ) 1 1 1 1 1 1 p a a p p b b p − = ⇒ = − = ¹ ¦ ` − = ⇒ = − = ¦ ) sustituyendo queda la ecuación ( ) ( ) 2 3 2 1 3 1 5 7 2 0 x x x x x − − + = − + − = y después de separar la raíz 2 resulta 2 3 1 0 x x − + = que no tiene raíces enteras. Finalmente, ( ) ( ) 1 1 3 1 1 3 p a a p p b b p − = − ⇒ = + = ¹ ¦ ` − = − ⇒ = + = ¦ ) sustituyendo queda la ecuación ( ) 3 3 2 3 1 9 27 26 0 x x x x − + = − + − = y después de separar la raíz 2 resulta 2 7 13 0 x x − + = que no tiene raíces. Problema 2 ¿Es posible colorear los puntos del plano cartesiano Oxy de coordenadas enteras con tres colores, de tal modo que cada color aparezca infinitas veces en infinitas rectas paralelas al eje Ox y tres puntos cualesquiera, cada uno de distinto color, no estén alineados? Justificar la contestación. Solución oficial. Probemos que tal coloración es posible. Pintemos el punto ( , ) x y de rojo si x y + es par, de blanco si x es impar e y es par y de azul si x es par e y es impar. Claramente se satisface la condición de que cada color aparezca infinitas veces en infinitas rectas paralelas al eje . OX -2- Supongamos ahora que 1 1 ( , ) x y sea rojo, 2 2 ( , ) x y sea blanco y 3 3 ( , ) x y sea azul. Entonces 2 1 x x − e 2 1 y y − tienen paridad opuesta y 3 2 x x − e 3 2 y y − son ambos impares. Por tanto: 2 1 3 2 3 2 2 1 ( )( ) ( )( ), y y x x y y x x − − ≠ − − con lo cual : 3 2 2 1 3 2 2 1 ( ) ( ) , ( ) ( ) y y y y x x x x − − ≠ − − lo que significa que tres puntos cualesquiera, cada uno de distinto color, no están alineados. Solución de Miguel Teixidó Román Consideremos una coloración de 2 d acuerdo con las siguientes reglas: -Azul si sus dos coordenadas son pares. -Verde si ambas coordenadas son impares. -Naranja cuando tienen una coordenada de cada paridad. Demostraremos que tal coloración cumple las condiciones requeridas. Observamos que tres puntos A, B, C de 2 están alineados si y sólo si existe k tal que AB k AC = uuur uuur . Caso 1. Supongamos que A azul, B verde y C naranja con la primera coordenada par están alineados, la condición AB k AC = uuur uuur sobre la primera componente queda: ( ) x x x x b a k c a − = − y reduciéndola módulo 2 resulta: ( ) 1 0 0 0 1 0· k k − = − ⇔ = contradicción que prueba que no existen tales puntos. Caso 2. Igual que el anterior con la primera coordenada de C impar y por tanto la segunda par. Razonando del mismo modo sobre la segunda componente tenemos: ( ) y y y y b a k c a − = − y módulo 2 resulta ( ) 1 0 0 0 1 0· k k − = − ⇔ = y la correspondiente contradicción. La segunda condición también se cumple: El azul se repite infinitamente en las rectas de la forma y k = para k par. El verde se repite infinitamente en las rectas de la forma y k = para k impar. El naranja se repite infinitamente en las rectas de la forma y k = para cualquier k . Problema 3 Diremos que un triángulo es multiplicativo si el producto de las longitudes de dos de sus lados es igual a la longitud del tercer lado. Sea ABC...XYZ un polígono regular de n lados con todos sus lados de longitud 1. Las n – 3 diagonales que salen del vértice A dividen al B A Z R Q P X Y -3- triángulo ZAB en n – 2 triángulos más pequeños. Probar que cada uno de esos triángulos es multiplicativo. Solución oficial. El ángulo formado pos dos diagonales consecutivas con un extremo en A es el mismo por inscrito en el mismo arco α. Sean PQ y QR dos segmentos adyacentes sobre el segmento ZB, determinados por tres diagonales consecutivas. Entonces 1 1 · · · 2 2 1 1 · · · 2 2 APQ AQR S h PQ AP AQ sen S h QR AQ AR sen α α = = = = donde h es la altura común a todos los triángulos. Por tanto · · AP AQ AQ AR PQ QR = , es decir, la razón r del producto de dos lados a la base es la misma para cualquier par de triángulos adyacentes, y por tanto es la misma para todos ellos. Pero el primero y el último son claramente multiplicativo al tener dos lados iguales y el tercero igual a 1. Se deduce que r = 1 y todos los triángulos son multiplicativos. Solución de Anas El Barkani que mereció una mención especial del jurado. Vamos a probar el enunciado demostrando que si un triángulo es multiplicativo, también lo es el colindante. Obsérvese primero que el triángulo ABQ es isósceles puesto que los ángulos A y B abarcan el mismo arco. Es obvio que el triángulo ABQ es multiplicativo pues 1 AB = . · ·1 QB AQ AB AQ AQ = = = (1) Ahora bien, si aplicamos el teorema de la bisectriz al triángulo APB tenemos que: · PQ QB AQ PQ AQ AP AP AB = = ⇒ = lo que prueba que APQ también es multiplicativo. c.q.d. Problema 4 Probar que para todo entero positivo n, la expresión decimal de 1 1 1 1 2 n n n + + + + es periódica mixta. Solución: Tenemos ( )( ) 2 1 1 1 3 6 2 1 2 1 2 n n n n n n n n + + + + = + + + + Sabemos que para que una fracción origine un decimal periódico mixto, una vez reducida debe tener en el denominador algún factor primo del conjunto { } 2, 5 y alguno que no sea ni 2 ni 5. Z A B C P Q -4- Veamos primero que la fracción anterior tiene en el denominador al menos una factor 2 más que en el numerador, en efecto Si n es par tenemos 2 n k = que una vez sustituido resulta: ( )( ) ( )( ) ( )( ) 2 2 2 1 1 1 3 6 2 12 12 2 6 6 1 1 2 1 2 2 2 1 2 2 2 2 1 1 n n k k k k n n n n n n k k k k k k + + + + + + + + = = = + + + + + + + + el numerador es impar y el denominados par. Si n es impar tenemos 2 1 n k = + que una vez sustituido resulta: ( )( ) ( )( )( ) 2 2 1 1 1 3 6 2 12 24 11 1 2 1 2 2 1 2 2 2 3 n n k k n n n n n n k k k + + + + + + = = + + + + + + + el numerador es impar y el denominados par. En ambos casos el denominador tiene al menos un factor 2 que no está en el numerador. Además la expresión ( )( ) 2 1 1 1 3 6 2 1 2 1 2 n n n n n n n n + + + + = + + + + muestra que el numerador no contiene el factor primo 3 (da resto 2 al dividirlo entre tres) mientras el denominador al ser producto de tres números consecutivos es múltiplo de tres. Problema 5 Sean , , , r s u v números reales cualesquiera. Probar que: mín { } 2 2 2 2 1 , , , . 4 r s s u u v v r − − − − ≤ Solución. Supongamos que los cuatro números 2 2 2 , , r s s u u v − − − y 2 v r − son mayores estrictamente que 1 . 4 Entonces 2 2 2 2 1 1 1 1 , 4 4 4 4 r s s u u v v r − + − + − + − > + + + pero esta expresión es equivalente a 2 2 2 2 1 1 1 1 0 2 2 2 2 r s u v | | | | | | | | > − + − + − + − | | | | \ ¹ \ ¹ \ ¹ \ ¹ que es una contradicción. Problema 6 En un triángulo de lados a, b, c el lado a es la media aritmética de b y c. Probar: a) 0º ≤ A ≤ 60º. b) La altura relativa al lado a es tres veces el inradio r. c) La distancia del circuncentro al lado a es R – r. Solución. a) Por la desigualdad triangular: 3 3 1 2 3 1 3 3 2 3 b c b b c b c b c b c b c c b c b c + ¹ ≤ + ⇔ ≤ ⇔ ≤ ¦ ¦ ⇒ ≤ ≤ ` + ¦ ≤ + ⇔ ≤ ⇔ ≥ ¦ ) Por el teorema de coseno: ( ) 2 2 2 2 2 3 3 2 2 cos cos 4 8 b c b c bc b c bc A A bc + + − = + − ⇒ = r d a R a c b h a B C A I T -5- dividiendo numerador y denominador por c 2 y llamando por comodidad de escritura b x c = , queda: ( ) 2 3 2 3 3 1 3 cos 8 8 4 8 x x A f x x x x − + = = = − + con 1 3 3 x ≤ ≤ . Fácilmente se comprueba que ( ) 1 3 1 3 f f | | = = | \ ¹ y que la derivada se anula en x = 1 donde hay un mínimo que vale ( ) 1 1 2 f = . También puede localizarse el mínimo sin recurrir a la derivada teniendo en cuenta la desigualdad de las medias: ( ) 2 3 2 3 3 1 3 1 3 1 1 3 cos ·2 8 8 4 8 4 8 4 8 x x A f x x x x x x − + | | = = = − + = − + + ≥ − + | \ ¹ con igualdad para x = 1. Resumiendo queda 1 cos 1 60º 0º 2 A A ≤ ≤ ⇔ ≥ ≥ . b) Designando A, B y C a los vértices opuestos a los lados a, b y c respectivamente, I al incentro y h a a la altura correspondiente al lado a como se indica en la figura, S al área y p al semiperímetro, tenemos: 3 3 1 2 2 3 1 2 2 2 a a a a b c ar S pr r ar ah r h S ah + + ¹ = = = ¦ ¦ ⇒ = ⇔ = ` ¦ = ¦ ) c) Pongamos d a a la distancia entre el circuncentro y el lado a. De una parte 2 2 2 4 a a d R = − (1) y de otra 2 2 A r r tg p a a = = − . Como 2 a R senA = y 1 2 2 2 A tg senA A tg + = , resulta: 2 2 2 2 2 4 1 2 2 4 4 4 r a a a R a r Rr r r r a + = = + ⇒ = − que sustituida en (1) queda: ( ) 2 2 2 2 2 2 2 4 a a a d R R Rr r R r d R r = − = − + = − ⇔ = − Solución de Elisa Lorenzo García. Los apartados a) y b) son esencialmente iguales a los de la solución oficial. Para el apartado c) se da la solución que sigue sin utilizar trigonometría. Q es la intersección de la bisectriz de A con la mediatriz de a que está en el punto medio del arco BC. Llamando x PB = , resulta -6- 2 3 4 b c a x y c b b y z x c z x + ¹ = + = ¦ ¦ − = + ⇒ = ` ¦ = + ¦ ) Aplicando el teorema de la bisectriz: c b BQ CQ = y 2 2 2 2 b c c bc BQ CQ a BQ b c + + + = = ⇒ = + además ' 4 b c BA + = por ser la mitad de a. Calculemos 2 2 3 3 ' ' 2 0 4 2 2 4 4 b c c bc c b b b c c c bc QA PQ BA BQ BQ BP c c b c b c + + − − + + + − = − − + = − + = − = − = + + de donde ' QA PQ = y como ' 90º IPQ QA S = = y ' IQP A QS = , resulta que los triángulos PIQ y A’SQ son iguales y ' IP A S r = = de donde queda finalmente: ' ' OA OS A S R r = − = − c a b A B C A' O I P S Q XLII Olimpiada Matemática Española Fase nacional 2006 (Sevilla) Primera sesión (24 de marzo) 1.- Sea un polinomio con coeficientes enteros. Demostrar que si existe un entero k tal que ninguno de los enteros es divisible por k, entonces no tiene raíces enteras. 2.- Las dimensiones de un ortoedro de madera son enteras. Pintamos toda su superficie (las seis caras), lo cortamos mediante planos paralelos a las caras en cubos de una unidad de arista y observamos que exactamente la mitad de los cubos no tienen ninguna cara pintada. Probar que el número de ortoedros con tal propiedad es finito. (Puede resultar útil tener en cuenta que ). . 3.- ABC es un triángulo isósceles con AB = AC. Sea P un punto cualquiera de la circunferencia tangente a los lados AB en B y a AC en C. Llamamos a, b y c a las distancias desde P a los lados BC, AC y AB respectivamente. Probar que: Segunda sesión (25 de marzo) 4.- Hallar todas las funciones que satisfacen la ecuación para todo par de números reales e positivos, siendo un número real positivo tal que 5.- Probar que el producto de cuatro naturales consecutivos no puede ser ni cuadrado ni cubo perfecto 6.- Las diagonales y de un cuadrilátero convexo se cortan en Denotamos por y a las áreas de los triángulos y del cuadrilátero respectivamente. Prueba que ¿Cuándo se alcanza la igualdad? Olimpiada Española 2006 Problema 1 Sea ( ) P x un polinomio con coeficientes enteros, demostrar que si existe un entero k tal que ninguno de los enteros ( ) ( ) ( ) 1 , 2 , , P P P k K es divisible por k, entonces ( ) P x no tiene raíces enteras. Solución. Por reducción al absurdo. Si n fuese una raíz, por una parte tenemos ( ) ( ) ( ) P x x n Q x = − y por otra siempre existen enteros q y r tales que n kq r = + , con 1 r k ≤ ≤ (basta hacer la división entera y en el caso de ser resto cero se rebaja el cociente en una unidad), entonces ( ) ( ) ( ) ( ) P r r n Q r kqQ r = − = − en contra de lo supuesto en el enunciado. Problema 2 Las dimensiones de un paralelepípedo de madera son enteras. Pintamos toda su superficie (las seis caras) y lo cortamos en cubos de una unidad de arista y observamos que exactamente la mitad de los pequeños cubos no tienen ninguna cara pintada. Probar que el número de paralelepípedos con tal propiedad es finito. (Puede resultar útil tener en cuenta que 3 1 1, 79... 1, 8 2 < ). Solución . Consiste en acotar el número de soluciones de la ecuación diofántica: ( )( )( ) 2 2 2 2 abc a b c = − − − y para ello suponiendo que a b c ≤ ≤ , se pone en la forma: 1 2 2 2 · · 2 a b c a b c − − − = , y como 3 3 2 1 2 1 2 1 2 2 2 a a a a a a − − −   ≤ < ⇒ < ≤     fácilmente se obtiene que 4 < a < 10. Fijado a se acotan de modo análogo los posibles valores de b: De una parte ( ) ( ) 2 2 2 2 2 · 2 2 2 2 a b c b a b a b c b a b − − − −   = ⇒ ≤ <   − −   y de ahí se sigue ( ) ( ) 2 5 1 2 2 2 2 6 a b a a b a − < < ≤ < − − con lo que para cada valor de a, b sólo puede tomar un conjunto finito de valores. Finalmente fijados a y b a lo sumo hay un valor para c que cumpla la ecuación inicial lo que prueba que el número de soluciones es finito. Problema 3 ABC es un triángulo isósceles con AB = AC. Sea P un punto cualquiera de la circunferencia tangente a los lados AB en B y a AC en C. Pongamos a, b y c a las distancias desde P a los lados BC, AC y AB respectivamente. Probar que: 2 · a b c = Solución: Pongamos m = PB; n = PC, Q, R y S las proyecciones de P sobre cada lado y sea P’ el punto diametralmente opuesto a P . Por la semejanza de los triángulos PBP’ y PBS se tiene: 2 2 2 m r m cr c m = ⇔ = (1) De modo análogo por la semejanza de PCP’ y PBC se cumple: 2 2 2 n r n br b n = ⇔ = (2) Por el teorema de los senos en PBC: ( ) 2 n sen PBC r ∧ = y en el triángulo rectángulo PQB: ( ) a sen PBC m ∧ = de donde 2 2 2 2 2 4 mn m n a a r r = ⇔ = y por (1) y (2) queda finalmente: 2 2 2 ·2 · 4 cr br a b c r = = Problema 4. Hallar todas las funciones : (0, ) f R ∞ → que satisfacen la ecuación ( ) ( ) 2 ( ) f x f y f f f xy x y λ λ     + =         para todo par de números reales x e y positivos, siendo λ un número real positivo tal que ( ) 1. f λ = Solución: Haciendo 1 x y = = en la ecuación funcional dada se tiene que 2 2 (1) ( ) 2 (1). f f f λ + = De este modo ( ) 2 (1) 1 0 f − = y así (1) 1. f = Sustituyendo ahora 1 y = en la ecuación funcional resulta ( ) (1) ( ) 2 ( ), f x f f f f x x λ λ   + =     que es equivalente a ( ) f x f x λ   =     para todo 0. x > Tomemos a continuación y x λ = y observemos que ( ) ( ) 2 ( ). f x f f f x f x x λ λ λ     + =         Entonces ( ) 1, f x f x λ   =     por lo que 2 ( ) 1 f x = para todo 0. x > m n a b c C B A P Q R S P' Sustituyendo x y t = = de nuevo en la ecuación funcional inicial se tiene que ( ) 2 2 2 ( ) f t f f t t λ   + =     y debido a ser el miembro de la izquierda positivo f es positivo y ( ) 1 f x = para todo 0. x > Se comprueba fácilmente que esta función constante e idénticamente igual a 1 es solución de la ecuación funcional del enunciado y además es la única. Problema 5 Probar que el producto de cuatro naturales consecutivos no puede ser ni cuadrado ni cubo perfecto. Solución. Si el producto ( ) ( )( ) 1 1 2 N n n n n = − + + fuese un cuadrado, basta ponerlo en la forma ( ) ( )( ) ( )( ) ( ) 2 2 2 2 1 1 2 2 1 1 N n n n n n n n n n n = − + + = + − + = + − − de donde se sigue una contradicción (no hay dos cuadrados consecutivos). Si N fuese cubo perfecto, podemos suponer que 2 n > (si n = 2 , N = 24). Distinguimos ahora dos casos: a) n impar, entonces n es primo con los otros tres factores y si N es cubo perfecto, también lo es ( )( )( ) 3 2 1 1 2 2 2 M n n n n n n = − + + = + − − pero si ( ) 3 3 3 2 2 2 2 1 n n n n n n > ⇒ < + − − < + y ya tenemos la contradicción pues entre dos cubos consecutivos no puede haber otro cubo. b) si n es par, 1 n + es impar y por tanto 1 n + es primo con el producto ( ) ( ) 3 2 1 2 2 M n n n n n n = − + = + − que también debe ser cubo perfecto. Finalmente, como ( ) 3 3 3 2 2 2 1 x x x x x x > ⇒ < + − < + se sigue la contradicción. Problema 6 Las diagonales AC y BD de un cuadrilátero convexo ABCD se cortan en . E Denotamos por 1 2 , S S y S a las áreas de los triángulos , ABE CDE y del cuadrilátero ABCD respectivamente. Prueba que 1 2 . S S S + ≤ ¿Cuándo se alcanza la igualdad? Solución: Denotando, como en el enunciado, la áreas de los triángulos BCE y DAE por 3 S y 4 S respectivamente, tenemos que probar que 1 2 1 2 3 4 . S S S S S S + ≤ + + + Elevando al cuadrado la anterior desigualdad, se obtiene la desigualdad equivalente 1 2 3 4 2 S S S S ≤ + (1). Sean ahora K y L los pies de las perpendiculares en la diagonal AC trazadas desde D y B respectivamente. Estos puntos K y L pueden estar dentro o fuera del segmento . AC Llamamos , , , . b BL d DK m AE n CE = = = = Entonces 1 2 3 1 1 1 1 1 , , , . 2 2 2 2 S mb S nd S nb S md = = = = Sustituyendo esta expresión en la desigualdad (1) se llega a 1 ( ), 2 mb nd nb md ⋅ ≤ + que es precisamente la desigualdad entre la media aritmética y la media geométrica de los dos productos nb y . md Esta última desigualdad se alcanza si y sólo si b m nb md d n = ⇔ = (2). Las rectas BL y DK son paralelas. Así , b BL BE d DK DE = = por la semejanza entre los triángulos BLE y . DKE La relación (2) se convierte en BE AE DE CE = (3). Y recíprocamente por la semejanza de triángulos (3) se verifica si y sólo si AB y CD son paralelos, es decir el cuadrilátero dado es un trapecio con los lados paralelos AB y CD. Esta es la condición para que se alcance (1). XLIII Olimpiada Matemática Española Fase nacional 2007 (Torrelodones) Primera sesión (23 de marzo) 1.- Sean cinco números positivos en progresión aritmética de diferencia d. Probar que 2.- Determinar todos los posibles valores enteros no negativos que puede tomar la expresión siendo y enteros no negativos tales que 3.- Sea O el circuncentro de un triángulo ABC. La bisectriz que parte de A corta al lado opuesto en P. Probar que se cumple: Segunda sesión (24 de marzo) 4.- ¿Cuáles son los números enteros positivos que se pueden obtener de exactamente 2007 maneras distintas, como la suma de al menos dos números enteros positivos consecutivos? ¿Cuál es el menor de todos ellos? Ejemplo: el número 9 se escribe exactamente de dos maneras distintas: 9 = 4 + 5 9 = 2 + 3 + 4 5.- Sea un número real positivo y un entero mayor que 1. Demostrar que 6.- Dada una semicircunferencia de diámetro AB = 2R, se considera una cuerda CD de longitud fija c. Sea E la 6.- Dada una semicircunferencia de diámetro AB = 2R, se considera una cuerda CD de longitud fija c. Sea E la intersección de AC con BD y F la intersección de AD con BC. Probar que el segmento EF tiene longitud constante y dirección constante al variar la cuerda CD sobre la semicircunferencia. 1 Olimpiada española 2007 Problema 1. Sean 0 1 2 3 4 , , , , a a a a a cinco números positivos en progresión aritmética de razón d. Probar que ( ) 3 3 3 3 3 2 0 1 3 4 1 4 4 . 10 a a a a a ≤ + + + Solución 1 (del autor de la propuesta). La desigualdad dada puede escribirse como 3 3 3 3 3 2 0 1 3 4 10 4 4 . a a a a a ≤ + + + y sumando 3 2 6a a ambos miembros se convierte en ( ) 3 3 3 3 3 3 2 0 1 2 3 4 1 4 6 4 . 16 a a a a a a ≤ + + + + Por otro lado como 0 1 2 3 4 , , , , a a a a a están en progresión aritmética, entonces ( ) ( ) 0 1 2 3 4 0 4 1 3 2 4 2 2 2 2 2 4 4 4 4 4 4 4 4 0 1 2 3 4 0 1 2 4 4 4 4 4 4 4 4 2 2 2 . 0 1 2 0 1 2 3 4 a a a a a a a a a a a a a a a | | | | | | | | | | | | | | | | + + + + = + + + + = | | | | | | | | \ ¹ \ ¹ \ ¹ \ ¹ \ ¹ \ ¹ \ ¹ \ ¹ | | | | | | | | | | | | | | | | + + = + + + + = | | | | | | | | \ ¹ \ ¹ \ ¹ \ ¹ \ ¹ \ ¹ \ ¹ \ ¹ Aplicando la desigualdad de Jensen a la función ( ) 3 f t t = , convexa en ( ) 0, +∞ , con 4 1 , 0 4 2 k k p k k | | = ≤ ≤ | \ ¹ resulta ( ) 4 4 0 0 k k k k k k f p a p f a = = | | ≤ | \ ¹ ∑ ∑ o equivalentemente, ( ) 3 3 3 3 3 3 3 3 3 3 3 2 0 1 2 3 4 0 1 2 3 4 4 4 4 4 4 1 1 4 6 4 0 1 2 3 4 2 16 k a a a a a a a a a a a | | | | | | | | | | ≤ + + + + = + + + + | | | | | \ ¹ \ ¹ \ ¹ \ ¹ \ ¹ . Obsérvese que la igualdad tiene lugar cuando los cinco números son iguales y hemos terminado. Solución 2. Llamando a al término central y d a la diferencia, la progresión es 2 , , , , 2 a d a d a a d a d − − + + y tenemos: ( ) ( ) ( ) ( ) 3 3 3 2 2 3 0 3 3 3 2 2 3 4 3 3 3 2 2 3 1 3 3 3 2 2 3 3 2 6 12 8 2 6 12 8 4 4 4 12 12 4 4 4 4 12 12 4 a a d a a d ad d a a d a a d ad d a a d a a d ad d a a d a a d ad d = − = − + − = + = + + + = − = − + − = + = + + + sumando: 2 3 3 3 3 3 2 0 1 3 4 4 4 10 48 a a a a a ad + + + = + dividiendo por 10 queda ( ) 3 3 3 3 3 2 0 1 3 4 1 4 4 4, 8 0 10 a a a a a ad + + + − = ≥ con independencia del valor de d. Solución 3. Como se trata de cinco términos en progresión aritmética, se tiene 0 4 2 1 3 2 a a a a a + = = + o también ( )( ) 0 4 2 2 2 2 . a a a d a d = − + Entonces ( ) ( ) 3 3 3 3 0 4 0 4 0 4 0 4 2 0 4 2 3 8 6 , a a a a a a a a a a a a + = + − + = − y ( ) ( ) ( ) ( ) 3 3 3 3 1 3 1 3 1 3 1 3 2 1 0 2 4 3 4 8 6 a a a a a a a a a a a a a + = + − + = − . Entonces, lo que hay que probar es ( ) 3 3 3 2 2 0 1 2 2 1 3 2 1 8 6 32 24 10 a a a a a a a a a ≤ − + − cuyo segundo miembro es ( ) 3 2 2 0 4 1 3 6 4 4 ; 10 a a a a a a − + trasponiendo términos, la desigualdad a probar se escribe como ( ) 2 2 2 2 3 3 2 3 2 2 2 2 2 2 2 3 24 24 4 4 4 3 3 3 0 5 5 5 a a d a d a a d a d − + − ≤ ⇔ − ≤ ⇔ − ≤ la última desigualdad es cierta y hemos terminado. Problema 2. Determinar todos los posibles valores enteros no negativos que puede tomar la expresión , 1 2 2 − + + mn n mn m siendo my n enteros no negativos tales que . 1 ≠ mn Solución. Sea N k k mn n mn m ∈ = − + + , 1 2 2 (naturales con el ). 0 En el caso , n m = el número 1 3 3 2 − + = m k es un entero positivo si 0 = m ó ; 2 = m de donde 0 = k ó 4 = k respectivamente. 3 El caso 0 = n lleva a que 2 m k − = y por tanto . 0 = = k m Consideremos ahora las soluciones ) , ( n m tales que . 0 > > n m Como la relación dada es equivalente a , 0 ) 1 ( 2 2 = + + − − k n mn k m observamos que si ) , ( n m es una solución y , 0 ) 1 ( > − − > m n k n entonces ) ) 1 ( , ( m n k n − − es también una solución. La desigualdad 0 ) 1 ( > − − m n k es cierta en todos los casos porque se convierte sucesivamente en una desigualdad obvia: , n n m k + > , 1 2 2 n n m mn n mn m + > − + + . 3 n m n − − > Del mismo modo la desigualdad m n k n − − > ) 1 ( es sucesivamente equivalente a: , 2 n n m k + < , 2 1 2 2 n n m mn n mn m + < − + + 1 3 2 − + > n n n m para . 1 > n Si , 1 3 2 − < n n esto es si , 4 ≥ n la desigualdad anterior es cierta y por tanto para cada solución ) , ( n m con 4 ≥ > n m encontramos una solución ) , ( p n con . 0 > > p n De este modo, cada solución es tal que . 3 ≤ n Para , 1 = n obtenemos , 1 3 2 − − + = m m k 4 = m ó . 7 , 2 = = k m Para , 2 = n obtenemos , 1 2 21 5 2 4 − − + = m m k 4 = m ó 4 , 11 = = k m ó . 7 = k Para , 3 = n obtenemos , 1 3 91 10 3 9 − + + = m m k que no conduce a ninguna solución. Entonces los posibles valores k enteros no negativos de la expresión del enunciado son 4 , 0 y . 7 Problema 3. Sea O el circuncentro de un triángulo ABC. La bisectriz que parte de A corta al lado opuesto en P. Probar que se cumple: 2 2 2 AP OA OP bc + − = Solución: Prolongamos AP hasta que corte en M al circuncírculo. Los triángulos ABM y APC son semejantes al tener dos ángulos iguales.(∠ACB = ∠AMB por inscritos en el mismo arco y ∠BAN = ∠CAN por bisectriz). Entonces: · c AP bc AM AP AM b = ⇔ = como AM = AP + PM, queda: ( ) 2 · bc AP AP PM AP AP PM = + = + c b A B C O P M 4 AP·PM es la potencia de P respecto de la circunferencia circunscrita y su valor es OA 2 – OP 2 sólo queda sustituir y resulta: 2 2 2 bc AP OA OP = + − Problema 4. ¿Cuáles son los números enteros positivos que se pueden obtener de exactamente 2007 maneras distintas, como la suma de al menos dos números enteros positivos consecutivos? ¿Cuál es el menor de todos ellos? Ejemplo: el número 9 se escribe exactamente de dos maneras distintas: 9 = 4 + 5 9 = 2 + 3 + 4 Solución ( ) ( ) ( )( ) ( )( ) 1 2 1 2 1 2 2 n a n N a a a n N n a n + + = + + + + + = ⇔ = + + K Si n es par ( ) 1 n + es impar y ( ) 2a n + es par. Si n es impar ( ) 1 n + es par y ( ) 2a n + es impar. Siempre que 2N en su descomposición en factores primos tenga un factor impar distinto de uno, existe una descomposición de N en suma de números consecutivos. En efecto, sea ( ) 2 1 N q Q = + con q, Q naturales y 1 ≥ q , 1 > Q , entonces: a) ) 2 )( 1 ( ). 1 2 ( 2 n a n Q q + + = + , y hacemos la siguiente descomposición: 1 1 2 + = + n q , es decir q n 2 = y 1 > n q a Q 2 2 2 + = , de donde q Q a − = Si 0 a > ya hemos terminado, en otro caso 0 Q q − ≤ y entonces, b) 1 2 + = n Q , de donde 1 2 − = Q n , es decir 1 n ≥ y 1 2 1 2 2 + = − + q Q a de donde Q q a − + =1 , es decir 1 a ≥ y ya hemos terminado. Entonces necesitamos el menor número que en su descomposición tenga 2007 factores impares sin contar el 1. Si sólo tiene un factor primo es: 2 . 3 2 2007 = N Si tiene más de uno .. γ β α r q p , el número de divisores que tiene es )...... 1 )( 1 )( 1 ( + + + γ β α , y este producto debe ser 2008 para que excepto l uno tenga 2007 divisores impares distintos. )...... 1 )( 1 )( 1 ( 251 . 2 2008 3 + + + = = γ β α , entonces: 7 250 5 . 3 = N , o bien 7 . 5 . 3 3 250 = N , o bien 11 . 7 . 5 . 3 250 = N y este último es el menor. Problema 5. Sea 1 a ≠ un número real positivo y n un entero positivo. Demostrar que 2 1 2 . 2 n n a a n a a − − + − < + − Solución. 5 La desigualdad dada 2 1 2 2 n n a a n a a − − + − < + − es equivalente a 2 2 2 2 2 1 1 2 2 , n n a a n a a − − | | − | \ ¹ < | | − | \ ¹ que a su vez equivale a que 1 , n n n α α α α − − − < − siendo . a α = Entonces, usando la desigualdad aritmético-geométrica, se tiene la desigualdad pedida: 2 1 1 2 4 2 2 1 2 4 ... (2 2) 1 1 1 2 1 (1 ... ) . 1 n n n n n n n n n n n n n n α α α α α α α α α α α α α α α − − − − − + + + − − − − − − = = + + + + > = = − − Problema 6. Dada una semicircunferencia de diámetro AB = 2R, se considera una cuerda CD de longitud fija c. Sea E la intersección de AC con BD y F la intersección de AD con BC. Probar que el segmento EF tiene longitud constante y dirección constante al variar la cuerda CD sobre la semicircunferencia. Solución. Como los triángulos EFC y EDF son rectángulos, el cuadrilátero EDFC es inscriptible y EF es el diámetro. Llamemos r al radio del circuncírculo de ECD, por el teorema de los senos en ECD: ( ) 2 1 sen c EF r E = = Pongamos α = ∠BOD y β = ∠COD. Entonces 180 90 2 2 E β β − = = − expresión que prueba que el ángulo E es constante al serlo β y además el punto E se mueve en el arco capaz de 2 90 β − sobre AB. Sustituyendo en (1) queda: ( ) 2 2 cos 2 c EF r β = = , por otra parte ( ) 3 R 2 c 2 sen = β ; eliminando β entre (2) y (3) y despejando EF resulta: 2 2 2 4 cR EF R c = − expresión que muestra que EF es constante al serlo c y R. Además F es el ortocentro del triángulo ABE como intersección de las alturas AD y BC, por ello EF que está sobre la tercera altura es siempre perpendicular a AB. También admite solución analítica aunque mucho más larga. β α F E D O A B C XLIV Olimpiada Matemática Española Fase nacional 2008 (Valencia) PRIMERA SESIÓN (28 de marzo) • 1.- Halla dos enteros positivos a y b conociendo su suma y su mínimo común múltiplo. Aplícalo en el caso de que la suma sea 3972 y el mínimo común múltiplo . 985928 • 2.- Prueba que para cualesquiera números reales b a, tales que , 1 , 0 < < b a se cumple la desigualdad siguiente: . 2 ) 1 ( ) 1 ( ) 1 )( 1 ( 2 2 2 2 < − − + − − + + b a b a b a ab • 3.- Sea 3 ≥ p un número primo. Se divide cada lado de un triángulo en p partes iguales y se une cada uno de los puntos de división con el vértice opuesto. Calcula el número máximo de regiones, disjuntas dos a dos, en que queda dividido el triángulo. No está permitido el uso de calculadoras. Cada problema se valora hasta 7 puntos. El tiempo de cada sesión es de tres horas y media. XLIV Olimpiada Matemática Española Fase nacional 2008 (Valencia) SEGUNDA SESIÓN (29 de marzo) • 4.- Sean p y q dos números primos positivos diferentes. Prueba que existen enteros positivos a y , b tales que la media aritmética de todos los divisores positivos del número b a q p n = es un número entero. • 5.- Dada una circunferencia y en ella dos puntos fijos A, B, otro variable P y una recta ; r se trazan las rectas PA y PB que cortan a r en C y D respectivamente. Determina dos puntos fijos de , r M y N, tales que el producto · CM DN sea constante al variar P. • 6.- A cada punto del plano se le asigna un solo color entre siete colores distintos. ¿Existirá un trapecio inscriptible en una circunferencia cuyos vértices tengan todos el mismo color? No está permitido el uso de calculadoras. Cada problema se valora hasta 7 puntos. El tiempo de cada sesión es de tres horas y media. 1 XLIV Olimpiada Matemática Española Fase nacional 2008 (Valencia) PRIMERA SESIÓN (28 de marzo) 1.- Halla dos enteros positivos a y b conociendo su suma y su mínimo común múltiplo. Aplícalo en el caso de que la suma sea 3972 y el mínimo común múltiplo . 985928 SOLUCIÓN: Sea p un número primo que divide a la suma b a + y a su mínimo común múltiplo [ ]. , b a Como [ ] b a p , al menos divide a uno de los dos enteros a ó . b Si , a p al dividir p a la suma , b a + también . b p (Obviamente el mismo razonamiento vale si hubiéramos supuesto que ). b p Por tanto podemos dividir los dos números a y b por p y también su mínimo común múltiplo [ ], , b a para obtener dos enteros 1 a y 1 b tales que p b b p a a = = 1 1 , y [ ] [ ] . , , 1 1 p b a b a = Sea el máximo común divisor de a y , b ( ). , b a d = Repitiendo el proceso anterior llegaremos a obtener dos enteros A y B tales que dB b dA a = = , y ( ) . 1 , = B A Entonces [ ] . , AB B A = Ahora es fácil determinar A y B a partir del sistema de ecuaciones [ ] . , ⎪ ⎪ ⎩ ⎪ ⎪ ⎨ ⎧ = + = + d b a AB d b a B A Es decir A y B son las raíces de la ecuación de segundo grado [ ] . 0 , ) ( 2 = + + − b a t b a t d Observamos que el discriminante de esta ecuación es no negativo. En efecto: [ ] . 0 ) ( 4 ) ( , 4 ) ( 2 2 2 ≥ − = − + = − + = ∆ b a ab b a b a d b a Si a y b son distintos, la ecuación anterior tiene por soluciones los dos enteros positivos d a A = y . d b B = En particular cuando 3972 = + b a y [ ] , 985928 , = b a tenemos que . 4 ) 985928 , 3972 ( = = d Por tanto A a 4 = y B b 4 = siendo A y B las raíces de la ecuación . 0 985928 3972 4 2 = + − t t Es decir 491 = A y 502 = B y los números buscados son 1964 = a (año de la primera OME) y 2008 = b (año de la actual edición de la OME). 2 XLIV Olimpiada Matemática Española Fase nacional 2008 (Valencia) PRIMERA SESIÓN (28 de marzo) 2.- Prueba que para cualesquiera números reales b a, tales que , 1 , 0 < < b a se cumple la desigualdad siguiente: . 2 ) 1 ( ) 1 ( ) 1 )( 1 ( 2 2 2 2 < − − + − − + + b a b a b a ab SOLUCIÓN: Se verifica que 3 x x < para todo ). 1 , 0 ( ∈ x Teniendo en cuenta que , 1 2 0 < + < b a utilizando la desigualdad anterior y aplicando la desigualdad entre las medias aritmética y geométrica, se tiene: 2 3 2 2 2 3 b a b a b a b a ab b a ab + = ⎟ ⎠ ⎞ ⎜ ⎝ ⎛ + + + ≤ ⎟ ⎠ ⎞ ⎜ ⎝ ⎛ + < ⎟ ⎠ ⎞ ⎜ ⎝ ⎛ + y . 2 1 3 2 1 1 1 2 1 ) 1 ( ) 1 ( 2 1 ) 1 ( ) 1 ( 3 b a b a b a b a b a b a b a + − = + − + − + − ≤ ≤ ⎟ ⎠ ⎞ ⎜ ⎝ ⎛ + − − − < ⎟ ⎠ ⎞ ⎜ ⎝ ⎛ + − − − Sumando las expresiones anteriores resulta , 1 2 1 ) 1 ( ) 1 ( 2 < ⎟ ⎠ ⎞ ⎜ ⎝ ⎛ + − − − + ⎟ ⎠ ⎞ ⎜ ⎝ ⎛ + b a b a b a ab o equivalentemente ( ) , 1 ) 1 ( ) 1 ( ) 1 )( 1 ( 2 1 2 2 2 2 < − − + − − + + b a b a b a ab de donde se obtiene inmediatamente la desigualdad del enunciado. 3 XLIV Olimpiada Matemática Española Fase nacional 2008 (Valencia) PRIMERA SESIÓN (28 de marzo) 3.- Sea 3 ≥ p un número primo. Se divide cada lado de un triángulo en p partes iguales y se une cada uno de los puntos de división con el vértice opuesto. Calcula el número máximo de regiones, disjuntas dos a dos, en que queda dividido el triángulo. SOLUCIÓN: En primer lugar veremos que tres de estos segmentos (cevianas) no pueden ser concurrentes. Sea el triángulo ABC y Z Y X , , puntos de las divisiones interiores de los lados AB AC BC , , respectivamente. Si BY AX, y CZ fueran concurrentes aplicando el teorema de Ceva tendríamos . 1 = ⋅ ⋅ YA CY XC BX ZB AZ Por otro lado, por la forma en que hemos construido los puntos de división, existen enteros positivos { }, 1 ,..., 2 , 1 , , − ∈ p m l k tales que . , , m p m YA CY l p l XC BX k p k ZB AZ − = − = − = Sustituyendo en la expresión anterior ), ( ) ( ) ( m p l p k p m l k − − − = o equivalentemente, ). ( ) ( ) ( 2 l p k p p l k p m p m l k − − = − − + De aquí resulta que p divide al producto , klm que es imposible y nuestra afirmación inicial queda probada. Dibujando las cevianas desde el vértice A el triángulo ABC queda dividido en p triángulos. Las cevianas trazadas desde B dividen cada uno de los p triángulos anteriores en p partes disjuntas, teniendo en total 2 p regiones. Cada ceviana trazada desde el vértice C aumenta el número de regiones en un número exactamente igual a su número de intersecciones con las rectas que encuentra (incluido el lado AB ). Es decir, . 1 2 1 ) 1 ( 2 − = + − p p Por tanto, el número máximo de regiones disjuntas dos a dos, en que queda dividido el triángulo ABC es . 1 3 3 ) 1 2 )( 1 ( 2 2 + − = − − + p p p p p 1 XLIV Olimpiada Matemática Española Fase nacional 2008 (Valencia) SEGUNDA SESIÓN (29 de marzo) 4.- Sean p y q dos números primos positivos diferentes. Prueba que existen enteros positivos a y , b tales que la media aritmética de todos los divisores positivos del número b a q p n = es un número entero. SOLUCIÓN: La suma de todos los divisores de n viene dada por la fórmula ), ... 1 ( ) ... 1 ( 2 2 b a q q q p p p + + + + + + + + como se puede comprobar desarrollando los paréntesis. El número n tiene ) 1 )( 1 ( + + b a divisores positivos y la media aritmética de todos ellos es . ) 1 ( ) 1 ( ) ... 1 ( ) ... 1 ( 2 2 + + + + + + + + + + = b a q q q p p p m b a Si p y q son ambos impares, tomando p a = y , q b = es fácil ver que m es un entero. Efectivamente: cada factor p p p p p + + + + + ... 1 3 2 y q q q q q + + + + + ... 1 3 2 tiene un número par de sumandos y por ejemplo, el primero se puede escribir como sigue ). ... 1 ( ) 1 ( ) 1 ( ... ) 1 ( ) 1 ( ... 1 1 2 1 2 3 2 − − + + + + = + + + + + + = + + + + + p p p p p p p p p p p p p p p Análogamente el segundo factor ). ... 1 ( ) 1 ( ... 1 1 2 3 2 − + + + + = + + + + + q q q q q q q q q Entonces ), ... 1 ( ) ... 1 ( 1 4 2 1 4 2 − − + + + + + + + + = q p q q q p p p m que es un entero positivo. Si 2 = p y q es impar, se eligen q b = y . ... 1 1 1 4 2 − + + + + = + q q q q a Entonces a q q q q q q q q q q q q q m q 2 ... 2 2 1 ) 1 ( ) ... 1 ( ) ... 1 ( ) 2 ... 2 2 1 ( 2 1 4 2 3 2 ... 2 1 4 2 + + + + = + + + + + + + + + + + + + + = − + + + − , que es entero. Para 2 = q y p impar, análogamente al caso anterior se eligen p a = y 1 4 2 ... − + + + = p p p p b y m es entero. Alternativamente y de una manera casi directa, se obtiene una solución completa observando que si p y q son primos impares se toman 1 = = b a y si 2 = p y q primo impar, entonces se consideran 2 1 − = q a y . 1 = b 2 XLIV Olimpiada Matemática Española Fase nacional 2008 (Valencia) SEGUNDA SESIÓN (29 de marzo) 5.- Dada una circunferencia y en ella dos puntos fijos A, B, otro variable P y una recta ; r se trazan las rectas PA y PB que cortan a r en C y D respectivamente. Determina dos puntos fijos de , r M y N, tales que el producto · CM DN sea constante al variar P. SOLUCIÓN: Trazamos las paralelas a r por A y B que cortan a la circunferencia en ´ A y ´ B respectivamente de modo que ´ ´BB AA es un trapecio isósceles. Las intersecciones de ´ AB y ´ BA con r determinan los puntos M y N buscados. En efecto, los triángulos AMC y DNB (sombreados en la figura) son semejantes ya que tienen dos ángulos iguales: , ´ NDB BP B MAC ∠ = ∠ = ∠ donde la primera igualdad es cierta por ser ángulos inscritos en el mismo arco y la segunda por ser ´ BB paralela a . r , ´ DNB B AB AMC ∠ = ∠ = ∠ con argumentos análogos a los anteriores. Estableciendo la proporcionalidad de los lados resulta , BN AM ND MC BN ND MC AM ⋅ = ⋅ ⇔ = cantidad que no depende de P. r N M B' A' C D A B P 3 Se observa que si la recta r pasa por el punto , A , C A M = = no se forma el triángulo . AMC En este caso 0 = CM y el producto , 0 = ⋅ DN CM es constante. Análogamente este producto es cero si la recta r pasa por B o por los puntos A y B en cuyo caso . 0 = = DN CM XLIV Olimpiada Matemática Española Fase nacional 2008 (Valencia) SEGUNDA SESIÓN (29 de marzo) 6.- A cada punto del plano se le asigna un solo color entre siete colores distintos. ¿Existirá un trapecio inscriptible en una circunferencia cuyos vértices tengan todos el mismo color? SOLUCIÓN: La idea inicial es considerar una circunferencia C de radio r y sobre ella bloques de 8 puntos 8 2 1 ,..., , A A A igualmente espaciados; es decir que los arcos 7 ,..., 1 1 = + i A A i i tengan igual longitud 0 > λ (que se elegirá convenientemente) para cada uno de los bloques. Se elige un sentido dado (por ejemplo, el antihorario). Se disponen entonces, en este sentido antihorario, 50 1 7 7 = + × bloques de 8 1 7 = + puntos cada uno en la semicircunferencia superior de , C tales que dos bloques distintos no se intersequen o solapen, para lo cual se toma λ suficientemente pequeño, por ejemplo . 400 0 r π λ < < Se observa que al menos hay dos puntos del mismo color en cada bloque. Se eligen dos de esos puntos y su color se le asocia al bloque. Y la distancia entre estos dos puntos que es uno de los siete números { }, 7 , 6 , 5 , 4 , 3 , 2 , 1 , ∈ = n n d n λ se le asigna también al bloque. De este modo a cada uno de los 50 bloques se le hace corresponder el par (color, distancia), indicado anteriormente. Como el número total de posibles pares es , 49 por el principio el palomar, existirán dos bloques R y Q a los que se les asocia el mismo par (color, distancia). Por tanto los cuatro puntos determinados por estos dos bloques tienen el mismo color. Y como los dos puntos del bloque R distan igual que los dos puntos del bloque , Q estando los cuatro puntos sobre la circunferencia , C necesariamente, estos cuatro puntos son los vértices de un trapecio inscriptible. NOTA: Este mismo razonamiento se podría hacer considerando un arco de circunferencia de radio r de longitud . 2 0 , r l l π ≤ < En este caso se tomaría . 400 0 , l < < λ λ Y también se podría generalizar a un número de colores ) 2 ( ≥ c c cualesquiera, considerando 1 + c puntos en vez de 1 7 + y 1 2 + c bloques disjuntos de 1 + c puntos cada uno, en vez de 1 7 2 + bloques de 1 7 + puntos cada uno. Y ahora λ debe cumplir que . ) 1 )( 1 ( 0 2 + + < < c c l λ Fase Nacional de la XLV Olimpiada Matemática Española Sant Feliu de Guixols (Girona), 27 de marzo de 2009 PRIMERA SESIÓN 1.- Halla todas las sucesiones finitas de números naturales consecutivos con tales que n , ,..., , 2 1 n a a a , 3 ≥ n . 2009 ... 2 1 = + + + n a a a 2.- Sean un triángulo acutángulo, ABC I el centro del círculo inscrito en el triángulo , ABC r su radio y R el radio del círculo circunscrito al triángulo Se traza la altura con perteneciente al lado Demuestra que . ABC , a h AD = D . BC ). 2 ( ) 2 ( 2 r h h R DI a a − − = 3.- Se pintan de rojo algunas de las aristas de un poliedro regular. Se dice que una coloración de este tipo es buena, si para cada vértice del poliedro, existe una arista que concurre en dicho vértice y no está pintada de rojo. Por otra parte, se dice que una coloración donde se pintan de rojo algunas de las aristas de un poliedro regular es completamente buena, si, además de ser buena, ninguna cara del poliedro tiene todas sus aristas pintadas de rojo. ¿Para qué poliedros regulares es igual el número máximo de aristas que se pueden pintar en una coloración buena y en una completamente buena? Justifica la respuesta. No está permitido el uso de calculadoras. Cada problema vale siete puntos. El tiempo de cada sesión es de tres horas y media. Fase Nacional de la XLV Olimpiada Matemática Española Sant Feliu de Guixols (Girona), 28 de marzo de 2009 SEGUNDA SESIÓN 4.- Determina justificadamente todos los pares de números enteros que verifican la ecuación ) , ( y x . 2009 4 2 = − y x 5.- Sean números reales positivos tales que c b a , , . 1 = abc Prueba la desigualdad siguiente + ⎟ ⎠ ⎞ ⎜ ⎝ ⎛ + 2 1 ab a + ⎟ ⎠ ⎞ ⎜ ⎝ ⎛ + 2 1 bc b 4 3 1 2 ≥ ⎟ ⎠ ⎞ ⎜ ⎝ ⎛ + ca c 6.- En el interior de una circunferencia de centro y radio O , r se toman dos puntos A y simétricos respecto de Se considera , B . O P un punto variable sobre esta circunferencia y se traza la cuerda perpendicular a Sea el punto simétrico de ´, PP . AP C B respecto de Halla el lugar geométrico del punto intersección de con al variar sobre la circunferencia. ´. PP , Q ´ PP , AC P No está permitido el uso de calculadoras. Cada problema vale siete puntos. El tiempo de cada sesión es de tres horas y media. Fase Nacional de la XLV Olimpiada Matemática Española Sant Feliu de Guixols (Girona), 27 de marzo de 2009 PRIMERA SESIÓN SOLUCIONES PROBLEMA 1.- Halla todas las sucesiones finitas de números naturales consecutivos con tales que n , ,..., , 2 1 n a a a , 3 ≥ n . 2009 ... 2 1 = + + + n a a a Primera solución: Supongamos que es la suma de números naturales consecutivos empezando por Entonces N n . 1 + k = + + + + + + = ) ( ... ) 2 ( ) 1 ( n k k k N [ ] [ ] = + + + − + + + + + + + + k n k k k ... 2 1 ) ( ... ) 1 ( ... 2 1 . 2 ) 1 2 ( 2 ) 1 ( 2 ) 1 )( ( + + = + − + + + n k n k k n k n k Teniendo en cuenta que 41 49 287 7 2009 1 2009 × = × = × = se tienen los siguientes casos: (1) Si y 7 = n ( ) 287 2 1 2 = + + n k resulta 283 = k con lo que . 290 289 288 287 286 285 284 2009 + + + + + + = (2) Si 7 2 = n y , 287 1 2 = + + n k resulta 136 = k con lo que . 150 ... 138 137 2009 + + + = (3) Si y 41 = n ( ) 49 2 1 2 = + + n k resulta 28 = k con lo que . 69 ... 31 30 29 2009 + + + + = (4) Si y 49 = n ( ) 41 2 1 2 = + + n k resulta 16 = k con lo que . 65 ... 19 18 17 2009 + + + + = (5) Los otros casos dan valores de k que no verifican el enunciado. Segunda solución: Claramente, es , 1 1 − + = n a a n de donde . 2 ) 1 ( 2009 1 − + = n n a n Entonces , 2 ) 1 ( 2 ) 1 ( 2009 2 − > − ≥ n n n luego , 4018 1 < − n y al ser resulta que , 63 3969 4018 4096 64 2 2 = > > = , 64 4018 < con lo que . 65 < n Supongamos en primer lugar que n es impar. Entonces, obviamente divide a y puede tomar los valores pues cualquier otro divisor impar de es mayor o igual que n 41 7 2009 2 × = n 41 , 7 ó , 49 2009 . 287 41 7 = × Se obtiene entonces , 2 1 2009 1 − − = n n a con valores respectivos y es decir, se obtienen las tres sucesiones 29 , 284 , 17 ; 290 ,..., 285 , 284 ; 69 ,..., 30 , 29 . 65 ,..., 18 , 17 No hay otras sucesiones con un número impar de términos. Supongamos finalmente que n es par. Entonces 2 n divide a con lo que ya que cualquier otro valor de ha de ser mayor o igual que lo que no es posible. Entonces , 2009 , 14 = n n , 65 82 41 2 > = × , 137 1 = a y la única sucesión con número par de términos es . 150 ,..., 138 , 137 Fase Nacional de la XLV Olimpiada Matemática Española Sant Feliu de Guixols (Girona), 27 de marzo de 2009 PRIMERA SESIÓN SOLUCIONES PROBLEMA 2 .- Sean un triángulo acutángulo, ABC I el centro del círculo inscrito en el triángulo , ABC r su radio y R el radio del círculo circunscrito al triángulo Se traza la altura con perteneciente al lado Demuestra que . ABC , a h AD = D . BC ). 2 ( ) 2 ( 2 r h h R DI a a − − = Primera solución: Sean E y M las proyecciones ortogonales de I sobre y respectivamente. BC , AD Se tiene: ; 2 A sen r AI = p S r = ⇒ 2 A sen p S AI ⋅ = (1) donde, evidentemente, es el área del triángulo y S ABC p es su semiperímetro. Por otra parte, , 2 cos 2 2 A A sen bc senA bc S ⋅ ⋅ = ⋅ = así que (1) se puede escribir como , 2 cos p A bc AI ⋅ = y ya que , ) ( 2 cos 2 bc a p p A − = obtenemos p a p bc AI ) ( 2 − = (2). Teniendo en cuenta que , 2 a h R bc ⋅ = , r S p = , 2 a h S a = la expresión (2) se escribe como ) 2 ( 2 2 1 2 2 1 2 2 r h R h r h R r S h S h R AI a a a a a − = ⎟ ⎟ ⎠ ⎞ ⎜ ⎜ ⎝ ⎛ − ⋅ = ⎟ ⎟ ⎟ ⎟ ⎠ ⎞ ⎜ ⎜ ⎜ ⎜ ⎝ ⎛ − ⋅ = (3).Como el cuadrilátero IEDM es un rectángulo, . r IE MD = = Aplicando el teorema de Pitágoras generalizado a ADI tenemos ⇔ ⋅ − + = AM h AI h DI a a 2 2 2 2 ), ( 2 2 2 2 MD h h AI h DI a a a − − + = y teniendo en cuenta los resultados anteriormente obtenidos resulta, finalmente, c.q.d. ), 2 ( ) 2 ( ) ( 2 ) 2 ( 2 2 2 r h h R r h h r h R h DI a a a a a a − − = − − − + = Segunda solución: Sean las longitudes de los lados y c b a , , CA BC, AB respectivamente, y sea T el punto donde la circunferencia inscrita es tangente al lado Por el teorema de Pitágoras, al ser . BC , // DT AD IT ⊥ se tiene que 2 2 2 2 2 2 AD AT r DT r DI − + = + = y la igualdad a demostrar es equivalente a . ) ( 2 4 2 2 a h r R r Rr AT + = + + Ahora bien, llamando al área de es conocido que S , ABC . 2 ) ( ) ( ) ( ) ( 4 1 2 ) ( 4 a h a c b a b a c a c b c b a c b a r R abc S = − + − + − + + + = + + = = De aquí se deduce que , 2 4 c b a abc Rr + + = , ) ( ) ( ) ( 2 c b a c b a b a c a c b r + + − + − + − + = bc h R a = 2 y . 2 ) ( ) ( ) ( 2 a c b a b a c a c b h R a − + − + − + = Obsérvese que . 4 2 2 2 4 2 2 2 2 c b a ca bc ab r Rr − − − + + = + Ahora bien, se sabe que , 2 b a c BT − + = . 2 c b a CT − + = Por el teorema de Stewart, tenemos que = ⋅ − ⋅ + ⋅ = CT BT BC AB CT AC BT AT 2 2 2 a c b c b bc a c b 2 ) ( ) ( 4 2 3 3 2 2 2 2 + − − − − + con lo que , 2 ) ( ) ( 2 4 2 2 2 2 2 2 a c b c b ca ab a c b r Rr AT + − = + + − + = + + e identificando términos se comprueba que esto coincide con el valor de a h r R ) ( 2 + y de este modo la igualdad requerida queda demostrada. Tercera solución: Nótese en primer lugar que , 2 2 2 2 2 2 2 ) ( 2 2 2 2 2 2 R IA A sen R r A sen C sen B sen r a r a c b a ar S r h a = = = − + = − = − donde es el área de y se ha utilizado que S ABC . 2 2 2 4 C sen B sen A sen R r = Sea ahora el punto P en el que la paralela a por BC I corta a la altura Claramente, . AD ). 2 ( ) 2 ( ) 2 ( ) ( 2 2 2 2 2 r h h R r h h IA r h IA r DI a a a a a − − = − − = − − + = Fase Nacional de la XLV Olimpiada Matemática Española Sant Feliu de Guixols (Girona), 27 de marzo de 2009 PRIMERA SESIÓN SOLUCIONES PROBLEMA 3.- Se pintan de rojo algunas de las aristas de un poliedro regular. Se dice que una coloración de este tipo es buena, si para cada vértice del poliedro, existe una arista que concurre en dicho vértice y no está pintada de rojo. Por otra parte, se dice que una coloración donde se pintan de rojo algunas de las aristas de un poliedro regular es completamente buena, si además de ser buena, ninguna cara del poliedro tiene todas sus aristas pintadas de rojo. ¿Para qué poliedros regulares es igual el número máximo de aristas que se pueden pintar en una coloración buena y en una completamente buena? Justifica la respuesta. Solución: Claramente, las coloraciones completamente buenas son un subconjunto de las coloraciones buenas, con lo que si el máximo número de aristas que se pueden pintar de rojo para obtener una coloración buena se puede alcanzar con una coloración completamente buena, la pregunta del enunciado tiene respuesta afirmativa. NOTA: En cada caso véase el recuadro de las figuras al final de la solución. En el caso de un tetraedro, existen 6 aristas, tales que en cada vértice confluyen de ellas. El número máximo de aristas pintadas de rojo en una coloración buena sería por lo tanto 3 , 4 6 3 2 = × pues en caso contrario existiría algún vértice donde más de 3 2 de las aristas estuvieran pintadas de rojo, es decir, todas las aristas estarían pintadas de rojo. La figura muestra una coloración completamente buena de un tetraedro con aristas rojas (el tetraedro ha sido deformado para poder ser dibujado en el plano). 4 De igual forma, en el cubo existen 12 aristas, tales que en cada vértice confluyen de ellas. El número máximo de aristas pintadas de rojo en una coloración buena sería, por lo tanto, 3 . 8 12 3 2 = × La figura muestra una coloración completamente buena con 8 aristas rojas. Finalmente, en el dodecaedro existen 30 aristas, tales que en cada vértice confluyen de ellas. El número máximo de aristas pintadas de rojo en una 3 coloración buena sería, por lo tanto, . 20 30 3 2 = × La figura muestra una coloración completamente buena de un dodecaedro con 12 aristas rojas. De lo anterior se deduce que para el tetraedro, el cubo y el dodecaedro, el número máximo de aristas rojas en una coloración buena se alcanza con una coloración completamente buena. En el octaedro existen 12 aristas, tales que en cada vértice confluyen 4 de ellas. El número máximo de aristas pintadas de rojo en una coloración buena es por lo tanto . 9 12 4 3 = × La figura muestra una coloración buena de un octaedro con aristas rojas. Ahora bien, como cada arista pertenece a dos caras, sumando para las 8 caras el número de aristas pintadas de rojo en dicha cara, obtenemos con lo que hay, al menos, dos caras con 3 aristas pintadas de rojo y una coloración buena con el número máximo de 9 aristas pintadas de rojo nunca puede ser completamente buena. 9 , 2 8 2× 18 + = Finalmente, en el icosaedro existen 30 aristas, tales que en cada vértice confluyen de ellas. El número máximo de aristas pintadas de rojo en una coloración buena es por lo tanto 5 . 24 30 5 4 = × La figura muestra una coloración buena de un icosaedro con 24 aristas rojas. Ahora bien, eso quiere decir, que sumando el número de aristas rojas de cada cara para todas las caras, obtenemos es decir, existen, al menos, caras con todas sus aristas rojas, y una coloración buena con el número máximo de aristas pintadas de rojo nunca puede ser completamente buena. , 8 20 2 48 + × = 8 24 Por lo tanto, los poliedros regulares que tienen la propiedad descrita en el enunciado son el tetraedro, el cubo y el dodecaedro (es decir, los poliedros tales que en cada vértice confluyen exactamente 3 aristas) y los que no la tienen son el octaedro e icosaedro (en cuyos vértices confluyen más de 3 aristas). FIGURAS FIGURAS Fase Nacional de la XLV Olimpiada Matemática Española Sant Feliu de Guixols (Girona), 28 de marzo de 2009 SEGUNDA SESIÓN SOLUCIONES PROBLEMA 4. - Determina justificadamente todos los pares de números enteros que verifican la ecuación ) , ( y x . 2009 4 2 = − y x Solución: Dada una solución cualquiera, es claro que también son soluciones y con lo que se puede asumir sin pérdida de generalidad que Supongamos entonces que es así. Es claro que ) , ( y x ) , ( ), , ( y x y x − − ), , ( y x − − . 0 , ≥ y x . 41 7 ) ( ) ( 2 2 2 ⋅ = + − y x y x Si y no son primos entre sí, su máximo común divisor al cuadrado divide a luego es 7 y divide a y a con lo que existen enteros no negativos y v tales que 2 y x − 2 y x + , 41 7 2009 2 ⋅ = x y x y x 2 ) ( ) ( 2 2 = − + + , 2 ) ( ) ( 2 2 2 y y x y x = − − + u , 7u x = y Como ambos factores han de ser enteros, se tiene que y con lo que v y 7 = . 41 ) 7 )( 7 ( 2 2 = − + v u v u 41 7 2 = + v u , 1 7 2 = − v u 21 = u y . 7 10 2 = v No existen pues soluciones enteras en este caso. Si y son primos entre sí, un posible caso es que y con lo que absurdo pues Resta entonces tan sólo el caso en que y que produce con lo que la única solución con enteros no negativos es e y las únicas soluciones en enteros son 2 y x − 2 y x + 1 2 = − y x , 2009 2 = + y x , 1004 2 = y . 32 1004 961 31 2 2 < < = 41 2 = − y x , 49 2 = + y x , 45 = x , 4 2 = y 45 = x , 2 = y ). 2 , 45 ( ) , ( ± ± = y x Fase Nacional de la XLV Olimpiada Matemática Española Sant Feliu de Guixols (Girona), 28 de marzo de 2009 SEGUNDA SESIÓN SOLUCIONES PROBLEMA 5. - Sean números reales positivos tales que Prueba la desigualdad siguiente c b a , , . 1 = abc + ⎟ ⎠ ⎞ ⎜ ⎝ ⎛ + 2 1 ab a + ⎟ ⎠ ⎞ ⎜ ⎝ ⎛ + 2 1 bc b 4 3 1 2 ≥ ⎟ ⎠ ⎞ ⎜ ⎝ ⎛ + ca c Solución: Como , 1 = abc entonces . 1 1 2 2 2 ⎟ ⎠ ⎞ ⎜ ⎝ ⎛ + = ⎟ ⎠ ⎞ ⎜ ⎝ ⎛ + = ⎟ ⎠ ⎞ ⎜ ⎝ ⎛ + c ca c abc ca ab a Análogamente se obtienen 2 2 1 1 ⎟ ⎠ ⎞ ⎜ ⎝ ⎛ + = ⎟ ⎠ ⎞ ⎜ ⎝ ⎛ + a ab bc b y . 1 1 2 2 ⎟ ⎠ ⎞ ⎜ ⎝ ⎛ + = ⎟ ⎠ ⎞ ⎜ ⎝ ⎛ + b bc ca c Por tanto la desigualdad requerida se convierte en + ⎟ ⎠ ⎞ ⎜ ⎝ ⎛ + 2 1 a ab + ⎟ ⎠ ⎞ ⎜ ⎝ ⎛ + 2 1 b bc , 4 3 1 2 ≥ ⎟ ⎠ ⎞ ⎜ ⎝ ⎛ + c ca equivalente a . 2 1 1 1 1 3 1 2 2 2 ≥ ⎥ ⎥ ⎦ ⎤ ⎢ ⎢ ⎣ ⎡ ⎟ ⎠ ⎞ ⎜ ⎝ ⎛ + + ⎟ ⎠ ⎞ ⎜ ⎝ ⎛ + + ⎟ ⎠ ⎞ ⎜ ⎝ ⎛ + c ca b bc a ab Usando ahora la desigualdad entre las medias aritmética y cuadrática, se obtiene . 1 1 1 3 1 1 1 1 3 1 2 2 2 ⎥ ⎦ ⎤ ⎢ ⎣ ⎡ ⎟ ⎠ ⎞ ⎜ ⎝ ⎛ + + ⎟ ⎠ ⎞ ⎜ ⎝ ⎛ + + ⎟ ⎠ ⎞ ⎜ ⎝ ⎛ + ≥ ⎥ ⎥ ⎦ ⎤ ⎢ ⎢ ⎣ ⎡ ⎟ ⎠ ⎞ ⎜ ⎝ ⎛ + + ⎟ ⎠ ⎞ ⎜ ⎝ ⎛ + + ⎟ ⎠ ⎞ ⎜ ⎝ ⎛ + c ca b bc a ab c ca b bc a ab Así es suficiente demostrar que 2 3 1 1 1 ≥ + + + + + c ca b bc a ab o equivalentemente , 2 3 ) 1 ( ) 1 ( ) 1 ( ≥ + + + + + c b abc b a abc a c abc que a su vez equivale a que . 2 3 ) 1 ( 1 ) 1 ( 1 ) 1 ( 1 ≥ + + + + + c b b a a c Poniendo z y b y x a = = , y x z c = en la última desigualdad resulta + ⎟ ⎟ ⎠ ⎞ ⎜ ⎜ ⎝ ⎛ + −1 z x y x + ⎟ ⎠ ⎞ ⎜ ⎝ ⎛ + −1 x y z y . 2 3 1 ≥ ⎟ ⎟ ⎠ ⎞ ⎜ ⎜ ⎝ ⎛ + − y z x z Sustituyendo ahora y x 1 , 1 = = β α y , 1 z = γ se llega a la desigualdad de Nessbit . 2 3 ≥ + + + + + β α γ α γ β γ β α La igualdad se alcanza si y sólo si . 1 = = = c b a Fase Nacional de la XLV Olimpiada Matemática Española Sant Feliu de Guixols (Girona), 28 de marzo de 2009 SEGUNDA SESIÓN SOLUCIONES PROBLEMA 6. - En el interior de una circunferencia de centro O y radio , r se toman dos puntos y simétricos respecto de Se considera un punto variable A , B . O P sobre esta circunferencia y se traza la cuerda perpendicular a Sea el punto simétrico de ´, PP . AP C B respecto de Halla el lugar geométrico del punto intersección de ´. PP , Q ´ PP con al variar , AC P sobre la circunferencia. Primera solución: Establezcamos primero que es constante. AC Método 1. Se obtiene C a partir de A aplicando un giro de con centro en O seguido de la simetría de eje PP º 180 ´. 1 A O A B P C Q A' P' e 1 e 2 M Descomponiendo el giro en producto de dos simetrías de ejes perpendiculares e paralelo a P y perpendicular a resulta que el triángulo es rectángulo en 2 e , AP C AA´ ´ A y además: 2 ´ AA = , MP 2 ´C A = ; OM de donde es decir ; 4 4 4 4 2 2 2 2 r OP MP OM = = + 2 AC = 2 AC , r = con independencia de la posición de P . Método 2 O A B P P ' C P '' Prolongamos hasta que corte de nuevo a la circunferencia en P Se tiene CP PA ´´. ´ ´ P = = ´´. AP B Además B P´ ´´; PP CA ´ es paralelo a luego el segmento es la imagen del segmento ´´ P P mediante la traslación de vector A P´´ P y como ´´ ´PP P ∠ es recto y ´´ ´P P es un diámetro, resulta . 2 ´´ ´P r P AC = = Finalmente, al ser PP la mediatriz de ´ , BC ; QB QC = se deduce entonces que r AC QA QC QA QB 2 = = + = + y Q describe la elipse de focos y A B y constante La recta . 2r ´ PP es la tangente en a la elipse. Q Segunda solución: Tomamos 1 = r y unos ejes de coordenadas en los que la ecuación de la circunferencia es y las coordenadas de con , 1 2 2 = + y x ), 0 , ( ), 0 , ( a B a A − . 1 0 < < a En vez de empezar por sea con la condición Por las condiciones del problema, es el punto medio de llamando a las coordenadas de se tiene Entonces la ecuación de la recta CA es , P ) , ( ´ 0 0 y x P . 1 2 0 2 0 = + y x ´ P ; BC , ( 1 1 y x , C . 2 2 0 1 0 1 ⎩ ⎨ ⎧ = + = y y a x x ), ( 0 0 a x x y y − = es decir . 0 0 0 0 = + − a y x y y x ) Las pendientes de B P´ y de P P´ son respectivamente a x y + 0 0 y . 0 0 y a x + − Por tanto la ecuación de P P´ es . 0 1 ) ( 0 0 0 = + − + + ax a x x y y Las coordenadas del punto intersección de y son: , Q AC P P´ . 1 ) 1 ( , 1 0 2 0 0 0 ⎟ ⎟ ⎠ ⎞ ⎜ ⎜ ⎝ ⎛ + − + + a x a y a x a x Q Denotando por y x, a las coordenadas de Q y despejando los valores de e se obtiene 0 x 0 y . 1 , 1 0 0 − − ax y = − − = y ax x a x Imponiendo ahora la condición se llega a , 1 2 0 2 0 = + y x 1 ) 1 ( ) 1 ( ) ( 2 2 2 2 = − + − − ax y ax x a y mediante operaciones se transforma en la ecuación , 1 1 2 2 2 = − + a y x que es la ecuación de una elipse. Tercera solución: Demostraremos en primer lugar que, dados dos puntos del plano, el conjunto de los puntos (del mismo plano) tales que B A, P 2 2 PB PA + es constante y mayor que es una circunferencia de centro el punto medio de y que tiene a los puntos , 2 AB AB A y B en su interior. En efecto, supongamos ) 0 , ( ), 0 , ( d B d A − = = y sea cualquier punto. Se tiene entonces ) , ( y x P = . 2 2 2 2 2 2 2 2 d y x PB PA + + = + Así que si se tiene que , 4 2 2 2 d k PB PA ≥ = + . 2 2 2 2 2 d d k y x ≥ − = + Sea entonces ahora R el punto donde corta a BC ´ PP (que es perpendicular a ). Este punto BC R satisface , 2 2 2 2 2 AP AR BR BP PR − = − = luego R está en la circunferencia y es distinto de , con lo que P ´. P R = Ahora bien, se tiene , 2 2 ´) ( ´ 2 ´) ( ´ ´ 2 2 2 2 2 2 AB k BP AP PP k BP AP BP AP BP AP − = − − − = − − + = ⋅ donde . ´ ´ 2 2 2 2 BP AP BP AP k + = + = Además, la potencia de respecto de la circunferencia es A , 2 2 2 2 2 2 2 AB k d k d r − = − = − con lo que el segundo punto en el que S AP corta a la circunferencia es tal que ´. ´ CP BP AS = = Como , ´ ´ AP PP CP ⊥ ⊥ se tiene que es paralelo a y es un paralelogramo. AS ´ CP C ASP´ Finalmente, , 4 4 2 ´ ´ 2 ´ ´ 2 2 2 2 2 2 2 2 2 r d k AB k BP AP AP AS AS AP PS PP = − = − + + = ⋅ ⋅ + + = + es decir, Como ' P S AC r = = 2 . , 2r AC BQ AQ = = + el lugar de es la elipse interiormente tangente a la circunferencia dada, con Q A y B como focos. Olimpiada Matemática Española RSME XLVI Olimpiada Matem´atica Espa˜ nola Fase nacional 2010 (Valladolid) Primera sesi´ on (26 de marzo) • Problema 1 Una sucesi´on pucelana es una sucesi´on creciente de diecis´eis n´ umeros impares positivos consecutivos, cuya suma es un cubo perfecto. ¿Cu´ antas sucesiones pucelanas tienen solamente n´ umeros de tres cifras? • Problema 2 Sean N 0 y Z el conjunto de todos los enteros no negativos y el conjunto de todos los enteros, respectivamente. Sea f : N 0 → Z la funci´ on que a cada elemento n de N 0 le asocia como imagen el entero f(n) definido por f(n) = −f n 3 − 3 n 3 donde x es la parte entera del n´ umero real x y {x} = x − x su parte decimal. Determina el menor entero n tal que f(n) = 2010. NOTA: La parte entera de un n´ umero real x, denotada por x es el mayor entero que no supera a x. As´ı 1,98 = 1, −2,001 = −3, 7π − 8,03 = 13. • Problema 3 Sea ABCD un cuadril´ atero convexo. Sea P la intersecci´on de AC y BD. El ´ angulo APD = 60 ◦ . Sean E, F, G y H los puntos medios de los lados AB, BC, CD y DA respectivamente. Halla el mayor n´ umero real positivo k tal que EG+ 3HF ≥ kd + (1 −k)s siendo s el semiper´ımetro del cuadril´ atero ABCD y d la suma de las longitudes de sus diagonales. ¿Cu´ ando se alcanza la igualdad? No est´ a permitido el uso de calculadoras. Cada problema se punt´ ua sobre siete puntos. El tiempo de cada sesi´on es de tres horas y media. Olimpiada Matemática Española RSME XLVI Olimpiada Matem´atica Espa˜ nola Fase nacional 2010 (Valladolid) Segunda sesi´ on (27 de marzo) • Problema 4 Sean a, b, c tres n´ umeros reales positivos. Demuestra que a +b + 3c 3a + 3b + 2c + a + 3b +c 3a + 2b + 3c + 3a +b +c 2a + 3b + 3c ≥ 15 8 • Problema 5 Sea P un punto cualquiera de la bisectriz del ´ angulo A en el tri´ angulo ABC, y sean A , B , C puntos respectivos de las rectas BC, CA, AB, tales que PA es perpendicular a BC, PB es perpendicular a CA y PC es perpendicular a AB. Demuestra que PA y B C se cortan sobre la mediana AM, siendo M el punto medio de BC. • Problema 6 Sea p un n´ umero primo y Aun subconjunto infinito de los n´ umeros naturales. Sea f A (n) el n´ umero de soluciones distintas de la ecuaci´on x 1 +x 2 +· · ·+x p = n, con x 1 , x 2 , . . . , x p ∈ A. ¿Existe alg´ un n´ umero natural N tal que f A (n) sea constante para todo n > N ? No est´ a permitido el uso de calculadoras. Cada problema se punt´ ua sobre siete puntos. El tiempo de cada sesi´on es de tres horas y media. Olimpiada Matemática Española RSME XLVI Olimpiada Matem´atica Espa˜ nola Fase nacional 2010 (Valladolid, 26 y 27 de Marzo) Soluciones oficiales • Problema 1 Una sucesi´on pucelana es una sucesi´on creciente de diecis´eis n´ umeros impares positivos con- secutivos, cuya suma es un cubo perfecto. ¿Cu´ antas sucesiones pucelanas tienen solamente n´ umeros de tres cifras? Soluci´ on: Sea la sucesi´on n, n + 2, . . . , n + 30. Entonces la suma es 1 2 16(2n + 30) = 8(2n + 30). Por tanto, es necesario que 2n + 30 sea un cubo perfecto. Ahora hay que contar el n´ umero de tales n que son impares y verifican 101 ≤ n ≤ 969. Los cubos pares entre 232 y 1968 son 512, 1000 y 1728, que corresponden a valores de n de 241, 485 y 849. Por lo tanto hay exactamente tres sucesiones pucelanas. • Problema 2 Sean N 0 y Z el conjunto de todos los enteros no negativos y el conjunto de todos los enteros, respectivamente. Sea f : N 0 → Z la funci´ on que a cada elemento n de N 0 le asocia como imagen el entero f(n) definido por f(n) = −f n 3 ¸ −3 n 3 ¸ donde x es la parte entera del n´ umero real x y {x} = x−x su parte decimal. Determina el menor entero n tal que f(n) = 2010. NOTA: La parte entera de un n´ umero real x, denotada por x es el mayor entero que no supera a x. As´ı 1,98 = 1, −2,001 = −3, 7π −8,03 = 13. Soluci´ on: Se prueba f´acilmente por inducci´ on que, si n = (a k a k−1 . . . a 0 ) 3 , entonces f(n) = k ¸ j=0 j impar a j − k ¸ j=0 j par a j En efecto, f(0) = 0, f(1) = −1, f(2) = −2. Supongamos que, para todo n menor que 3t, f(n) = ¸ k j=0 j impar a j − ¸ k j=0 j par a j . Entonces, si t = b k . . . b 0 3 , 3t = b k . . . b 0 0 3 , 3t +1 = b k . . . b 0 1 3 , 3t +2 = b k . . . b 0 2 3 . Por lo tanto, como f(3t) = −f(t), f(3t +1) = −f(t) −1, f(3t +2) = −f(t) −2, la propiedad sigue siendo cierta para todo entero n menor que 3t + 2. Luego, para todo n = (a k a k−1 . . . a 0 ) 3 , f(n) = ¸ k j=0 j impar a j − ¸ k j=0 j par a j . De esta forma, se obtiene el menor n = 2020 . . . 20 3 , tal que f(n) = 2010. Este n´ umero contiene 1005 doses; su valor en base decimal es: 3 · 3 2010 −1 4 • Problema 3 Sea ABCD un cuadril´ atero convexo. Sea P la intersecci´on de AC y BD. El ´ angulo APD = 60 ◦ . Sean E, F, G y H los puntos medios de los lados AB, BC, CD y DA respectivamente. Halla el mayor n´ umero real positivo k tal que EG+ 3HF ≥ kd + (1 −k)s siendo s el semiper´ımetro del cuadril´ atero ABCD y d la suma de las longitudes de sus diagonales. ¿Cu´ ando se alcanza la igualdad? Soluci´ on: Probaremos que k = 1 + √ 3 y que la igualdad se da si, y s´ olo si, ABCD es un rect´angulo. X Z Y W H G F E P A B C D Sean W, X, Y y Z cuatro puntos exteriores a ABCD de modo que los tri´ angulos ABW y DCY sean equil´ateros, el tri´ angulo BCX sea is´osceles en X, el tri´ angulo AZD sea is´osceles en Z y BXC = AZD = 120 ◦ . Los cuadril´ ateros WAPB, XBPC, Y CPD y ZDPA son c´ıclicos. Luego, por el teorema de Ptolomeo, se obtiene que: WP = PA+PB, XP √ 3 = PB +PC , Y P = PC +PD, ZP √ 3 = PD +PA Por otro lado, WPY = WPB + 60 ◦ + CPY = WAB + 60 ◦ + CDY = 180 ◦ Luego W, P, Y est´an alineados y, de forma an´ aloga, Z, P, X est´an alineados. Luego: WY = WP +PY = PA+PB +PC +PD = AC +BD XZ = XP +PZ = 1 √ 3 (PB +PC +PD +PA) = 1 √ 3 (AC +BD) Por la desigualdad, triangular: WY ≤ WE +EG+GY , XZ ≤ XF +FH +HZ Luego: AC +BD ≤ AB √ 3 2 +EG+DC √ 3 2 , 1 √ 3 (AC +BD) ≤ BC 2 √ 3 +FH + AD 2 √ 3 Por lo tanto, sumando, AC +BD ≤ AB √ 3 2 +EG+DC √ 3 2 √ 3(AC +BD) ≤ BC √ 3 2 + 3FH +AD √ 3 2 1 + √ 3 (AC +BD) ≤ EG+ 3FH +s √ 3 o sea, EG+ 3FH ≥ 1 + √ 3 d −s √ 3 Luego, si k = 1 + √ 3, entonces EG+ 3FH ≥ kd + (1 −k)s. La igualdad se dar´ a si, y s´olo si, por un lado, W, E, G, Y est´an alineados y, por otro lado, X, F, H, Z tambi´en est´an alineados. Como WE es perpendicular a AB y GY es perpendicular a DC, AB y DC deben ser paralelas y, de forma an´aloga, BC y AD tambi´en deben ser paralelas, luego ABCD debe ser un paralelogramo. Adem´ as, la recta EG es perpendicular a DC, lo que implica que ABCD es un rect´angulo y se comprueba f´ acilmente que si ABCD es un rect´angulo, entonces se da la igualdad. Luego, la igualdad se da si, y s´ olo si, ABCD es un rect´angulo. Ahora, sea un n´ umero real positivo l tal que EG+3HF ≥ ld+(1−l)s. Entonces, si ABCD es un rect´angulo, kd + (1 −k)s ≥ ld + (1 −l)s o sea k(d −s) ≥ l(d −s) Pero la desigualdad triangular implica que d > s, lo que implica que k ≥ l. Luego el n´ umero real buscado es k = 1 + √ 3 y la igualdad se da si, y s´ olo si, ABCD es un rect´angulo. • Problema 4 Sean a, b, c tres n´ umeros reales positivos. Demuestra que a +b + 3c 3a + 3b + 2c + a + 3b +c 3a + 2b + 3c + 3a +b +c 2a + 3b + 3c ≥ 15 8 Soluci´ on: Haciendo a = x 1 , b = x 2 , c = x 3 y llamando s = x 1 +x 2 +x 3 , resulta que el lado izquierdo de la desigualdad se escribe como S = s + 2x 1 3s −x 1 + s + 2x 2 3s −x 2 + s + 2x 3 3s −x 3 Por otro lado, S + 6 = 3 ¸ k=1 s + 2x k 3s −x k + 2 = 3 ¸ k=1 7s 3s −x k con lo que S + 6 = 7s 3s −x 1 + 7s 3s −x 2 + 7s 3s −x 3 = 7s 3 ¸ k=1 1 3s −x k Dado que ¸ 3 k=1 (3s −x k ) = 8s, entonces s = 1 8 ¸ 3 k=1 (3s −x k ), y S + 6 = 7 8 3 ¸ k=1 (3s −x k ) 3 ¸ k=1 1 3s −x k ≥ 63 8 de donde resulta que S = s + 2x 1 3s −x 1 + s + 2x 2 3s −x 2 + s + 2x 3 3s −x 3 ≥ 63 8 −6 = 15 8 La igualdad tiene lugar cuando x 1 = x 2 = x 3 , es decir, cuando a = b = c. • Problema 5 Sea P un punto cualquiera de la bisectriz del ´ angulo A en el tri´ angulo ABC, y sean A , B , C puntos respectivos de las rectas BC, CA, AB, tales que PA es perpendicular a BC, PB es perpendicular a CA y PC es perpendicular a AB. Demuestra que PA y B C se cortan sobre la mediana AM, siendo M el punto medio de BC. Soluci´ on: Sea E el punto de intersecci´on de PA y B C . Si P se mueve sobre la bisectriz AI (I es el incentro), la figura PB C E es homot´etica de s´ı misma con respecto al punto A. Luego E describe una recta que pasa por A. La bisectriz AI corta a la circunferencia circunscrita a ABC en F, que se proyecta en el punto medio A m de BC; si P = F, la recta B C es la recta de Simson de F, luego el lugar geom´etrico de E es la mediana AA m . • Problema 6 Sea p un n´ umero primo y A un subconjunto infinito de los n´ umeros naturales. Sea f A (n) el n´ umero de soluciones distintas de la ecuaci´on x 1 +x 2 +· · · +x p = n, con x 1 , x 2 , . . . , x p ∈ A. ¿Existe alg´ un n´ umero natural N tal que f A (n) sea constante para todo n > N ? Soluci´ on: Para demostrar el enunciado procederemos por contradicci´ on. Supongamos que existe un n´ umero N para el que se cumpla la propiedad anterior. Como el conjunto A es infinito, tomemos a ∈ A mayor que N. Vamos a estudiar el valor de f A (pa) y f A (pa + 1). Por hip´ otesis, se cumple que f A (pa) = f A (pa + 1). Sea S = (s 1 , s 2 , . . . , s p ) soluci´ on de la ecuaci´ on x 1 +x 2 +· · · +x p = n, s 1 , s 2 , . . . , s p ∈ A Entonces, cualquier permutaci´ on de los ´ındices da lugar a una nueva soluci´ on de la ecuaci´ on (posiblemente repetida si se permutan valores iguales). Diremos que una soluci´ on S = (s 1 , s 2 , . . . , s p ) es asociada a una soluci´on S = s 1 , s 2 , . . . , s p si la primera se obtiene a partir de la segunda mediante permutaci´ on de ´ındices. Sea S = (s 1 , s 2 , . . . , s p ) una soluci´ on del problema y sea Q = (q 1 , q 2 , . . . , q p ) una soluci´ on asociada a S con la propiedad que q 1 = q 2 = · · · = q r 1 = q r 1 +1 = q r 1 +2 = · · · q r 1 +r 2 , y as´ı de manera sucesiva hasta llegar a q r 1 +r 2 +···+r k = q p . En otras palabras, Q se obtiene a partir de S agrupando los valores s i que son iguales. En particular, r 1 + r 2 + · · · + r k = p. Con esta notaci´ on, el n´ umero de soluciones asociadas a S (contando tambi´en S) es igual a p! r 1 !r 2 ! · · · r k ! . Obs´ervese que si todos los r i son estrictamente menores que p, entonces dicha expresi´on es congruente con 0 m´odulo p, puesto que el cociente de factoriales es un n´ umero natural y en el denominador no hay ning´ un t´ermino m´ ultiplo de p. Ya tenemos todas las herramientas que necesit´ abamos. Volviendo al problema original, observar que (a, a, . . . , a) . .. . p es soluci´on de x 1 +x 2 +· · · +x p = pa. Por lo tanto f A (pa) ≡ 1 (m´od p): la soluci´ on (a, a, . . . , a) . .. . p no se asocia a ninguna otra, mientras que cualquier otra soluci´ on de la ecuaci´ on x 1 +x 2 +· · · +x p = pa tiene un n´ umero m´ ultiplo de p de asociadas. Por otro lado no existen soluciones de x 1 +x 2 +· · · +x p = pa+1 con todas las x i iguales (su valor tendr´ıa que ser a + 1 p ), con lo que seg´ un lo anterior f A (pa + 1) ≡ 0 (m´od p) y llegamos a una contradicci´ on. a) La bola vuelve a S y entonces el jugador pierde. b) La bola llega a G y entonces el jugador gana. Se pide la probabilidad de que el jugador gane y la duración media de las partidas. Problemas de la XXIX Olimpiada 1.- En una reunión hay 201 personas de 5 nacionalidades diferentes. Se sabe que, en cada grupo de 6, al menos dos tienen la misma edad. Demostrar que hay al menos 5 personas del mismo país, de la misma edad y del mismo sexo. Solución (Mª Gaspar Alonso-Vega) Si en cada grupo de 6 personas, 2 son de la misma edad, sólo puede haber 5 edades diferentes, ya que, si hubiese 6 edades diferentes, eligiendo una persona de cada edad tendríamos 6 personas de edades distintas contra la hipótesis. Como 200 = 2 · 100 + 1⇒ al menos hay 101 personas del mismo sexo. 101 = 5 · 20 + 1 ⇒ al menos hay 21 personas de la misma edad y sexo. 21 = 4 · 5 + 1 ⇒ al menos hay 5 personas de la misma nacionalidad, edad y sexo. 2.- Escrito el triángulo aritmético: 0 1 1 4 ............. 1991 1992 1993 3 5 7 ............................ 3983 3985 4 8 12 ........................................ 7968 ......................................................................... 2 3 donde cada número es la suma de los dos que tiene encima (cada fila tiene un número menos y en la última sólo hay un número). Razonar que el último número es múltiplo de 1993. Solución. Si representamos los elementos de la primera fila por a0, a1, a2, ........ los elementos de la segunda serán: a0 + a1, a1 + a2, a2 + a3, .............. los de la tercera serán : a0 + 2a1 + a1, a1 + 2a2 + a3, .............. para la cuarta : a0 + 3a1 + 3a1 + a1, a1 + 3a2 + 3a3 + a4,............ Supongamos que los dos primeros elementos bp,0 y bp,1 de la fila p-ésima son:  p − 1  p − 1  p − 1 b p ,0 =  ; a 0 +  a 1 +.......+ a  0   1   p − 1 p−1  p − 1  p − 1  p − 1 b p ,1 =  a 1 +  a 2 +.......+ a  0   1   p − 1 p entonces, el primer elemento de la fila siguiente será : p p  p b p+1,0 =  a 0 +  a 1 +.......+ a p 0  1  p (*) en nuestro caso la primera fila tiene 1994 elementos, la segunda 1993, ... y la última corresponde a p + 1 = 1994 y su único elemento será 1993 1993 1993 b 1994 =  •0+  • 1+.......+  • 1993  0   1  1993 .. . AC.1 en la sucesión hay infinitos términos de la forma 10p-1 .2Rr”.. 99 = 102 . “Si I.9.. se verifica: IO2 = R2 . estudiar: a) Cuántas figuras de 4 puntos existen con las condiciones del enunciado..2r ≥ 0 es una consecuencia del teorema de Euler.... Consideremos la sucesión: 9. Finalmente 999. 999 = 103 .Se dan 16 puntos formando una cuadrícula como en la figura: D A De ellos se han destacado A y D.. r y R los radios de las circunferencias inscrita y circunscritas. R . Bellot) La desigualdad propuesta. 3.. son múltiplos de tres... 999. p ≠ 5 la afirmación queda demostrada. 4.1 con p ≠ 2....Justificar razonadamente que.. es decir. 99.1 entonces si p es primo con 9 (p ≠ 3)..9 = 9 · 111. es primo con 9 luego divide a 111. c) Si cada punto se designa por un par de enteros (Xi.. .Yj| extendida a los seis pares AB. AD.9 = 10n ..2r) ≥ 0 ⇒ R .. 111111. 5.1 ≡ 1 (mód p) si p ≠ 2. .2r ≥ 0. En ese conjunto de cuaternas..999. Entonces se tiene: 9 = 10 .. en cualquier triángulo.. el diámetro de la circunferencia inscrita no es mayor que el radio de la circunferencia circunscrita.. Puesto que.999. O son el incentro y el circuncentro de un triángulo. p ≠ 5 y p primo.. Solución (F. razonar que la suma: |Xi .Demostrar que para todo número primo p distinto de 2 y de 5.. p divide al producto. Queda el caso p = 3 que es evidente ya que los infinitos números: 111.. BC. existen infinitos múltiplos de p de la forma 1111. b) Cuántas de ellas son geométricamente distintas.. por el teorema de Fermat: 10p-1 .1 (escrito sólo con unos). Se pide fijar de todos los modos posibles otros dos puntos B y C con la condición de que las seis distancias determinadas por los cuatro puntos sean distintas.. CD es constante. BD..9 (el último tiene n nueves)..1.. Solución (Alvaro Begué Aguado) Veamos primero que p tiene infinitos múltiplos de la forma 999..   es múltiplo de 1993 para todo k menor que 1993 y por tanto b1993 es  k  múltiplo de 1993.... no deducibles unas de otras por transformaciones de igualdad....Xj| + | Yi .1.1993 Al ser 1993 primo..1.. Entonces IO2 = R (R .1. Yi). Por tanto existen 32 posiciones posibles y 8 “originales”. Por el mismo motivo los sumandos correspondientes a AC y CD valen entre los dos siempre 6. El esquema del centro contiene 3 posiciones “originales” y cada una de ellas genera otras cuatro por aplicación de las dos simetrías. en total 16. BC.1) para cada una de ellas dibujamos us esquema con las posibles posiciones del punto C. Sólo queda el sumando |Xi . . El correspondiente a AD es constante y vale 3+3 = 6. la diagonal principal siempre contiene puntos “prohibidos” C1 C2 B A C4 C3 A A C3 D B C1 D C2 B C1 D C1 El esquema de la izquierda contiene 4 posiciones “originales” y cada una de ellas genera otras cuatro por aplicación de las dos simetrías. Las tres posiciones “fundamentales” (no deducibles unas de otras por las simetrías anteriores) son aquellas en las que B está en los puntos de coordenadas (0.1). Un criterio general para prohibir ubicaciones es localizar aquellos puntos que estan en la “mediatriz” de dos puntos ya situados. en total 12. AC.Solución El problema admite dos ejes de simetría coincidentes con las diagonales del cuadrado. esto contesta a los apartados a) y b).Xj| + | Yi . AD.0) y las de D(3. (0. en total 4. El esquema de la derecha contiene 1 posición “original” que genera otras cuatro por aplicación de las dos simetrías.2) y (1. La suma completa es entonces constante y vale 6 + 6 + 6 + 3 = 21. la posición de B en gris y las de Ci en blanco. Los correspondientes a AB y BD valen en conjunto siempre 6 ya que A está en fila inferior y columna izquierda y D en la fila superior y columna derecha. Usaremos coordenadas enteras con origen en A. Clasificaremos las soluciones posibles por la posición del punto B respecto del vértice A.Yj| correspondiente a BC que por simple comprobación en todos los casos “originales” vale siempre 3.0). Como A y D son dados y fijos. nosotros supondremos que el origen está en A con lo que las coordenadas de A son (0. BD y CD. los seis sumandos corresponden a las parajas AB. Las posiciones “prohibidas” se dibujan en negro. Para el apartado c) hay que suponer que los enteros asignados a cada punto son sus coordenadas en un origen cualquiera. . Bellot) Podemos representar el desarrollo del juego mediante un diagrama en árbol: S 1 1/3 A 1/3 S 1/2 C D B D G C D S A C D C 1/3 A S C B D G La probabilidad de que el juego tenga longitud 2 es 1 3 1 1 1 2 La probabilidad de que el juego tenga longitud 4 es : • • • 2 • 2 = 2 3 2 3 3 1 1 1 1 22 La probabilidad de que el juego tenga longitud 6 es : • • • 2 • 2 • 2 • = 3 . etc. b) La bola llega a G y entonces el jugador gana.Una máquina de juego de un casino tiene una pantalla en la que se ofrece un esquema como el de la figura. A B Se pide la probabilidad de que el jugador gane y la duración media de las partidas. Para comenzar el juego aparece una bola en el punto S.6. S D G Solución (F. la duración media M de un juego es la suma de cada longitud por la probabilidad respectiva : n ∞ ∞ 2 n−1 2   n M = ∑ n 2n = ∑ n=1 3 n=1  3  serie aritmético-geométrica que se suma por el mismo método que la geométrica: 2   2 M 2  3 M− M = = + = 2 ⇒ M = 2 •3= 6 2 3 3 3 1− 3 2 . A cada impulso que recibe del jugador. en general 3 2 3 3 3 la probabilidad de que el juego tenga longitud 2n es: 2 n−1 3n Entonces. esa bola se mueve hasta una de las letras inmediatas con la misma probabilidad para cada C una de ellas. La partida termina al ocurrir el primero de los dos hechos siguientes: a) La bola vuelve a S y entonces el jugador pierde. .: 1 2 22 1 P = 2 + 3 + 4 +..etc..La probabilidad P de ganar será la suma de las probabilidades de ganar en 4 pasos más la de que gane en 6 pasos ..= 3 3 3 3 .. . Para cada par de puntos P y Q. Calcular las medidas de los ángulos del triángulo BCD. tal que OC = c.Con 21 fichas de damas.OLIMPIADA MATEMÁTICA ESPAÑOLA Fase Nacional 1994 (Madrid) Primera sesión 1.. unas blancas y otras negras. La bisectriz de su ángulo C corta al lado opuesto en el punto D. sin que en la expresión aparezcan razones trigonométricas.. 2. Segunda sesión 4.. Probar que m + n es par. prescindiendo de una de las regiones. Razonar cuál es el lugar geométrico de esa proyección. Q están en una esfera. Se da cuenta de que. Z. Demostrar que siempre hay cuatro fichas del mismo color situadas en los vértices de un rectángulo. se forma un rectángulo de 3x7. los cuatro puntos O. Para ello recurre a seis regiones que le transmiten los datos de la siguiente tabla: Región A B C D E F Soleados o lluviosos 336 321 335 343 329 330 Inclasificables 29 44 30 22 36 35 La persona encargada de la encuesta no es imparcial y tiene esos datos más detallados. 3. Conocidos n y m hallar el número de lados distintos que quedan en el interior del polígono y el número de vértices distintos que quedan en ese interior. la observación da un número de días lluviosos que es la tercera parte del de días soleados. Sobre la arista Z se toma un punto fijo C.Demostrar que si entre los infinitos términos de una progresión aritmética de números enteros positivos hay un cuadrado perfecto. 5. Sobre X e Y se toman respectivamente dos puntos variables P y Q de modo que la suma OP + OQ sea una constante dada k. de modo que cada lado de esos m triángulos lo es también de otro triángulo contíguo o del polígono dado. entonces infinitos términos de la progresión son cuadrados perfectos.. 6.Un polígono convexo de n lados se descompone en m triángulos. Y. Razonar también cuál es el lugar geométrico de W. . con los interiores disjuntos. C.Sea OXYZ un triedro trirectángulo de vértice O y aristas X. siendo iguales sus ángulos B y C.El ángulo A del triángulo isósceles ABC mide 2/5 de recto. Expresar la medida a del lado BC en función de la medida b del lado AC.. cuyo centro W se proyecta sobre el plano OXY. Razonar cuál es la región de la que prescindirá.Una oficina de Turismo va a realizar una encuesta sobre número de días soleados y número de días lluviosos que se dan en el año.. P. . Sobre la arista Z se toma un punto fijo C.. cuyo centro W se proyecta sobre el plano OXY. G  0. G y H en la figura) F P y al ser el triedro trirectángulo.  2 2    2 2  2 2 2 2 2 k del plano XOY. Parametrizando con la distancia OP= λ tenemos trivialmente en la referencia OXYZ la siguientes coordenadas: P(λ.c). Q(0.0 .0). . C(0.  . Y.. Como (a + d)2 = a2 + 2ad + d2 = a2 + (2a + d)d. C.. Sobre X e Y se toman respectivamente dos puntos variables P y Q de modo que la suma OP + OQ sea una constante dada k.0... Z Solución En la figura se muestran con trazo discontínuo las circunferencias que resultan de intersecar la esfera con los planos coordenados. basta tomar k = 2a + d para obtener otro cuadrado en la progresión. H  .. Z... a2 + 2d. Razonar también cuál es el lugar geométrico de W. El lugar de W es una recta paralela a la 2 c anterior situada en el plano z = .. Para cada par de puntos P y Q. . . más concretamente es la intersección de los planos: 2  k x + y = 2  c  z=  2 El lugar de F es la recta x + y = . entonces infinitos términos de la progresión son cuadrados perfectos. Sea la progresión: a2. los cuatro puntos O. G y H estan en los puntos medio de los segmentos PQ. a2 + d. F.0. tal que OC = c.Problemas de la XXX Olimpiada Madrid 1994 1. P...0.0). W  . .k-λ. Solución Bastará probar que a partir de un cuadrado perfecto podemos construir otro. Las C proyecciones del centro W de la esfera sobre G W H estos planos coinciden con los centros de estas Y O Q circunferencias (denotados F. λ k−λ   k − λ c  λ c  λ k − λ c F . 2. QC y CP X que son diámetros de sus circunferencias.a2 + kd.Sea OXYZ un triedro trirectángulo de vértice O y aristas X.. Q están en una esfera.  . Razonar cuál es el lugar geométrico de esa proyección.Demostrar que si entre los infinitos términos de una progresión aritmética de números enteros positivos hay un cuadrado perfecto. Para los lados se tiene: DC = AD = a. ∠ ADC = 108º por tanto ∆BCD y ∆ADC son isósceles y ademas ∆BCD es semejante al ∆ABC. Expresar la medida a del lado BC en función de la medida b del lado AC. ∠ CDB = ∠ BDC = 72º en el triángulo ADC ∠ DAC = ∠ ACD = 72º. b 2 2 . Expresando la proporcionalidad derivada de la semejanza anterior: b-a A a D a b b−a a a  a = ⇔ a 2 = b 2 − ab ⇔ a 2 + ab − b 2 = 0 ⇔   + − 1 = 0 b b a b 2 B C y resolviendo queda ( 5 − 1) b a 5 −1 = ⇔a = es decir a es la sección áurea de b. ∠ ABC = ∠ ACB = 72º en el triángulo CBD ∠ BCD = 36º. sin que en la expresión aparezcan razones trigonométricas. la suma de días soleados o lluviosos de las restantes ha de ser múltiplo de 4. la observación da un número de días lluviosos que es la tercera parte del de días soleados. siendo iguales sus ángulos B y C. La bisectriz de su ángulo C corta al lado opuesto en el punto D. prescindiendo de una de las regiones.a. Se da cuenta de que. Esta suma vale para las seis regiones 1994 que dividido entre 4 da resto 2. BD = b .3. Calcular las medidas de los ángulos del triángulo BCD. Para ello recurre a seis regiones que le transmiten los datos de la siguiente tabla: Región A B C D E F Soleados o lluviosos 336 321 335 343 329 330 Inclasificables 29 44 30 22 36 35 La persona encargada de la encuesta no es imparcial y tiene esos datos más detallados.Una oficina de Turismo va a realizar una encuesta sobre número de días soleados y número de días lluviosos que se dan en el año.El ángulo A del triángulo isósceles ABC mide 2/5 de recto. Suprimiendo esta región quedan entre las cinco restantes 416 días lluviosos y 3·416 = 1248 días soleados. Razonar cuál es la región de la que prescindirá. 4... Solución Al suprimir una región. Solución Con los datos del enunciado tenemos: en el triángulo ABC ∠ BAC = 36º. El único dato de esta columna que da resto 2 al dividirlo entre 4 es 330 correspondiente a la región F. Demostrar que siempre hay cuatro fichas del mismo color situadas en los vértices de un rectángulo.2 = n + 2v . hay lados interiores distintos. para que se cumplan las condiciones del enunciado. de modo que cada lado de esos m triángulos lo es también de otro triángulo contíguo o del polígono dado. cada triángulo se descompone en dos y el número de triángulos es ahora: n + 2v . es decir. por ejemplo el negro.2. Por otra parte si dos números son iguales sus filas forman rectángulo.. Supongamos que el número de vértices v sólo depende de m y n. 6.5. uniendo un vértice del polígono con los otros. El problema tendría solución en un tablero de 3x6 tal como se muestra en la figura. De este modo cada fila representa un número escrito en base 3 2. Al añadir un vértice hay dos posibilidades: a) El vértice está en el interior de un triángulo. entonces.2 + 2 = n + 2(v + 1) . quitando los anteriores quedan 6 para 7 filas por lo que necesariamente hemos de repetir y formar rectángulo. En particular 3m . debe unirse a cada uno de los tres vértices del triángulo que se divide en tres y el número de triángulos ahora es: n + 2v . entonces hay que unirlo con el vértice opuesto de cada uno de los dos triángulos que comparten ese lado. se forma un rectángulo de 3x7. En conclusión: m− n + 2 m = n + 2v . luego todas las filas han de representar números distintos. Supongamos que hay v vértices interiores y n + 2v . Razonemos por inducción sobre v.2 triángulos.2 triángulos. Asignaremos el color blanco a la cifra 0 y el negro a la 2 cifra 1. Conocidos n y m hallar el número de lados distintos que quedan en el interior del polígono y el número de vértices distintos que quedan en ese interior. Solución Dispondremos el tablero en posición vertical.2 + 2 = n + 2(v + 1) . Con tres cifras en base dos existen 23 = 8 números distintos.n son interiores. 2 luego m y n tienen la misma paridad y m + n es par. con 7 filas y 3 1 columnas. b) El vértice está en un lado. Por la consideración anterior hemos de excluir los números 000 y 111.n es par.Con 21 fichas de damas. Solución Como hay m triángulos.2 ⇒ v = 2 ..Un polígono convexo de n lados se descompone en m triángulos. Probar que m + n es par. necesariamente habrá 5 un rectángulo ya que no podemos colocar en ninguna fila dos fichas 6 negras y sólo podemos llenar un máximo de 5 filas en total sin formar rectángulo. se divide en n . hay 3m lados. y como lado interior 3m − n pertenece a dos triángulos. de ellos 3m . unas blancas y otras negras. con los interiores disjuntos. Si no hay ningún vértice interior (v = 0). En primer lugar es fácil ver que si en una fila se colocan todas las 4 fichas del mismo color.2. . OLIMPIADA MATEMÁTICA ESPAÑOLA Fase nacional 1995 (Castellón) Primera sesión 1. AB es un segmento fijo y C un punto variable dentro de él. 6. halla las soluciones enteras de la ecuación: p. ACB' y CBA' en el mismo semiplano definido por AB... ABC' en el semiplano opuesto. pero de tal forma que no haya ningún círculo dentro de otro.n = 0 nx3 .Recortamos varios círculos de papel (no necesariamente iguales) y los extendemos sobre una mesa de modo que haya algunos solapados (con parte interior común). Prueba que es imposible ensamblar las piezas que resultan de recortar las partes no solapadas y componer con ellas círculos distintos.5mnx . Se construyen triángulos equiláteros de lados AC y CB. Demuestra: a) Las rectas AA'.En la figura. Demuestra que: Segunda sesión 4. Calcula el valor mínimo de S(A) 2. b y c unidades.2m3 . hallar el lugar geométrico de P cuando C varía en el segmento AB.y 5.2 m2 x2 . existe un triángulo no obtusángulo cuyos lados miden a. que tengan la propiedad de que si a..n2 = 0 (n no nulo) tengan una raíz común... Se denomina S(A) a la suma de los perímetros considerados en la definición de A. . la primera tendrá dos raíces iguales y determina entonces las raíces de las dos ecuaciones en función de n.Se consideran conjuntos A de cien números naturales distintos. y otro de lado AB.. b y c son elementos cualesquiera de A (iguales o distintos). b) Si llamamos P al punto común a las tres rectas del apartado a). 3. BB' y CC' son concurrentes.Siendo p un número primo.Demuestra que en el caso de que las ecuaciones: x3 + mx .Por el baricentro G de un triángulo ABC se traza una recta que corta al lado AB en P y al lado AC en Q.(x + y) = x. B''. B'' y C'' de los tres triángulos forman un triángulo equilátero.lugar geométrico de P cuando C varía en el segmento AB. . C'' y P están sobre una circunferencia. c) Los centros A''. d) Los puntos A''. m no puede ser no obtuságulo. Solución: La frontera de las piezas recortadas (que no sean círculos completos) está formada por arcos cóncavos y convexos (vistos desde fuera) que se cortan en puntos que llamaremos vértices. pero nunca dos convexos ya que éstos únicamente provienen de la frontera de los círculos iniciales. por tanto el que da lugar al mínimo S(A).99 2 ≥ 0 ⇔ n ≥ 99 + 99 2 + 99 2 ⇔ n ≥ 99( 1 + 2 ) ⇔ n ≥ 240 . n. . 241.. será m ≥ n + 99. Solución: Si n es el menor de los elementos de A y m el mayor. Luego S ( A) = 30000 • (240 + 241 + 242+. si m es lo menor posible. es seguro que el conjunto no cumple la condición del enunciado pues m2 ≥ (n+99)2 ≥ 2n2 y el triángulo de lados n. 100 • ( 240 + 339 ) = 868500000 unidades.. Se denomina S(A) a la suma de los perímetros considerados en la definición de A. los ángulos que forman los arcos en cada vértice no son de 0º ni de 180º ya que excluimos las tangencias interiores.. b y c unidades. de ellos habrá 30000 de longitud 240. b y c son elementos cualesquiera de A (iguales o distintos). 242. En particular la condición se cumple para el conjunto: A = {240. existe un triángulo no obtusángulo cuyos lados miden a.. 2.. Además. etc.Se consideran conjuntos A de cien números naturales distintos. Prueba que es imposible ensamblar las piezas que resultan de recortar las partes no solapadas y componer con ellas círculos distintos. m sea no obtusángulo debe ocurrir que m2 ≤ 2 n2 . En un vértice pueden concurrir dos arcos cóncavos o uno cóncavo y otro convexo. pero de tal forma que no haya ningún círculo dentro de otro. m = n + 99 deberá ser (n + 99)2 ≤ 2n2 . Este es. Calcula el valor mínimo de S(A). que es 1003 = 1000000.. con lo que el número de lados en total será de 3000000.198n . Si n < 240. n. que tengan la propiedad de que si a.Recortamos varios círculos de papel (no necesariamente iguales) y los extendemos sobre una mesa de modo que haya algunos solapados (con parte interior común). tendrá sus elementos respectivamente iguales o mayores que los de éste. al tener A cien elementos distintos. 2 .. Para que el triángulo isósceles de lados n. El número de triángulos que debe considerarse es el de variaciones ternarias con repetición de los elementos de A. o sea: n 2 .+339 ) = 30000 • Este es el valor mínimo buscado.Olimpiada Española 1995 1. 339} Cualquier otro conjunto que cumpla la condición.. otros tantos de longitud 241.. 1-x = AP. tenemos: D A Por semejanza de ∆AQP y ∆QMC: QC MC MC = = QA AP 1 − x PB GB 1 = = .. el punto P es un vértice de la pieza. De no ser así pueden encontrarse contraejemplos como el “despiece” que se muestra en la figura: 2 3. La igualdad se alcanza para PB = x = 1 1 ⇔ MC = ⇔ PQ paralela al lado BC. La tangente al círculo en P deja a todos los arcos en un mismo semiplano. Existe al menos un punto P de la frontera de dicho círculo en el que concurren tres o más arcos de la frontera de las pieza ensambladas (P es vértice de dos o más piezas). y avanzando en P este sentido. Demuestra que: PB QC 1 • ≤ PA QA 4 Solución: Dupliquemos el triángulo trazando AD paralela a BC y CD paralela a BA como muestra la figura y tomemos la longitud del lado AB como unidad.Por el baricentro G de un triángulo ABC se traza una recta que corta al lado AB en P y al lado AC en Q. Nota. Elegido un sentido de rotación en P a partir de la tangente. MD GD 2 Q G P C B M Por semejanza de ∆GPB y ∆GMD: Luego: MD = 2x y MC = 1 .Supongamos que tenemos un círculo obtenido ensamblando piezas recortadas. Sustituyendo en el primer miembro de la relación del enunciado queda: PB QC 1 x (1 − 2 x ) 1 2 • ≤ ⇔ ⇔ 9 x 2 − 6 x + 1 ≥ 0 ⇔ ( 3x − 1) ≥ 0 2 ≤ PA QA 4 4 (1 − x ) Relación válida para cualquier x. Llamando M a la intersección de CD con la recta PQ y x = PB. Como el arco que forman dichas piezas no puede ser ni 0º ni 180º. el primer arco que encontramos es convexo y el último cóncavo.que quedan excluidas las tangencias interiores.2x. Esto es contradictorio pues en ningún vértice pueden concurrir dos arcos convexos vistos desde fuera. Por lo tanto es necesario que existan dos arcos consecutivos uno convexo y el otro cóncavo los cuales forman parte de la frontera de una de las piezas ensambladas. 3 3 . Hay que entender en el enunciado. Al ser p primo sólo hay cuatro posibilidades: a .Halla las soluciones enteras de la ecuación: p·(x + y) = x·y siendo p un número primo.. entonces a = 1 . x = 0. la primera tendrá dos raíces iguales y determina entonces las raíces de las dos ecuaciones en función de n. también lo será (β.1 divide a p. x = 2p ⇒ y = 2−1 p( p + 1) = p+1. Entonces α3 + mα = n (1) y sustituyendo en la segunda ecuación se obtiene.1 son primos entre sí.1 = p. sustituyendo en la primera ecuación y despejando n queda: n = α3 . (p(1-p). p(p+1)). Si p divide a x.α3 = 0 en contra de lo supuesto. x = p·a.1 = -p. 0). 2  -3α (ii) (i) Si m = -α2. α).2m3 .1 = ± p. Solución: Ya que p es primo. Examinemos todos los casos. tras hacer operaciones: 6m α4 + 8m2 α2 +2m3 = 0 Supongamos m ≠ 0. x = p(p + 1) ⇒ y = p + 1− 1 p(1 − p) iiii) a . si (α. 1− p − 1 En resumen las soluciones son: (0. p+1). (p-1. x = p(1 . tiene la raíz α doble. β) es solución.5mnx .2 m2 x2 . luego a . 2p).Demuestra que en el caso de que las ecuaciones: x3 + mx . Solución: Sea α la raíz común de ambas ecuaciones. (p(p+1). i) a .p) ⇒ y = = p−1. entonces a = 0.p. nx3 . entonces a = 2. (a∈Z) la ecuación se puede poner como: pa p( pa + y)) = pay ⇒ pa + y = ay ⇒ y = ya que a es entero a −1 además a y a . De la ecuación resulta que p divide a x o p divide a y. Como la ecuación es simétrica respecto de x e y. ii) a . p(1-p)). iii) a . 5.1 = 1.4. y = 0 2p = 2p . entonces simplificando la relación anterior queda: 3 α4 + 4m α2 +m2 = 0 (2)  −α 2 (i) Resolviendo (2) respecto de m obtenemos m =  Analicemos cada caso. y por la simetría añadimos (p+1.. Por tanto (i) queda descartado. p ≠ 0 y p ≠ 1. efectivamente.n = 0 .1 = -1. sustituyendo en la primera ecuación y despejando n queda: n = α3 -3 α3 = -2 α3 y la primera ecuación queda: 2 x 3 − 3α 2 x + 2α 3 = ( x − α) ( x 2 + αx − 2α 2 ) = ( x − α) ( x + 2α) que. . p-1). (2p. (ii) Si m = -3α3. entonces a = p + 1.n2 = 0 (n ≠ 0) tengan una raíz común.1 = ± 1 y a . A C' C'' C B'' A'' P A' B b) Como P está definido por la intersección de la recta CC’ con la circunferencia circunscrita al triángulo ABC’ el lugar pedido es el arco APB de esa circunferencia. hallar el lugar geométrico de P cuando C varía en el segmento AB. Si m = 0. por ello. d) Los puntos A’’. ACB’ y CBA’ en el mismo semiplano definido por AB. Se construyen triángulos equiláteros de lados AC y CB. C' 2 C'' A A'' C B'' P B 6. c) Los lados del triángulo son perpendiculares a las cuerdas PA . BB’ y CC’ son concurrentes.En la figura. De modo análogo se ve que P está alineado con AA’. Evidentemente (arco capaz) ∠APB=120º y PC’ es su bisectriz con lo que ∠APC=∠CPB=60º y P ha de estar en las circunferencias circunscritas a los triángulos ACB’ y BCA’. B’’ y C’’ de los tres triángulos forman un triángulo equilátero. Como ∠CPB’=120º y ∠CPB=60º sumando queda: ∠BPB’=180º y P está alineado con BB’. y otro de lado AB.α) resulta −2α 3 ( x − α)( x + 5α) = 0 . c) Los centros A’’. B’’. PB y PC que forman ángulos de 60º o 120º. C’’ y P están sobre una circunferencia. AB es un segmento fijo y C un punto variable dentro de él. 2 Entonces la segunda ecuación es de la forma −2α 3 ( x 3 + 9αx 2 + 15α 2 x + 25α 3 ) = 0 . Demuestra: a) Las rectas AA’.. b) Si llamamos P al punto común a las tres rectas del apartado a). y. dividiendo por (x . B' . cuyas raíces son α y −5 α siendo doble la última. Por tanto las tres circunferencias se cortan en P.de n = -2 α3 obtenemos α = 3 − n . A' B' Solución: a) Se traza la circunferencia circunscrita al triángulo ABC’ y se llama P a la intersección de CC’ con ella. las dos ecuaciones son iguales y sus tres raíces son las mismas pero la primera no tiene dos raíces iguales por lo que en el enunciado debería haberse añadido m ≠ 0. entre sí forman ángulos iguales de 60º. ABC’ en el semiplano opuesto. B’’. .d) Basta comprobar que los centros C’’. A’’ y P verifican el teorema de Tolomeo: PC ' ' A' ' B ' ' = PA' ' B ' ' C ' ' + PB ' ' A' ' C ' ' ⇔ PC ' ' = PA' ' + PB ' ' ⇔ AB = AC + CB siendo la última igualdad evidente por construcción. ¿Qué volumen de agua cabe en el recipiente formado?. entonces B no vigila a A. éste vigila al tercero. 10 agentes cualesquiera pueden ser numerados de forma que el primero vigila al segundo.. 5. Además. .c números reales..La figura de la izquierda se compone de seis pentágonos regulares de lado 1m..OLIMPIADA MATEMÁTICA ESPAÑOLA Fase nacional 1996 (Tarragona) Primera sesión 1. 3. 6.Discutir la existencia de soluciones de la ecuación según los valores del parámetro real p.. entonces: Segunda sesión 4.. Se dobla por las líneas de puntos hasta que coincidan las aristas no punteadas que confluyen en cada vértice. Se consideran las funciones .Sean a.. Se sabe que si el agente A vigila al agente B. Cada uno de ellos vigila a algunos de sus colegas...Los números naturales a y b son tales que: es entero...En Port Aventura hay 16 agentes secretos. y resolverla siempre que sea posible. Demostrar que el máximo común divisor de a y b no es mayor que 2..Sea G el baricentro del triángulo ABC. b . Si se verifica demostrar que el triángulo es isósceles. el último (décimo) vigila al primero. Demostrar que también se pueden numerar de este modo 11 agentes cualesquiera.. Sabiendo que demostrar que si -1 < = x < = 1. . Si se verifica: AB + GC = AC + GB demostrar que el triángulo es isósceles. en los triángulos CC’A y BB’A tenemos por la desigualdad triangular: mb + b c > c. la relación del enunciado se escribe: 2  a 2 + c2 b2  − − 3 2 4  a 2 + b2 c2  − .d (a. Tarragona 1996 a+1 b+1 + es entero. m c + > b . Fase Nacional. Llamando B’ y C’ a los puntos medios de AC y Ab respectivamente. b a ab Sea d = m. entonces a 2 + b 2 + ab es divisible por d2 y también lo son a2 + b2 y a + b...Olimpiada Matemática. . Teniendo en cuenta el teorema de la mediana. se tiene : a + b ≥ d2 ⇔ a + b ≥ d 2. 2 4  c− b= multiplicando y dividiendo por la expresión conjugada queda: 3 2 c − b2 ) c + b 2 4(  c−b= ⇔ ( c − b) m c + m b −  =0  3 mc + m b 2  Probaremos que el segundo factor es positivo. Como ab es divisible por d2.b). 2 2 Sumando ambas desigualdades se obtiene el resultado. Solución: Primera solución. Demostrar que el máximo b a a+b 1.Los números naturales a y b son tales que: común divisor de a y b no es mayor que Solución: Se tiene: a + 1 b + 1 a2 + b2 + a + b + = .Sea G el baricentro del triángulo ABC. Segunda solución.c. y al ser a y b naturales. de donde se deduce la conclusión. C’ a los puntos medios de los lados BC. AC y AB respectivamente y dividiendo por dos la condición del enunciado podemos escribirla como: B' C' M G C' A C' G B' A B' G .Sean a. y entonces AA’ además de mediana es altura y el triángulo es isósceles. 2 2 4  2 4  1 por otra parte. 2 2 como − 1 ≤ x ≤ 1. -1 . Llamando M al punto medio de C’B’ . B’. Así pues g( x ) ≤ 1+ x 1− x + + 1− x2 = 2 − x2 ≤ 2 2 2 . f (1) ≤ 1. c números reales. c tales que se tenga idénticamente: f (x) = Ax (x + 1) + Bx (x − 1) + C(x 2 − 1) . entonces: f (x ) ≤ 5 4 f (0) ≤ 1. Sabiendo que g (x ) = cx 2 + bx + a .. demostrar que si -1≤ x ≤ 1. Particularizando para x = 1. g( x) = x 2 f   . y g ( x) ≤ 2. por tanto C’B’ ha de ser perpendicular a AA’. + = + 2 2 2 2 C B A' es decir los puntos C’ y B’ están en una elipse de focos A y G. Se consideran las funciones: f (x) = ax 2 + bx + c.A Llamando A’. 1 + x ≥ 0. para x ≠ 0.0 y resolviendo el sistema queda: f ( x) = f (1) f ( −1) x( x + 1) + x( x − 1) + f ( 0)(1 − x 2 ) 2 2 ∀x ∈ R De aquí se deduce: f (x ) ≤ 1 1 x (x + 1) + x (x − 1) + 1 − x 2 . resulta x x 5  1 5 f (x) ≤ (1 + x) + ( 1 − x) + 1 − x 2 = − x 2 + x + 1 = −  x −  ≤ . 3. 1 − x ≥ 0 y 1 − x 2 ≥ 0 . Solución: Podemos conseguir coeficientes A. B. Entonces  x g( x ) = f (1) f (−1) ( 1 + x) + (1 − x) + f (0)( x 2 − 1) 2 2 2 válido para − 1 ≤ x ≤ 1 . b. M esta en la mediana AA’ y no es el centro de la elipse (punto medio del segmento AG). f (−1) ≤ 1. y resolverla siempre que sea posible. entonces B no vigila a A..En Port Aventura hay 16 agentes secretos. Se sabe que si el agente A vigila al agente B..Discutir la existencia de soluciones de la ecuación x2 − p + 2 x2 − 1 = x según los valores del parámetro real p... no existe solución. Demostrar que también se pueden numerar de este modo 11 agentes cualesquiera.... éste vigila al tercero. ( 4 − 3p) 8( 2 − p) como p > 0.. a i + b i + c i = 15. x= Sustituyendo en la ecuación dada se obtiene 4−p 8( 2 − p) ... como 2 x 2 − 1 > 0 . entonces x 2 − p > x . |p| = p. Cada uno de ellos vigila a algunos de sus colegas. Además.. y finalmente: 2 +2 p2 8( 2 − p) = 4−p 8( 2 − p) 4 − 3p + 2 p = 4 − p ⇒ 4 − 3p = 4 − 3p ⇒ 4 − 3p ≥ 0 ⇔ 0 ≤ p ≤ 4 3 5. 10 agentes cualesquiera pueden ser numerados de forma que el primero vigila al segundo. .. Solución: Diremos que los agentes A y B son neutrales si A no vigila a B ni B vigila a A. Por tanto p ≥ 0.2. b i + c i ≤ 8 ∀i = 1.16 Es claro que . de donde x = Como x ∈ R. a i + c i ≤ 8..... ci el número de agentes que son neutrales con Ai. Solución: Si p < 0. Sean: ai el número de agentes que vigilan a Ai. el último (décimo) vigila al primero. así que 2 4−p 8( 2 − p) .. Aislando un radical y elevando al cuadrado dos veces se llega a la ecuación: 8( 2 − p) x 2 ( 4 − p) .An los agentes.. bi el número de agentes que son vigilados por Ai. A2.4. p < 2.. Sean A1. Numeramos los otros 10 espías como C1. Establezcamos primero algunas relaciones conocidas para un pentágono regular de lado 1. (Figura central). entonces no se podrían numerar 10 espías en la forma indicada. lo cual es imposible. Combinando las relaciones anteriores obtenemos ci ≤ 1. Se dobla por las líneas de puntos hasta que coincidan las aristas no punteadas que confluyen en cada vértice. De este modo llegamos a la contradicción de que todos los espías del grupo vigilan a B.. Solución: La figura formada por el agua es un tronco de pirámide pentagonal cuya base menor es el pentágono dado y cuya base mayor es otro pentágono regular que tiene por lado la diagonal del anterior paralela a la arista de la base como se muestra en la figura inferior derecha... Más abajo.. se ha dibujado en forma invertida para una mejor comprensión del dibujo. Por semejanza de los triángulos ABE y PCD tenemos: 1 d 1+ 5 = ⇔ d2 − d −1 = 0 ⇒ d = = ϕ (1) d −1 1 2 A E h 72º O P D C R B B C R-r C H r A H-h 1 A 1 36º B . B no puede vigilar a C2. Entonces si C1 vigila a B. etc . (Figura de la izquierda).. Supongamos que ninguno de los Ci sea neutral respecto de B.C10 de modo que C1 vigila C2. C2. .. Por tanto para cualquier espía el número de sus colegas neutrales es 0 ó 1.. pues en tal caso C1.. luego C2 vigila B .. C2.La figura de la izquierda se compone de seis pentágonos regulares de lado 1m.. C10 vigila a C1..C10 formaría un grupo en las condiciones del problema.Notemos que si una cualquiera de las dos últimas desigualdades no se verificase. . ¿Qué volumen de agua cabe en el recipiente formado?. Sea B uno cualquiera de los espías de este grupo. Supongamos que hubiera un grupo de 11 espías que NO se pudiera numerar en la forma descrita. Por tanto cada uno de los 11 espías debe tener uno y solo uno del grupo neutral con él. B.. 6. Llamemos d a la diagonal. Razonemos por reducción al absurdo. sabemos que V = ϕ3v . tenemos: (ϕ − 1) 2 4 − ϕ2 ϕ −1 r ( H − h) r 1 − ( R − r ) 2 1 − r 2 (ϕ − 1) 2 H h H−h = = ⇒h= = = = R r R−r R−r r (ϕ − 1) ϕ −1 Como ϕ verifica la ecuación (1): ϕ2 = ϕ +1. queda: Vt = ϕ (ϕ 3 − 1) 15 5 ϕ (ϕ 2 + ϕ + 1) 5 ϕ (ϕ + 1) 5 2ϕ + 1 = = = 3 4 4 − ϕ 2 (ϕ − 1) 4 − ϕ 2 12 6 3− ϕ 6 3− ϕ 4 − ϕ2 y sustituyendo el valor de ϕ de (1). h. tenemos para la expresión de h: (ϕ − 1) 2 4 − ϕ2 ϕ −1 = 4 − ϕ 2 − ϕ 2 + 2ϕ − 1 1− 1− h= (ϕ − 1) 4−ϕ 2 = 3 − 2 ϕ − 2 + 2ϕ (ϕ − 1) 4−ϕ 2 = 1 (ϕ − 1) 4 − ϕ2 Sustituyendo las expresiones de a y h y poniendo ϕ3 -1= (ϕ-1)(ϕ2 + ϕ + 1). sustituir y operar: El área a la calculamos sumado 5 triángulos isósceles de lados iguales r.ϕ es el llamado número áureo y representa la relación entre la diagonal y el lado de un pentágono regular. v al de la pequeña. queda finalmente: Vt = 5 2 + 5 15 + 7 5 = ≅ 2. 2 2 2 4 Para calcular h. (hemos usado 2rsen36º = 1 de (2)). En nuestro caso es la relación de semejanza entre las bases del tronco de pirámide. r formando 72º 5 5 5 5 a = r 2 sen 72º = r 2 2 sen 36º cos 36º = r cos 36º = rϕ . y para el volumen del tronco de cono Vt queda: 1 Vt = V − v = ϕ 3 v − v = v(ϕ 3 − 1) = ah(ϕ 3 − 1) . Sólo nos 3 queda calcular a. por la semejanza de los triángulos de la figura central. 2 2 4 2r 2 sen 36º 4 − ϕ2 Llamando V al volumen de la pirámide grande . d ϕ 1+ 5 1 1 1 Además : cos 36º = = = y para el radio r : sen 36º = ⇔r= = ( 2) . siendo a el área del pentágono de lado 1.554m 3 3 5− 5 12 . . El coche comienza con el depósito vacío. siempre se puede elegir un punto de comienzo que le permita completar la vuelta.Calcular la suma de los cuadrados de los cien primeros términos de una progresión aritmética.Demostrar que en un cuadrilátero convexo de área unidad. ¿Cuál es el mayor número de puntos de A que es posible elegir de manera que TRES cualesquiera de ellos NO sean vértices de un triángulo rectángulo isósceles?.. . 5.OLIMPIADA MATEMÁTICA ESPAÑOLA XXXIII Olimpiada Matemática Española Fase nacional 1997 (Valencia) Primera sesión 1. que son vértices de los cuadrados unidad.Se consideran las parábolas y = x2 + px + q que cortan a los ejes de coordenadas en tres puntos distintos por los que se traza una circunferencia. Determinar todos los enteros tales que es natural. Segunda sesión 4. 6. Demostrar que todas las circunferencias trazadas al variar p y q en R pasan por un punto fijo que se determinará. la suma de las longitudes de todos los lados y diagonales no es menor que .Sea p un número primo. Sea A el conjunto de los 16 puntos interiores. pero que no están en los lados del cuadrado inicial... 3. 2. Entre todos los depósitos contienen la cantidad exacta que el coche necesita para dar una vuelta. posición y cantidad de combustible de cada depósito. y que la suma de los términos de lugar par vale +1. sabiendo que la suma de ellos vale -1. b) El tamaño del depósito es suficiente para albergar toda la gasolina necesaria para dar una vuelta.Un coche tiene que dar una vuelta a un circuito circular. Notas: a) El consumo es uniforme y proporcional a la distancia recorrida. Demostrar que con independencia del número.Un cuadrado de lado 5 se divide en 25 cuadrados unidad por rectas paralelas a los lados.. En el circuito hay n depósitos con cierta cantidad de gasolina.. Notas: a) El consumo es uniforme y proporcional a la distancia recorrida.. . Entre todos los depósitos contienen la cantidad exacta que el coche necesita para dar una vuelta. 2.Calcular la suma de los cuadrados de los cien primeros términos de una progresión aritmética. que son vértices de los cuadrados unidad. la suma de las longitudes de todos los lados y diagonales no es menor que 2 2 + 2 .Demostrar que en un cuadrilátero convexo de área unidad. ( ) 6. 3. Determinar todos los enteros k∈Z tales que k 2 − kp es natural. sabiendo que la suma de ellos vale -1.XXXIII Olimpiada Matemática Española Fase Nacional Valencia..Se consideran las parábolas y = x2 + px + q que cortan a los ejes de coordenadas en tres puntos distintos por los que se traza una circunferencia. siempre se puede elegir un punto de comienzo que le permita completar la vuelta. Segunda Sesión 4. Sea A el conjunto de los 16 puntos interiores. pero que no están en los lados del cuadrado inicial. Demostrar que todas las circunferencias trazadas al variar p y q en R pasan por un punto fijo que se determinará.Un cuadrado de lado 5 se divide en 25 cuadrados unidad por rectas paralelas a los lados. Demostrar que con independencia del número. En el circuito hay n depósitos con cierta cantidad de gasolina.Sea p un número primo. El coche comienza con el depósito vacío.. b) El tamaño del depósito es suficiente para albergar toda la gasolina necesaria para dar una vuelta. posición y cantidad de combustible de cada depósito.Un coche tiene que dar una vuelta a un circuito circular.. 5. ¿Cuál es el mayor número de puntos de A que es posible elegir de manera que TRES cualesquiera de ellos NO sean vértices de un triángulo rectángulo isósceles?... y que la suma de los términos de lugar par vale +1. Marzo 1997 Primera Sesión 1. que son vértices de los cuadrados unidad. 4.+ 99) +d2 (12 + 22 +. 12. El resto es fácil de calcular. a lo sumo 2 de los puntos del conjunto {1. 13}. 15.Calcular la suma de los cuadrados de los cien primeros términos de una progresión aritmética. 6. Supongamos que hubiera un conjunto M de 7 puntos verificando la condición del enunciado. 4. a + 99d.  ( a + a + 99d )50 = −1 Para calcular a y d resolvemos el sistema:  que operado y resuelto sale: ( a + d + a + 99d ) 25 = 1 a = -2.. 8} y {2.Soluciones 1.. {3. por ejemplo {1.. 4. 9. 13.. y que la suma de los términos de lugar par vale +1. 13. 11. a lo sumo figurarán dos de ellos en M. de ello se deduce en M deben figurar al menos tres puntos de {2. 7. entonces tenemos que hallar: S = a2 + (a + d)2 + (a + 2d)2 +. ¿Cuál es el mayor número de puntos de A que es posible elegir de manera que TRES cualesquiera de ellos NO sean vértices de un triángulo rectángulo isósceles?.. sabiendo que la suma de ellos vale -1. 14..Un cuadrado de lado 5 se divide en 25 cuadrados unidad por rectas paralelas a los lados.. 12} forzosamente de uno de ellos habremos de tomar dos puntos y uno de otro. 14. 12. El resultado final es S = 299. 7. 11. 8. . 7. a + d.98 2. 16}. Notemos que si cuatro puntos forman un cuadrado.. 9} y {1. Como {7.. a + 2d. pero que no están en los lados del cuadrado inicial. Si descomponemos este conjunto en dos subconjuntos “cuadrados” y disjuntos : {3.+ 99 = 4950. 12 + 22 +. Los paréntesis son progresiones de primer y segundo orden. Sea la progresión a... 10. {2.. 3. 10. 11}.. d = 0.. 2. 8. 14} forman triángulos rectángulo isósceles. . 15} forman un cuadrado por tanto a lo sumo podremos elegir dos números entre {5. 15} . Además {5. 3.+ 992). Sea A el conjunto de los 16 puntos interiores. luego a los sumo 6 de los puntos elegidos deben estar en M y por tanto al menos un punto de M debe ser del conjunto “interior” de A: {6. Los puntos de los conjuntos {1.06. 12} .. 10. 5} forman cuadrados y su unión forma el “contorno exterior” de A. 6. 1 + 2 + ... 16..+ (a + 99d)2 =100a2 + 2ad (1 + 2 + .. 16} deberán figurar en M. Por la simetría de la figura supongamos que es el 7..98. 9}. Numeremos los puntos como indica la figura 13 9 5 1 14 10 6 2 15 11 7 3 16 12 8 4 Por simple tanteo se obtiene un conjunto de seis puntos verificando la condición del enunciado. 16..+ 992 = 328350. 8.. (1) p La mediatriz de AB es la recta paralela al eje OY de ecuación x = − . 1) . Si los dos puntos se eligen del segundo las ínicas maneras son {2.2n. Se tiene: k 2 − kp = n ⇔ k 2 − pk − n 2 = 0 ⇒ k = p2 + 4n2 = a2 ⇔ p2 = (a+2n)(a-2n). llamésmole a. de nuevo incompatibles con cualquier elección del punto que falta en el primer conjunto. transforma C en U(0. 1) con independencia de p y q como se comprueba por simple sustitución.C. 4. así que los cuatro puntos A. q). B(β.a) (x . Solución: Pongamos p ± p 2 + 4n 2 2 El radicando ha de ser cuadrado perfecto. En resumen el número máximo de elementos es 6. 0). C(0. La ecuación de la 4 2 2 2 p 2 + (1 − q ) p q + 1  + y − =   .. Puesto que la parábola corta al eje de abscisas en dos puntos.  y para el radio  2 2   circunferencia es:  x +  2 r= p 2 + (1 − q ) . 3. 12} y {4.Si tomamos dos puntos del primero las únicas posibilidades son {3.ab) La inversión de polo el origen que transforma A en B. Verificando α + β = . B(b. 0). 8} ambas incompatibles con cualquier elección del punto restante en el segundo conjunto. 11} y {6. 2 Hallando la mediatriz de AC. se podrá escribir en la forma: y = (x . Determinar todos los enteros k∈Z tales que k 2 − kp es natural. 2ª Solución (geométrica). Como p es primo y a + 2n ≥ a . sólo hay dos posibilidades: 1) a + 2n = p2 y a . 0).Sea p un número primo. que una vez operada queda:  2 2  4 x 2 + y 2 + px − (1 + q ) y + q = 0 que se verifica para el punto (0.p.B. Los tres puntos que definen la circunferencia son A(α. Demostrar que todas las circunferencias trazadas al variar p y q en R pasan por un punto fijo que se determinará. C(0.Se consideran las parábolas y = x2 + px + q que cortan a los ejes de coordenadas en tres puntos distintos por los que se traza una circunferencia. 1ª Solución (analítica) Sean α y β las raíces . 10}. Claramente el punto fijo se puede obtener a partir de tres circunferencias concretas.b) y los puntos de intersección son A(a. cortando con la anterior y teniendo en cuenta (1) se obtiene para el  p q + 1 centro las coordenadas  − .2n =1 2) a + 2n = p y a . (1) .2n = p . y αβ = q. 0)..U son concíclicos y todas las circunferencias pasan por el punto fijo U. 1ª a d q b c p Sea el cuadrilátero de lados a.(b + d))2 + 4 (a + c)(b + d) ≥ 4 (a + c)(b + d) ≥ 16.En el caso 1) a = p2 + 1 p2 − 1 . de todos los paralelogramos de área 2. b. Consiste en establecer directamente las desigualdades (1) y (2). Para los lados. el cuadrado tiene perímetro mínimo que vale 4 2 . tenemos: ab cd 1≤ + 2 2 Descomponiendo ahora en dos triángulos mediante la diagonal p resulta: 1≤ bc da + 2 2 y de ambas desigualdades se obtiene: ab + bc + cd + da ≥ 4. Por el teorema isoperimétrico. lo que exige p ≠ 2 (n natural). Solución 2ª (Sin usar la propiedad isoperimétrica). c. de donde p + q ≥ 8 = 2 2 (1)..q)2 + 4pq ≥ 4pq ≥ 8. luego 2( p + q ) ≥ 4 2 ⇔ p + q ≥ 2 2 (1) En cuanto al los lados por el mismo teorema para una cuadrado de área 1 el perímetro es 4 luego: a + b + c + d ≥ 4 (2) Sumando(1) y (2) se obtiene el resultado. k 3 = p.Demostrar que en un cuadrilátero convexo de área unidad. k 2 = −  . Pero:(a + b + c + d)2 = ((a + c) . la suma de las longitudes de todos los lados y diagonales no es menor que 2 2 + 2 . Trazando la paralelas por cada vértice a la diagonal que no pasa por él se forma un paralelogramo de área 2 y lado p y q. entonces k = 2 o k = 0 Su p ≠ 2 entonces quedan los cuatro valores: 2 2  p + 1  p − 1 k1 =   . de donde a + b + c + d ≥ 4 (2) . d y diagonales p y q. Sustituyendo los valores de a en (1) y operando queda: Si p = 2 . si descomponemos el cuadrilátero en dos triángulos mediante la diagonal q. Si α es el ángulo que forman las diagonales. 2 4 En el caso 2) resulta a = p . n = 0. tenemos: pq pq 1= sen α ≤ ⇔ pq ≥ 2 2 2 pero (p + q)2 = (p . ( ) Solución. n= . k 4 = 0  2   2  5. d2. d2 = a2 . El coche comienza con el depósito vacío. si hay varios depósitos consecutivos positivos o negativos. Los tramos bajo el eje 1 6 1 2 3 4 5 7 2 representan las situaciones imposibles. Hagamos el gráfico del consumo comenzando en un punto de aprovisionamiento cualquiera... Basta comenzar en ese punto para asegurar que el recorrido es posible. Concursante) Sean c1. b2. La pendiente de los tramos inclinados vale : − ∑c ∑d i i .. a2.. gráficamente equivale a trasladar el eje OX en sentido vertical hasta el punto más bajo con lo que aseguramos que ninguna zona queda bajo el eje. Miembro del Jurado). En efecto... Solución 1 (Sergi Elizalde .. posición y cantidad de combustible de cada depósito... b1. . Demostrar que con independencia del número. Llamamos: a1.. Solución 2 (Mª A. La hipótesis de que el total de combustible es la cantidad exacta para dar la vuelta se traduce en que la gráfica comienza y termina en el eje OX. Por otra parte..b2 .. el problema se reduce a .cn las cantidades de combustible en cada uno de los n depósitos y sean d1. siempre se puede elegir un punto de comienzo que le permita completar la vuelta. b) El tamaño del depósito es suficiente para albergar toda la gasolina necesaria para dar una vuelta.bn Diremos que un depósito es positivo o negativo según lo sea di . Entre todos los depósitos contienen la cantidad exacta que el coche necesita para dar una vuelta.. en la figura el punto 3... La función resultante (trazo continuo) tiene un mínimo. d1 = a1 -b1 .. Por ello podemos suponer sin pérdida de generalidad que di ≠ 0 para todo i. .. . López Chamorro. la ubicación del depósito i no influye en la ordenación del recorrido. Así. La nueva gráfica puede trazarse a partir del punto 3 siguiendo el mismo trazado hacia la derecha y trasladando la parte anterior (tramos 1-2 y 2-3) al punto final de la gráfica anterior (de puntos en la figura). En el circuito hay n depósitos con cierta cantidad de gasolina. el tramo limitado por ellos se puede considerar como un único tramo positivo o negativo.bn a la cantidad de gasolina necesaria para ir del depósito ai al siguiente. Notemos que los tramos inclinado tienen todos la misma pendiente. Si di = 0 . c2..Un coche tiene que dar una vuelta a un circuito circular. Se numeran los depósitos de 1 a n comenzando por uno cualquiera en sentido antihorario. Notas: a) El consumo es uniforme y proporcional a la distancia recorrida.....dn las distancias a recorrer desde cada depósito hasta el siguiente. 6.an a la cantidad de gasolina de cada depósito..Basta sumar (1) y (2) para obtener lo pedido. dn = an . Agrupando los tramos por parejas. uno con más gasolina que otro. Como n < 2k . Así reducimos el caso a un número de depósitos n1 < n/2. en cuyo caso empezando por el que tenga más combustible se puede completar el circuito. El caso de un sólo depósito es trivial. Se empieza y termina en ese único depósito.tener un número par de depósitos alternativamente positivos o negativos. a lo sumo en k . éstas resultarán positivas o negativas y volvemos a repetir el proceso.1 etapas llegaremos a tener 2 depósitos. . . gira un ángulo a en torno a O. 3.. escritos en base 10. que sean iguales al cubo de la suma de sus cifras...Un cuadrado ABCD de centro O y lado 1.Hallar todas las funciones estrictamente crecientes y tales que: f(n + f(n)) = 2 f(n) para n = 1. Hallar el área común a ambos cuadrados. demostrar que: Segunda sesión 4.Hallar todos los números naturales de 4 cifras. 5. paralelas a las tangentes en C y en B a la circunferencia circunscrita. Si D y E son puntos sobre el lado BC tales que AD y AE son. 3..OLIMPIADA MATEMÁTICA ESPAÑOLA XXXIV Olimpiada Matemática Española Fase nacional 1998 (Tarazona) Primera sesión 1.Determina los valores de n para los que es posible construir un cuadrado de n x n ensamblando piezas del tipo: . ... 6..Hallar las tangentes de los ángulos de un triángulo sabiendo que son números enteros positivos.Se considera el triángulo ABC y su circunferencia circunscrita. 2. respectivamente. 2. además: BM + MP + PA = 1 ⇔ b cos α + b + b sen α = 1. tenemos: x + y = 1− x + y 2 2 A' B B' O M P A D' C D C' (1) relación que elevada al cuadrado y simplificada queda: 2 xy = 1 − 2 x 2 + y 2 ( 2) pero x = x 2 + y 2 cos α. la altura es constante (de trazos en la figura) y vale ½. PA’ = b sen α . el área de cuatro triángulos vale 2xy.Un cuadrado ABCD de centro O y lado 1. de donde b= y el área pedida es: 1 sen α + cos α + 1 A' B B' O M P A D' C C' D . gira un ángulo α en torno a O. Solución 1. y sustituyendo en (1) resulta: x 2 + y 2 (1 + cos α + sen α ) = 1 ⇔ x 2 + y 2 = 1 1 + sen α + cos α sustituyendo en (2) y operando obtenemos: 2 xy = 1 − 2 sen α + cos α − 1 . En el triángulo PA’M se tiene: MA’ = b cos α.Olimpiada Española 1998 1.. Tomando como base b = MP. Como el lado B’A’ vale 1. El área pedida consta de 8 triángulos como el sombreado en la figura OPM. ya que la función es periódica con periodo de un cuarto de vuelta. y = x 2 + y 2 sen α . pero BM = MA’ y PA = PA’. El área pedida S(α) sale restando del área del cuadrado cuatro triángulos como el PA’M. Hallar el área común a ambos cuadrados. Llamando x al cateto PA’ e y al cateto A’M. = 1 + sen α + cos α sen α + cos α + 1 Finalmente para el área pedida obtenemos: S( α ) = 1 − sen α + cos α − 1 2 = con 0 ≤ α ≤ 90º sen α + cos α + 1 sen α + cos α + 1 Solución 2. Por la simetría bastará considerar 0 < α < 90º. y = 8. 17·18·19 y 18·19·20 sustituimos en (3) y analizamos cada caso. valores que llevados a (2) con s = 18 se obtiene t = 2 y finalmente n = 5832 3º 999x + 99y + 9z = 18·19·20 ⇔ 111x + 11y + z = 760 resulta x = 6. valores que llevados a (2) con s = 17 se obtiene t = 3 y finalmente n = 4913 2º 999x + 99y + 9z = 17·18·19 ⇔ 111x + 11y + z = 646 de donde x = 5.s que son múltiplos de 9: 16·17·18.s = (s .. escritos en base 10. resulta 11 ≤ s ≤ 21 Si n = xyzt. valores que llevados a (2) con s = 19 resulta una contradicción. Solución: Sea n un número verificando el enunciado. Como 1000 ≤ n ≤ 9999 y n = s3 . habida cuenta de que s3 .1) s (s + 1) y por (1). y s la suma de sus cifras. z = 1. Resumiendo. y = 8. z = 3. y = 9. 1º 999x + 99y + 9z = 16·17·18 ⇔ 111x + 11y + z = 544 resulta inmediatamente x = 4. sólo hay tres valores de s3 .Hallar todos los números naturales de 4 cifras. las únicas soluciones son .s (3) (2) (1) cuyo segundo miembro ha de ser múltiplo de 9 (por serlo el primero) y. que sean iguales al cubo de la suma de sus cifras.S(α ) = 8 11 1 2 = con 0 ≤ α ≤ 90º 2 2 sen α + cos α + 1 sen α + cos α + 1 2. z = 6. tenemos: 1000x + 100y + 10z + t = s3 x+y+z+t=s restando queda: 999x + 99y + 9z = s3 . 4913 y 5832 . .Se considera el triángulo ABC y su circunferencia circunscrita. paralelas a las tangentes en C y en B a la circunferencia circunscrita. Estableciendo la proporcionalidad entre sus lados. .3. Si D y E son puntos sobre el lado BC tales que AD y AE son. B Estableciendo la proporcionalidad entre sus lados. resulta: BE AB 2 = ⇔ BE ⋅ BC = AB AB BC ( 2) M Dividiendo las igualdades (1) y (2) se obtiene el resultado. respectivamente. resulta: CD AC 2 = ⇔ CD ⋅ BC = AC AC BC A 2 D E C (1) De modo análogo los triángulos ∆ABC y ∆ABE son semejantes pues: ∠AEB = ∠EBM = ∠BAC y el ángulo ∠ABE es común. demostrar que: BE AB = CD AC 2 Solución: Los triángulos ∆ABC y ∆ADC son semejantes pues tienen los tres ángulos iguales ya que: ∠ADC = ∠BCM = ∠BAC (la primera igualdad por ser AC y CM paralelas y la segunda por ser ∠BCM ángulo semiinscrito) y el ángulo ∠ACD es común. γ los tres ángulos y supongamos α ≤ β ≤ γ ..4. 3 π π . tendría que ser α < y 2 4 3π . Si tg α > 1. por ser enteros positivos. Sean α. Existe una visualización “sin palabras” de la solución: arc tg 1 + arc tg 2 + arc tg 3 = π. β. β α γ . Solución. se sigue tg β = 2 y tg γ = 3. imposible porque α + β + γ = π.Hallar las tangentes de los ángulos de un triángulo sabiendo que son números enteros positivos. Si fuera γ ≥ entonces tg α no es entero. con lo que: 4 tg( β + γ ) = −1 = relación que operada se convierte en: tg β + tg γ 1 − tg β tg γ (tg β -1)(tg γ -1) =2 de donde. entonces α ≥ arc tg 2 > arc tg Por tanto tg α = 1 y β + γ = 3= π . 3. f(1 + b) = 2b. y resulta que f(1). por tanto han de ser consecutivos. f(n) = n . f(2) = 1 + b. Solución: Supongamos f(1) = b. el primero vale b y el último 2b. si f(n) = c .5. . f(2).f(1+ b) son b + 1 naturales. f(3) = 2 + b.. f(n + c) son consecutivos.< f(1+b) = 2b = b + b. f(1 + b) = b + b. para n > 1.…< f(n + c) = c + c y los números f(n). En general..Hallar todas las funciones f : N → N estrictamente crecientes y tales que: f(n + f(n)) = 2 f(n) para n = 1.…….. Así pues. como f es estrictamente creciente.1 + f(1) . resulta entonces: f(1) = b. Entonces. 2. …. f(n + c) = 2c = c + c y resulta que: c = f(n) < f(n + 1) <. f(n + 1).. distintos.….. se tiene: b = f(1) < f(1 +1 ) < …. se tiene: 3a + b = 3b + a ⇔ a = b. Si pintamos cada cuadradito alternativamente de blanco y negro como en un tablero de ajedrez. Supongamos que fuera posible.Determina los valores de n para los que es posible construir un cuadrado de n × n ensamblando piezas del tipo: Solución: Evidentemente n2 debe ser múltiplo de 4 y.. Si n = 4k podemos dividir cualquier cuadrado n × n en k2 sub-cuadrados del tipo 4 × 4 cada uno de los cuales lo podemos rellenar en la forma señalada en la figura de la izquierda. Tenemos: ( 4 k + 2) 2 2 a+b= = ( 2 k + 1) = 4 k 2 + 4 k + 1 4 luego a + b ha de ser impar. Por otra parte. Veamos que en ese caso la repuesta es negativa. hay dos posibilidades para cada pieza: B B N B N N B N Sea a el número de piezas del tipo de las de la izquierda y b el número de piezas del tipo de las de la derecha. como hay tantas casillas blancas como negras.6. de donde a + b = 2a ha de ser par en contradicción con lo anterior. Queda sólo considerar el caso n = 4k + 2. por tanto n necesariamente es par. . …. b y . Se sacan al azar (sin reposición) tarjetas de la caja y se anota la suma de los dígitos de cada tarjeta extraída. numeradas del 100 al 999. sólo una ficha.-. Una caja contiene 900 tarjetas.. Sea el radio de la circunferencia inscrita. y negra por el otro. Después de varios movimientos ¿será posible quitar todas las fichas del tablero? Segunda sesión 4. … tal que a12 + a22 +……. Cada ficha es blanca por un lado. Probar que :i) ii) las distancias desde G a los lados a.. La mediana del triángulo ABC correspondiente al vértice C tiene longitud m. Casillas vecinas son las que están unidas por un segmento.El baricentro del triángulo ABC es G. 2. el resto de las fichas tiene la cara blanca hacia arriba. an.OLIMPIADA MATEMÁTICA ESPAÑOLA XXXV Olimpiada Matemática Española Fase nacional 1999 (Granada) Primera sesión 1. que está situada en un vértice. Sobre cada casilla se coloca una ficha. tangentes a la parábola de ecuación y = x2 en los puntos A y B. Determinar el área del triángulo ABC en función de m..Las rectas t y t’.+ an2 es un cuadrado perfecto para todo entero positivo n. Inicialmente. se juega un solitario.Probar que existe una sucesión de enteros positivos a1. tiene la cara negra hacia arriba. Denotamos por c respectivamente.. En cada movimiento se retira sólo una ficha negra del tablero y se da la vuelta a cada una de las fichas que ocupan una casilla vecina.Sobre un tablero en forma de triángulo equilátero como se indica en la figura. a2. ¿Cuál es la menor cantidad de tarjetas que se deben sacar. 3. para garantizar que al menos tres de esas sumas sean iguales? 5. se cortan en el punto C. ¿Cuál es el mínimo número de rectas necesarias para que N>1999? . No hay tres rectas que pasen por un mismo punto.Se divide el plano en un número finito de regiones N mediante tres familias de rectas paralelas.6.. a2 = 4 con 32 + 42 = 52.b 2 a+b  y su intersección C es: C  .XXXV OLIMPÍADA MATEMÁTICA ESPAÑOLA FASE NACIONAL Primera Sesión Granada. tangentes a la parábola de ecuación y = x2 en los puntos A y B. para n = 2 basta tomar a1 = 3. a2. Veamos que podemos encontrar un entero positivo an+1 2 tal que k 2 + a n +1 = p 2 . k 2 = p 2 − a n +1 = ( p + a n +1 )( p − a n −1 ) .Y. … tal que a12 + a22 +……. ab  . Solución: Sean A(a. La mediana del triángulo ∆ ABC correspondiente al vértice C tiene longitud m.  2  a+b La mediana CM está en la recta: x = . paralela al eje 2  a + b a2 + b2  OY. 2 En efecto. b2) . Solución: Lo haremos por inducción sobre n. m = 2 Probar que existe una sucesión de enteros positivos a1. 2 Y t t' M A B X t': y = 2bx . Las coordenadas de M son:   2 . B(b. Las rectas t y t’. Supongamos que a12 + a22 +……. b = p − a n +1 . . Las ecuaciones de t y t' son: t: y = 2ax .+ an2 es un cuadrado perfecto para todo entero positivo n. se cortan en el punto C. Pongamos a = p + a n +1 . Determinar el área del triángulo ∆ ABC en función de m. a2) . 12 de Marzo de 1999 Problema 1. an.….    C Tenemos: m = CM = 2 2 Poniendo [ XYZ ] para denotar el área del triángulo de vértices X. 2 .Z queda finalmente: (a − b )2 y si h es la altura del triángulo ∆ BMC resulta: h = b − a = m3 2 m 2 [ABC ] = 2[BMC ] = 2 1 m 2 Problema 2.a .+ an2 = k2 . Después de varios movimientos ¿será posible quitar todas las fichas del tablero? Solución: En el tablero. Por tanto el número pedido es 27 + 25 + 1 = 53. Sobre un tablero en forma de triángulo equilátero como se indica en la figura. hay casillas de tres tipos : vértice. Cada una de ellas tiene. Distinguiremos dos casos 1. sólo una ficha. En cada movimiento se retira sólo una ficha negra del tablero y se da la vuelta a cada una de las fichas que ocupan una casilla vecina. lado. . b = 1 queda: p = m +1 = En ambos casos hemos encontrado an+1 entero verificando el enunciado. para garantizar que al menos tres de esas sumas sean iguales? Solución: Hay 27 posibles resultados para la suma de dígitos (de 1 a 27). Sobre cada casilla se coloca una ficha. a n +1 = m = 2 2 2. y ninguna ficha tiene un número impar de casillas vecinas. Se sacan al azar (sin reposición) tarjetas de la caja y se anota la suma de los dígitos de cada tarjeta extraída.. entonces k2 = 2m + 1. Si pudiéramos retirar todas las fichas del tablero. b = 2 queda: p = m +1 = k2 k2 + 1. Esa ficha era inicialmente blanca.a y b son impares. habría un momento en que quedaría sobre él una única ficha negra. Casillas vecinas son las que están unidas por un segmento. 13 de Marzo de 1999 Problema 3. respectivamente. tiene la cara negra hacia arriba. el resto de las fichas tiene la cara blanca hacia arriba. porque una ficha se vuelve cada vez que se retira una ficha vecina.. Pero esto es imposible. Las sumas 1 y 27 sólo se puede obtener de un modo (100 y 999) En el caso más desfavorable al sacar 52 (27 + 25) tarjetas todas repetirán suma dos veces y en la siguiente (extracción 53) una de ellas aparecerá por tercera vez.a y b son pares. entonces k2 = 4m . a n +1 = ⋅ . k2 = . ¿Cuál es la menor cantidad de tarjetas que se deben sacar. se juega un solitario. Inicialmente. Una caja contiene 900 tarjetas. 2 2 2 2 La última expresión exige que a y b son de la misma paridad. dos. a n +1 = m − 1 = −1 4 4 k 2 −1 k 2 −1 + 1. Tomando a = 2m +1. Tomado a = 2m. Cada ficha es blanca por un lado. cuatro o seis casillas vecinas. Segunda Sesión Granada. Problema 4. y negra por el otro. o interiores. luego ha tenido que cambiar de color un número impar de veces. numeradas del 100 al 999.Tenemos: p = a+b a −b a+b a−b . que está situada en un vértice. p el semiperímetro y S el área de ABC.Problema 5. resulta : y a a 3 de modo análogo para gb y gc . g b ≥ . gb = . Denotamos por g a . Sea r el radio de la circunferencia inscrita. El baricentro del triángulo ∆ ABC es G. c y altura común r).. gc = (2) 3 a 3 b 3 c a+b+c b+c 2r = 1+ ≥ 2 . g c las distancias desde G a los lados a. Probar que: 2r 2r 2r i) g a ≥ . HA. g b . haciendo los inversos y sumando resulta: 1 1 1 3a 3b 3c 3 + + = + + = g a g b g c r ( a + b + c) r ( a + b + c) r ( a + b + c) r B C Por otra parte sabemos que r·(a + b + c) = 2S finalmente. aplicando la desigualdad entre las medias aritmética y armónica: ga + g b + gc g + gb + gc 3 3 ≥ = =r⇔ a ≥3 1 1 1 3 3 r + + ga g b gc r Nota. g c ≥ 3 3 3 ga + gb + gc ≥3 ii) r Solución 1 (del autor del problema): A c gc G b gb i) Es sabido que uniendo G con cada vértice.Sumando las tres desigualdades de a) sólo obtenemos Solución 2 (de Ramón José que mereció mención especial) i) Consideremos los puntos MA. GA como indica la figura. y despejando queda: r a+b+c r a+b+c r a+b+c . de donde g a ≥ y por la desigualdad triangular (b + c ≥ a) . Por tanto. se forman tres triángulos BGC de base a y altura ga . b y c respectivamente. AGC de base b y altura gb y AGB de base c y altura gc de la misma área. Pondremos hA a la altura correspondiente a A. llamando S al área de ABC: a·ga = b·gb = c·gc = 2S 3 (1) ga a (basta unir el incentro con los tres vértices y quedan tres triángulos de bases a. ga + g b + gc ≥2 r . Sustituyendo 2S en (1). ga = b) De (2) . b. como S = pr resulta g A ≥ ii) Usaremos la desigualdad x + siempre x positivo). ¿Cuál es el mínimo número de rectas necesarias para que N>1999? Solución: . ordenando y operando resulta: 1+ a a b b c c 1 1 1  1 1 1  1 1 1 + + + 1 + + + + 1 ≥ 9 ⇔ a  + +  + b + +  + c  + +  ≥ 9 b c a c a b a b c a b c a b c sacando factor común. (Consideraremos x a b b c  c a  + + + + + ≥ 6 b a c b a c Sumando 3. Entonces hA hA = 3 gA (1) Por la desigualdad triangular: G gA B M A GA HA C b + c ≥ a ⇔ 2 p ≥ 2a ⇔ p ≥ a ⇔ a ≤1 p multiplicando por hA y teniendo en cuenta (1) gA ≥ ah A 2S ⇔ gA ≥ 3p 3p queda: 2 r. No hay tres rectas que pasen por un mismo punto. queda: 2p 2p 2p + + ≥ 3 (2) 3a 3b 3c Por otra parte. queda: (g a + g b + g c ) p ≥ 3 S Finalmente usando de nuevo S = pr. 3 g b = hB . 3 Análogamente obtendríamos las correspondientes desigualdades para gB y gC . Tenemos entonces: 1 2 ≥ 2 que se deduce de la obvia ( x − 1) ≥ 0 . Se divide el plano en un número finito de regiones N mediante tres familias de rectas paralelas. resulta ga + gb + gc ≥3 r Problema 6. dividiendo por 3 y poniendo 2p = a + b + c. 3b y 3c y sustituyendo en (2).A Los triángulos AMAHA y GMAGA son semejantes siendo la razón de semejanza 3 (propiedad del baricentro sobre cada mediana). finalmente. resulta 2 S = 3 g a a = 3 g b b = 3 g c c Despejando 3a. 3 g c = hC . como 3 g a = h A . ( ) . n 2 = x 2 + y 2 + z 2 − 2m ≥ 3m ⇔ m ≤ y N = n + m +1 ≤ n + 3 3 n2 Para n = 76. La primera recta de la segunda familia determina en el plano (x +1)·2 regiones. La primera recta de la tercera familia es cortada por las x + y rectas existentes en x + y + 1 partes y cada una de estas partes divide a en dos a cada región existente de modo que el número de regiones se incrementa en x + y + 1 regiones. entonces 2 n2 n2 +1..Supongamos que hay x rectas en la primera familia. Tenemos: 1 2 2 2 m = x 2 + y 2 + z 2 − ( y − z ) + (z − x ) + ( x − y ) ≤ x 2 + y 2 + z 2 .. Así. y = 25. la y -ésima determina (x + 1)(y + 1) regiones.. y en la segunda y z en la tercera. resulta: 3 m = 1925 y N = 2002. Las x rectas de la primera familia determinan x + 1 regiones. Cada recta de la tercera familia aumenta las regiones existentes en la misma cantidad. n 2 + + 1 > 2002 . z = 25. la segunda (x +1)·3.. luego el número total de regiones N vale: N = ( x + 1)( y + 1) + z ( x + y + 1) = x + y + z + xy + xz + yz + 1 = n + m + 1 con n = x + y + z y m = xy + xz + yz . si n = 76 = x + y + z con x = 26. 3. ambas tienen la misma probabilidad. Q(x) = x4 + cx3 + bx2 + ax + 1. Halla la probabilidad de que se crucen.. Segunda sesión 4. que depende sólo de C1 y C2. Ambas parten a la vez siguiendo caminos de longitud mínima con la misma velocidad constante.. es decir.La figura muestra un plano con calles que delimitan 12 manzanas cuadradas.Sean los polinomios: P(x) = x4 + ax3 + bx2 + cx + 1. existe un natural n tal que: . Halla las condiciones que deben cumplir los parámetros reales a. Por B se traza una recta variable que corta de nuevo a C1 y C2 en dos puntos que llamaremos P r y Qr respectivamente.. Una persona P va desde A hasta B y otra Q desde B hasta A..Dos circunferencias secantes C1 y C2 de radios r1 y r2 se cortan en los puntos A y B. Demuestra la siguiente propiedad: Existe un punto M. tal que la mediatriz del segmento PrQr pasa por M. 2.Encuentra el mayor número entero N que cumpla las siguientes condiciones : a) b) tiene sus tres cifras iguales. Q(x) = 0. es suma de números naturales consecutivos comenzando en 1. b y c (a distinto de c) para que P(x) y Q(x) tengan dos raíces comunes y resuelve en ese caso las ecuaciones P(x) = 0.OLIMPIADA MATEMÁTICA ESPAÑOLA XXXVI Olimpiada Matemática Española Fase nacional 2000 (Palma de Mallorca) Primera sesión 1. En cada punto con dos posibles direcciones a tomar. . 5..Nota: es la parte entera de x. .Demuestra que no existe ninguna función que cumpla: f(f(n)) = n + 1. Demuestra que al menos dos de ellos están a distancia menor o igual que 1.Tomemos cuatro puntos situados en el interior o el borde de un cuadrado de lado 1. 6. Q(x) = 0.Olimpiada Española 2000 Problema 1.ax + 1 Q(x) = x4 .2x2 . Halla las condiciones que deben cumplir los parámetros reales a. Sustituyendo estos valores en P(x) y Q(x) obtenemos el sistema: 2 + a + b + c = 0  2 − a + b − c = 0 que nos da las condiciones: b = -2 a = -c Los polinomios quedan en la forma: P(x) = x4 + ax3 .ax .1) = 0 Resolviendo las ecuaciones de segundo grado queda finalmente: Soluciones de P(x) = 0: x = 1.1) (x2 .2x2 + ax + 1 Para resolver las ecuaciones P(x) = 0. b y c (a ≠ c) para que P(x) y Q(x) tengan dos raíces comunes y resuelve en ese caso las ecuaciones P(x) = 0. Solución de Virginia García Madurga de Zaragoza. x = -1. Q(x) = x4 + cx3 + bx2 + ax + 1.1) = 0 Q(x) =(x + 1)(x .Q(x) = 0. 1 y -1. x = − a + a2 + 4 − a − a2 + 4 . x = -1.Q(x) = (a -c) x3 + (c .ax3 . Sean los polinomios: P(x) = x4 + ax3 + bx2 + cx + 1. entre ellas tienen que estar las raíces comunes Como 0 no es raíz ni de P(x) ni de Q(x). separamos por Ruffini las raíces conocida 1 y -1 y quedan las ecuaciones en la forma: P(x) = (x + 1)(x . x= 2 2 [ ] a + a2 + 4 a − a2 + 4 . Las raíces comunes a ambos polinomios serán raíces de la diferencia: P(x) . sacando primero x factor común: x (a − c )x 2 + (c − a ) = 0 Las tres raíces son: 0. Q(x) = 0. x = Soluciones de Q(x) = 0: x = 1.1) (x2 + ax . las dos raíces comunes tiene que ser 1 y -1. x= 2 2 .a) x Resolvemos la ecuación P(x) . 2) con probabilidad . 1 1 1 1 = La probabilidad de que P llegue a (0. 8 2 Sólo se puede cruzar con Q si éste está en (1. Halla la probabilidad de que se crucen. 2) ambos movimientos con probabilidad . 3 1 La probabilidad de que P llegue a (1. 2) y (2. 2). 3) pero Q pasa a (0. Todos los caminos tienen longitud 7. se han marcado en rojo todas las posibles posiciones de P tras el B tercer movimiento y en verde las de Q. 2 111 1 La probabilidad de que se crucen entre (0. Como P y Q recorren caminos de longitud mínima. 2 3311 9 = La probabilidad de cruzarse es 8 8 2 2 28 Caso 3. Caso 1. Definamos un sistema de coordenadas con origen en A y unidad el lado de un cuadrado. 3) es: = 8 8 2 27 Caso 2. 3) o en (2. 2) y (1. P llega a (0. P llega a (2. 2)). 3) lo que sucede 1 1 1 1 también con probabilidad = 2 2 2 8 A P está obligado a pasar a (1. 2). 3) es: 2 2 2 8 Sólo se puede cruzar con Q si éste está en (1. 3 . entonces se cruzarán entre (1. entonces se cruzarán entre (1. 2) es 3  = (hay tres modos de llegar (1. P sólo puede ir a la derecha o arriba y Q a la izquierda o abajo. 3) y Q hacia (1. P llega a (1. 3).Problema 2. 3) con 1 probabilidad . P y Q sólo se podrán encontrar entre el 3º y el 4º movimiento. 2 311 1 3 La probabilidad de cruzarse es = 8 8 2 2 28 3 b) Q llega a (2. Ambas parten a la vez siguiendo caminos de longitud mínima con la misma velocidad constante. Una persona P va desde A hasta B y otra Q desde B hasta A. 2) y Q hacia (1. 3) con probabilidad . 2) ambos movimientos con probabilidad . 3) si P se mueve hacia 8 1 (1. 3) y (1. ambas tienen la misma probabilidad. A Solución de Fernando Cruz Robledillo (Madrid 2). Distingamos ambos casos: 1 a) Q llega a (1. B La figura muestra un plano con calles que delimitan 12 manzanas cuadradas. 1). 2) si P se mueve hacia 8 1 (2. En cada punto con dos posibles direcciones a tomar. Politécnica): Sea O el punto medio del segmento M1M2.γ .180º + 2 (γ + ε) = γ + 2 ε De modo análogo. Por B se traza una recta variable que corta de nuevo a C1 y C2 en dos puntos que llamaremos Pr y Qr respectivamente.(γ + ε) y como el triángulo M2BQr es isósceles. γ = ∠M1BM2. Solución de Luis Emilio García Martínez (Valencia U. es entre (3. 1).180º +2ε -180º + γ = 2ε + γ Resulta que para cualquier posición de la recta variable los triángulos MM1Pr y MM2Qr son iguales y por tanto MPr = MQr y M está en la mediatriz de PrQr. Demuestra la siguiente propiedad: Existe un punto M. resulta: 1 3 9 9 9 3 1 37 + 8 + 8 + 8 + 8 + 8 + 7 = 7 256 2 2 2 2 2 2 2 3 y la de cruzarse 28 Problema 3. Encuentra el mayor número entero N que cumpla las siguientes condiciones : N a) E   tiene sus tres cifras iguales. 0) es 1 . la probabilidad de cruzarse entre los puntos (2.2ε y ∠PrM1M = 360º . 27 La probabilidad pedida es la suma de todos los caso. Problema 4. se tiene: ∠ PrM1B = 180º . 0) y (3.(∠PrM1B + 180º . 0) y (4. Pr M1 M A C2 M2 O ε ε C1 γ B Qr ∠MM2Qr = 180º . Como M no depende de la recta variable queda probada la propiedad del enunciado. Dos circunferencias secantes C1 y C2 de radios r1 y r2 se cortan en los puntos A y B.γ) = 360º . 1) y ( 3.γ + ∠BM2Qr = 180º . 28 9 y 28 Caso 4. P llega a (3.2 ∠M2BQr = -180º + 2 (γ + ε) y por tanto. demostraré que todas las mediatrices de los segmentos PrQr pasan por el simétrico de B respecto de O. 2) es: la de cruzarse entre (2. 1) es 9 . ∠BM2Qr = 180º . Entonces: ∠M2BQr = 180º . por ser el triángulo PrM1B isósceles. Sean ε = ∠PrBM1.Procediendo de modo análogo. 0). que depende sólo de C1 y C2. tal que la mediatriz del segmento PrQr pasa por M. 1) y (2. 3 . La probabilidad de cruzarse entre (3. + (n-1) + n . existe un natural n 3 N tal que E   = 1 + 2 + . De la condición α + β + γ + δ = 360º.. es decir. Supongamos que distribuimos 4 puntos en el cuadrado de manera que cada una de las seis distancias se mayor que 1. Entonces hay dos posibilidades: a) Los cuatro puntos forman un cuadrilátero convexo. digamos por ejemplo α ≥ 90º. δ los ángulos del cuadrilátero convexo. β. se deduce que necesariamente uno de los ángulos ha de ser mayor o igual que 90º. Sabemos que α + β + γ + δ = 360º. Juntando las dos condiciones. + n ⇒ z = ⇒ n 2 + n − 2z = 0 ⇒ n = 2 2 3 (la otra raíz es negativa).. Recuperando la condición a): N N z = E  = 111·6 = 666 ⇒ 667 > > 666 ⇒ 2001 > N > 1998 3 3 Por tanto el mayor N que cumple a) y B es N = 2000 Problema 5.. Tomemos cuatro puntos situados en el interior o el borde de un cuadrado de lado 1. N Condición a): z = E  = 111·k. queda: n= − 1 + 1 + 8·111·k 2 Como n es natural. Tenemos (ver figura): Pi Pj > 1 . 3 Nota: E(x) es la parte entera de x. i ≠ j P2 α δ P3 γ P4 luego . b) Los cuatro puntos forman un cuadrilátero no convexo.N b) E   es suma de números naturales consecutivos comenzando en 1.. γ. Veamos ambos casos: P1 a) sean α. Solución de Roberto Alonso Pérez del País Vasco. ∀k ∈ N. Vamos a demostrarlo por reducción al absurdo. el radicando ha ser cuadrado perfecto lo que ocurre sólo para k = 6 que sustituido en la expresión anterior resulta n = 36. 1 ≤ k ≤ 9 3 n (n + 1) − 1 + 1 + 8z N Condición b): z = E  = 1 + 2 + 3 + . Demuestra que al menos dos de ellos están a distancia menor o igual que 1. Además cualquier pareja de puntos del β interior (o frontera) del cuadrado están a una distancia d ≤ 2 ya que el diámetro de dicho cuadrado es 2 .. Solución de Manuel Pérez Molina del Alicante. . Si su centro es C.1. Solución de Alberto Suárez Real de Oviedo. f(1) = a + 1.. P4 estará en el 2 2 3 3 interior de uno de los tres triángulos que resultan de unir C con cada C vértice y la distancia de P4 a uno de los vértices será menor o igual que el 2 circunradio. Supongamos que f(n .1) = a + n . f (f (0 )) = f (a ) = 1 del mismo modo. 2 < 1.P1P3 = P1P2 + P2 P3 − 2P1P2 ⋅ P2 P3 cos α 2 2 2 como el cuadrilátero es convexo. 2a + n = n + 1 ⇒ a = 1 ∉N 2 hemos llegado a una contradicción y la condición supuesta es falsa con lo que queda demostrado la inexistencia de la función f. cumple: x 1 ≤ Problema 6. 3 Demuestra que no existe ninguna función f : N → N que cumpla: f(f(n)) = n + 1. Supongamos que exista f : N → N | f (f (n )) = n + 1 . uno de ellos. Cada lado de dicho triángulo es menor o igual que 2 (diámetro del cuadrado) y por tanto estará contenido en un triángulo equilátero de lado 32 2 2 . Por último si tres puntos están alineados se reduce al caso b) y si los cuatro puntos están alineados llamando x1.. entonces f( a + n -1) = a + n luego hemos probado por inducción que f ((n )) = f (a + n ) = 2a + n entonces. digamos x1. Supongamos que el punto interior es P4. Por el enunciado: f (f (0 )) = 1. es decir menor que y por tanto menor que 1.. b) Si se forma un cuadrilátero no convexo podemos elegir tres de los cuatro puntos formando un triángulo de modo que el cuarto punto sea interior. y circunradio 2 = < 1 . tenemos: x1 + x 2 + x 3 ≤ 2 y por el principio del palomar.. x2.. .. 3 hemos encontrado un par de puntos a distancia menor o igual que 1. f(2) = a + 2. f(a + 1) = 2. x3 a las distancias entre puntos consecutivos. Se tiene que f(0) = a ∈ N.. 90º ≤ α ≤ 180º y por tanto cos α ≤ 0 y en consecuencia: P1 P3 ≥ P1P2 + P2 P3 > 2 ⇒ P1P3 > 2 2 2 2 lo que es imposible. de modo que el triángulo ABP verifica: AP =BP Sobre cada uno de los otros dos lados de ABC se construyen exteriormente triángulos BQC y CRA. Q.Probar que la gráfica del polinomio P(x) es simétrica respecto del punto A(a.. a3.. Demostrar que al menos uno de esos triángulos tiene todos sus ángulos agudos.. Probar que: . ambos semejantes al triángulo ABP cumpliendo: BQ = QC y CR = RA Probar que los puntos P. C y R o están alineados o son los vértices de un paralelogramo.ABCD es un cuadrilátero inscrito en una circunferencia de radio 1 de modo que AB es un diámetro y el cuadrilátero admite circunferencia inscrita. .Los números enteros desde 1 hasta 9 se distribuyen en las casillas de una tabla 3x3. Segunda sesión 4.Se tienen cinco segmentos de longitudes a1. b) si y sólo si existe un polinomio Q(x) tal que: P(x) = b + (x -a)Q((x-a)2) 2... a4 y a5 tales que con tres cualesquiera de ellos es posible construir un triángulo. a2.OLIMPIADA MATEMÁTICA ESPAÑOLA XXXVII Olimpiada Matemática Española Fase nacional 2001 (Murcia) Primera sesión 1. 3. Después se suman seis números de tres cifras: los tres que se leen en filas de izquierda a derecha y los tres que se leen en columnas de arriba abajo. ¿Hay alguna distribución para la cual el valor de esa suma sea 2001? 5.Sea P un punto en el interior del triángulo ABC. . 6.2..} el conjunto de los números naturales) que cumple.Probar que: .. Calcular el valor máximo de f (n) cuando n£ 2001. n Î N. Hallar el menor número natural n tal que f (n) = 2001. .Determinar la función f : N ® N (siendo N = {1. para cualesquiera s. las siguientes condiciones: f (1) = f (2s) = 1 y si n < 2s.3. entonces f (2s + n) = f (n) + 1.. resulta: ∠QPR = 360º − (180 – 2α) − (A + B) = 180º + 2α . Lo que significa y  2 2  P( a + h ) = b + hQ (h ) que la gráfica de P es simétrica respecto del punto A(a . Sea x − a = h ó x = a + h. + a n h n + a 0 − a1 + a 2 h 2 − .. Sobre cada uno de los otros dos lados de ABC se construyen exteriormente triángulos BCQ y CAR... = a m = 0. m = n − 1 n impar. Por último P( x ) = R( h ) = b + ( x − a )Q (( x − a ) 2 ). c ambos semejantes al triángulo ABP.. para todo h ∈ R.. en el interior del triángulo ABC.Prueba que la gráfica del polinomio P es simétrica respecto del punto A(a . + a n h n . 2.Olimpiada Española 2001 Primera sesión 1. la condición anterior se escribe de la forma: a 0 + a1 h + . para todo x ∈ R. m = n n par. Entonces :  P( a − h ) = b − hQ (h 2 ) P (a − h ) + P( a + h ) = b. Solución: Supongamos primero que exista el polinomio P que cumple las condiciones requeridas.. Q. es decir cuando 180º −∠ACB α= . por tanto PBQ y APR son semejantes (y al ser PB = PA son iguales).. Se deduce que a 2 = a 4 = . R Probar que los puntos P. a 0 = b. ABC es semejante a APR .. La alineación es un caso particular y se producirá cuando ∠ACB + 2α = 180º. P B Solución: b a Los triángulos ABC y PBQ son semejantes pues tienen un ángulo igual ∠ABC = ∠PBQ y los lados que lo forman proporcionales: C c BP = Q a BQ De modo análogo. + ( −1) n a n h n = 2b es decir a 0 + a 2 h 2 + . para cada h ∈ R.Sea P un punto..(180º − ∠ACB ) = 2α + ∠ACB ∠QCR = ∠ACB + 2α ∠PRC = 180º − 2α −∠ARP = 180º ..2α − ∠ACB Las cuatro igualdades establecen que los dos pares de ángulos opuestos del cuadrilátero PQCR son iguales y es un paralelogramo. Para R(h ) = a 0 + a1 h + . Por tanto ahora se tiene que R ( h ) = b + a1 h + a 3 h 3 + .. b) sí y sólo sí existe un polinomio Q tal que: P( x ) = b + ( x − a )Q (( x − a ) 2 ). de modo que A el triángulo ABP es isósceles...2α − ∠ACB ∠PQC =180º − 2α −∠BQP = 180º . para algún polinomio Q. P( x ) = P( a + h ) = R( h ). + a m h m = b. 2 1 . y así existe un polinomio Q tal que R(h ) = b + hQ (h 2 ). P(a − h ) + P(a + h ) = b es equivalente a Sea x = a + h. En particular: ∠ARP = ∠ACB y ∠BQP = ∠ACB Llamando α = ∠BAP = ∠ABP. b). porque P(a − h ) = R( − h ). C y R o están alineados o son los vértices de un paralelogramo. La condición 2 R( −h ) + R (h ) = 2h... resulta 2a 2 ≤ a 2 + a3 < 2a1 a 2 .3. Demuestra que al menos uno de esos triángulos tiene todos sus ángulos agudos. luego a 5 < a12 + a 2 + 2a1 a 2 (4) Sumando las desigualdades (1).. a4 y a5 tales que con tres cualesquiera de ellos es posible construir un triángulo. tendríamos: 2 2 a12 + a 2 ≤ a3 (1) 2 2 a 2 + a3 ≤ a 4 (2) 2 2 2 2 a 3 + a 4 ≤ a 5 (3) Pero (desigualdad triangular): 2 2 a5<a1 + a2. y por tanto a2 < a1.Están dados 5 segmentos de longitudes a1. en contradicción con la ordenación inicial.(3) y (4) tenemos: 2 2 2 2 2 2 2 2 a12 + 2a 2 + 2a3 + a 4 + a5 < a3 + a 4 + a 5 + a12 + a 2 + 2a1 a 2 es decir. 2 . a2.(2). Si ningún triángulo es acutángulo. a3. 2 2 a 2 + a3 < 2a1 a 2 2 2 2 Como a 2 ≤ a3 . Solución: Supongamos que 0 < a1 ≤ a 2 ≤ a3 ≤ a 4 ≤ a5 . .. 2β = ∠AOD. no habrá ninguna distribución para la que la suma indicada tome el valor 2001.. 2γ = ∠COD.h + i) = 2. queda: 3 . cos γ = 1 − de donde: b = 4 − p 2 cos α − psenα 4 − p2 p2 = . Después se suman seis números de tres cifras: los tres que se leen en filas y los tres que se leen en columnas. 2α = ∠BOD. entonces β = 90 − ( α + β) y además: a = 2 senα. ¿Hay alguna distribución para la cual el valor de esa suma sea 2001? Solución Consideremos la distribución: a b c d e f g h i Resulta: S = abc + def + ghi + adg + beh + cfi = 100 (a + c + f + b + a + d + g) + 10(d + e + f + b + e + h) + (g + h + i + c + f + i) = 200 a + 110b + 101c + 110d + 20e + 11f + 101g + 11h + 2i Módulo 9 tenemos: S = 2(a + b + c+..Segunda sesión 4. b = 2 senβ = 2 cos(α + γ ) = 2 cos α cos γ − 2 senα senγ A β O α B Vamos a expresar la condición (1) en función del ángulo α y el dato p que determina por completo el cuadrilátero. p = CD.Los números enteros desde 1 hasta 9 se distribuyen en las casillas de una tabla 3x3. p = 2 senγ.45 = 0 Como 2001 no es múltiplo de 9. Probar que: CD ≤ 2 5 − 4 D p Solución: C Sea O el centro de la semicircunferencia.ABCD es un cuadrilátero inscrito en una circunferencia de radio 1 de modo que AB es un diámetro y el cuadrilátero admite circunferencia inscrita. 4 2 sustituyendo en (1). b a La condición necesaria y suficiente para que 2γ ABCD admita una circunferencia inscrita es: p + 2 = a + b (1) Como 2α + 2β + 2γ = 180º.. 5. Pongamos a = BC. b = AD. p + 2 = 2 senα + 4 − p 2 cos α − psenα o lo que es lo mismo: 4 − p 2 cos α + (2 − p )senα = p + 2 (2) Por tanto, existirá circunferencia inscrita para los valores de p que hagan compatible la ecuación (2) en la incógnita α. Puede expresarse el seno en función del coseno y estudiar el discriminante de la ecuación de segundo grado que se obtiene, pero es más rápido interpretar la ecuación (2) como el producto r r escalar de los vectores u (cos α , senα ) de módulo 1 y v 4 − p 2 , 2 − p . La condición (2) queda: r v cos δ = p + 2 (3) r r siendo δ el ángulo formado por los vectores u y v . r 2 Para que (3) sea compatible debe cumplirse p + 2 ≤ v = 4 − p 2 + (2 − p ) , elevando al cuadrado y operando queda: ( ) p2 + 8p − 4 ≤ 0 Las raíces de la ecuación son p = ±2 5 − 4 . Como p es positivo la condición final es: 0≤ p≤2 5 −4 6.- Determinar la función f : N → N (siendo N = {1,2,3,...} el conjunto de los números naturales) que cumple, para cualesquiera s, n ∈ N, las dos siguientes condiciones: a) f (1) = 1, f (2s) = 1. b) Si n < 2s, entonces f (2s + n) = f (n) + 1. Calcular el valor máximo de f (n) cuando n ≤ 2001. Hallar el menor número natural n tal que f (n) = 2001. Solución Para cada número natural n definimos f (n) como la suma de las cifras de la expresión de n escrito en base 2. Está claro que esta función f cumple las condiciones a) y b). Además, es la única función que las cumple, porque el valor de f (n) viene determinado por las condiciones a) y b). Probamos esa afirmación por inducción sobre n. Si n = 1 o n = 2s, f (n) = 1. Supongamos n > 1, n ≠ 2s y que es conocido f (m) para todo m < n; se puede escribir n = 2s + m con m < 2s tomando 2s la mayor potencia de 2 que es menor que n; entonces f (n) = f (m) + 1. Ahora, es fácil resolver las dos cuestiones que nos plantean: En el primer caso, se trata de ver cuántos unos puede tener como máximo un número menor o igual que 2001 escrito en base 2. Ese número, escrito en base 2, es, obviamente, 1111111111, que corresponde a n = 1023 = 210 - 1. Es f (n) = 10. En el segundo caso, razonando de manera análoga, se observa que la respuesta es n = 2 2001 -1. 4 XXXVIII Olimpiada Matemática Española Fase nacional 2002 (La Rioja) Primera sesión (5 de abril) 1.- Hallar todos los polinomios P(t) de una variable, que cumplen: P(x2 - y2) = P(x + y)·P(x – y) para todos los números reales x e y. 2.-En un triángulo ABC, A’ es el pie de la altura relativa al vértice A y H el ortocentro. a)Dado un número real positivo k tal que , encontrar la relación entre los ángulos B y C en función de k. b) Si B y C son fijos, hallar el lugar geométrico del vértice A para cada valor de k. 3.-La función g se define sobre los números naturales y satisface las condiciones: ·g(2) = 1 ·g(2n) = g(n) ·g(2n + 1) = g(2n) + 1 Sea n un número natural tal que 1 £ n £ 2002. Calcula el valor máximo M de g(n). Calcula también cuántos valores de n satisfacen g(n) = M. Segunda sesión (6 de abril) 4.-Sea n un número natural y m el que resulta al escribir en orden inverso las cifras de n. Determinar, si 4.-Sea n un número natural y m el que resulta al escribir en orden inverso las cifras de n. Determinar, si existen, los números de tres cifras que cumplen 2m + S = n, siendo S la suma de las cifras de n. 5.-Se consideran 2002 segmentos en el plano tales que la suma de sus longitudes es la unidad. Probar que existe una recta r tal que la suma de las longitudes de las proyecciones de los 2002 segmentos dados sobre r es menor que . 6.-En un polígono regular H de 6n + 1 lados (n entero positivo), R vértices se pintan de rojo y el resto de azul. Demostrar que el número de triángulos isósceles que tienen sus tres vértices del mismo color no depende del modo de distribuir los colores en los vértices de H. de donde P (0) = 1 ó P (0) = 0. Fácilmente se comprueba que Q (u ) satisface la ecuación funcional (**). b) Si B y C son fijos. Sea ahora P (0) = 0. y). tg HBA ' = ctg C = HA ' . resulta B  − . haciendo v = 0 en (*) se deduce que P (0) = P (u ) P (0) para todo u ∈ R. Sea P (0) = 1. A C' H B' a) Tenemos: BA ' = c cos B . v ∈ R. que cumplen: P ( x 2 − y 2 ) = P ( x + y ) ·P ( x − y ) para todos los números reales x e y. Problema 2 En un triángulo ABC. Por tanto P(u ) = u n con n ∈ N .Soluciones Olimpiada Española 2002 Problema 1 Hallar todos los polinomios P(t) de una variable. 0  . También puede hacerse sin el cambio de variable haciendo x = y = 0 se llega a P(0) = ( P(0))2 . siendo Q (u ) un polinomio de grado una unidad inferior al grado de P (u ). Entonces P (u ) = uQ(u ). Poniendo u = v = 0 en (**) se obtiene P(0) = ( P(0)) 2 . para todos u . la condición (1) se escribe:  2  2   a2  y y · = k ⇔ y2 = k  − x2  a a  4  −x +x 2 2 que. 0  y llamando A(x. C  . encontrar la relación entre los ángulos B y HA ' C en función de k. BA ' De donde: k= B A' C AA ' c sen B = ⇔ tg B·tg C = k (1) HA ' c cos B ctg C b) Poniendo a = BC. Recíprocamente se comprueba sin dificultad que P ( x) ≡ 1 y P( x) = x n con n ∈ N satisfacen la ecuación funcional inicial (*). La ecuación funcional dada P( x 2 − y 2 ) = P ( x + y ) P( x − y ) (*) es equivalente a la ecuación funcional P(uv) = P (u ) P(v) (**) con el cambio de variables u = x + y y v = x − y. A’ es el pie de la altura relativa al vértice A y H el ortocentro. AA ' a) Dado un número real positivo k tal que = k . AA ' = c sen B . hallar el lugar geométrico del vértice A para cada valor de k. es decir P (u ) ≡ 1. una vez operada resulta: x2 y2 + 2 = 1 (2) a 2 ka 4 4 . tomando unos ejes con origen en el punto medio de BC y eje OX sobre el lado  a  a  BC. Además está la solución trivial P(x) ≡ 0. Hay cinco soluciones de g(n) = 10: 1023. resulta M = 10. n = ak 2k + ak −1 2k −1 + K + a1 2 + a0 = ak K a1a0( 2) . donde aj = 0 o 1. . semidistancia focal = k − 1 y semieje mayor = n. 1791. elipse con eje mayor sobre OX.ecuación de una elipse en la que distinguimos dos casos: Si k < 1. Para cualquier natural n. Calcula también cuántos valores de n satisfacen g(n) = M. 2 2 H A F A H B F' A' F C B A' C F' Problema 3 La función g se define sobre los números naturales y satisface las condiciones: g(2) = 1 • g(2n) = g(n) • g(2n + 1) = g(2n) + 1 • Sea n un número natural tal que 1 ≤ n ≤ 2002. consideramos su representación binaria. 1919 y 1983. Supuesto cierto para k. elipse con eje mayor sobre OY. Entonces g(n) es el número de unos de n escrito en base 2. 1535. Como 211 = 2048 > 2002 > 1024 = 210. Calcula el valor máximo M de g(n). ( ) ( ( ) ( ) k j =0 g ak K a1a01( 2 ) = g 1 + 2·ak K a1a0( 2) = 1 + ∑ a j ) ( ) k j =0 donde se han aplicado las propiedades de g y la hipótesis inductiva. Probaremos por inducción que g ( n ) = ∑ a j por inducción sobre k: j =0 k Para k = 0 es cierto: g 1( 2 ) = g (1) = 1 . hay dos casos para k + 1: g ak K a1a0 0( 2) = g 2·ak K a1a0( 2) = ∑ a j . semidistancia focal = a a 1 − k y semieje mayor = 2 2 a a Si k >1. Por lo tanto. entonces a = c = 0 y no hay solución por ser números de tres cifras. Problema 5 Se consideran 2002 segmentos en el plano tales que la suma de sus longitudes es la unidad. a partir de uno cualquiera dado. entonces a = 10. π 3 Falta considerar el caso trivial de que todos los segmentos tengan la misma dirección en cuyo caso ni hay polígono pero tomando la recta perpendicular a la dirección común sale d = 0. Si c = 1. Construimos ahora un polígono convexo de 4004 lados “uniendo” los vectores uno a continuación del otro. 200c – 97a es múltiplo de 11. Por la primera congruencia. 42. c + a = 0. 4c + a = 0. Como a + c = 11. m = cba = 100c + 10b + a 2 m + S = n nos da: 200c + 20b + 2a + (a + b + c) = 100a + 10b + c.Problema 4 Sea n un número natural y m el que resulta al escribir en orden inverso las cifras de n. n = abc = c + 10b + 100a. y c + a + b congruente con 0. 17. los números de tres cifras que cumplen 2m + S = n. Si c + a = 11. Consideramos los 2 x 2002 = 4004 vectores así obtenidos y los ordenamos por sus direcciones entre 0 y 2π respecto de un sistema de referencia ortonormal arbitrario. Si c + a = 0. Módulo 9: 2(c + a + b) congruente con 0. 24. 35. o 31 y ser múltiplo de 3. o bien c + a = 11. Por d otra parte la circunferencia de centro O y radio está totalmente contenida en el interior del 2 polígono y entonces su circunferencia es menor que el perímetro del polígono. 2 2 Es decir: dπ < 2 y d < < . a = 3. Luego c = 1 o c = 8. 10. resulta que a + c es congruente con 0 módulo 11. tenemos que 3c debe tomar uno de los valores -11. 200c – 97a es múltiplo de 7. Por lo tanto. es decir. Tomamos entonces uno de los lados más próximos a O. Módulo 11: 2(c + a) es 0. es decir 200c + 11b – 97a = 0. 14. 3 Cada segmento determina dos vectores de igual módulo y sentido opuesto. Trabajando módulo 7: 4c + a es congruente con 0 módulo 7. 3. -4. Pero n = 378 no es solución y no existen números con las condiciones pedidas. La proyección del polígono sobre la recta que contiene a este segmento es d y por tanto la suma de las proyecciones sobre la recta anterior es también d. y como mcd (2. Probar que existe una recta r tal que la suma de las longitudes de las proyecciones de los 2002 segmentos 2 dados sobre r es menor que . entonces b = 7. 21. Claramente el perímetro de este polígono es 2 Además es un polígono centrado y simétrico. . Determinar. imposible. 28. sea d el segmento perpendicular a ese lado y a su opuesto que pasa por el centro O. si existen.11) = 1. siendo S la suma de las cifras de n. Si c = 8. respecto de un punto O (la prueba de esta observación es sencilla y es necesario hacerla). 7. Problema 6 En un polígono regular H de 6n + 1 lados (n entero positivo). cada diagonal y cada lado del mismo pertenece sólo (exactamente) a tres triángulos isósceles distintos (la demostración es sencilla y se debe hacer). respectivamente. AAR. (se deben probar estas dos nuevas relaciones). 2 2 2 2 donde A es el número de vértices azules. A = 6n + 1 − R. Se observa que el resultado es también cierto si el polígono H tiene 6n + 5 lados. uno azul y el otro rojo o los tres rojos y ninguno azul. ARR y RRR el número de triángulos isósceles cuyos vértices son los tres azules. Debido a que el número de lados del polígono H deja de resto uno al dividirse entre seis. Análogamente establecemos: 3 × RA = 2 × AAR + 2 × ARR y 3 × RR = ARR + 3 × RRR. dos azules y uno rojo. Demostrar que el número de triángulos isósceles que tienen sus tres vértices del mismo color no depende del modo de distribuir los colores en los vértices de H. de azul y de rojo o ambos de rojo. AR y RR los números de segmentos que son lados y diagonales cuyos extremos respectivamente están coloreados ambos de azul. r vértices se pintan de rojo y el resto de azul. porque cada diagonal o lado de H pertenece a tres triángulos isósceles y los triángulos isósceles con tres vértices azules tienen tres lados con sus dos extremos azules. Esto completa la prueba. Análogamente denotamos por AAA. Las tres relaciones obtenidas conducen a que: 1 1 1 1 AAA + RRR = RR + AA − × RA = × R × ( R − 1) + × A × ( A − 1) − × R × A. Los triángulos isósceles con dos vértices azules tienen sólo un lado con sus extremos de color azul y los triángulos isósceles con menos de dos vértices azules no tiene ningún lado con los extremos del mismo color azul. Entonces 3 × AA = 3 × AAA + AAR. . Denotamos por AA. Ensartamos 2n bolas blancas y 2n bolas negras formando una cadena abierta.b2 sea también un elemento de M? 3.Las alturas del triángulo ABC se cortan en el punto H. se haga en el orden que se haga.Probar que para cualquier primo p distinto de 2 y 5 existe un múltiplo de p cuyas cifras son todas nueves. Determinar el valor del ángulo BCA Segunda sesión (4 de marzo) 4... siempre es posible cortar un segmento de cadena exactamente con n bolas blancas y n bolas negras. el número 2a .XXXIX Olimpiada Matemática Española Fase nacional 2003 (Canarias) Primera sesión (3 de marzo) 1. 2. Se sabe que AB = CH. Por ejemplo si p = 13. 5. Demuestra que.¿Cuáles son las posibles áreas de un hexágono con todos los ángulos iguales y cuyos lados miden 1. b cualesquiera de M. 4. 5 y 6 en algún orden? 6.-¿Existe algún conjunto finito de números reales M que contenga al menos dos elementos distintos y que cumpla la propiedad de que para dos números a. 3. . Demostrar que tanto x como x2 son irracionales. 999999 = 13·76923 2..-Sea x un número real tal que x3 + 2x2 +10x = 20.. De (1) se desprende que : x ≤ 2x – y2 ⇒ 0 ≤ x – y2 ⇒ y2 ≤ x <y . Problema 3 Las alturas del triángulo ABC se cortan en el punto H. Como M es finito. con x = Mín M e y = Máx M. n tales que am − an ≡ 0 ( p ) . b cualesquiera de M. 90 − α C α α A' B . no existe ningún y ∈ R que pueda ser máximo de M por lo que no estaría acotado y no sería finito. a . A Solución de Ibón Arregui Bilbao del País Vasco. Por ende: ∀p ∃ai tal que p | ai y el enunciado queda probado. + ∞) . Por tanto x > 0 y 0 < x < y. Se sabe que AB = CH. el número 2a . y esto sólo es cierto si y ∈ [1. Considérense en dicho caso los números {a . H C' Ángulo C < 90º. Problema 2 ¿Existe algún conjunto finito de números reales M que contenga al menos dos elementos distintos y que cumpla la propiedad de que para dos números a.Olimpiada Española 2003 Problema 1 Probar que para cualquier primo p distinto de 2 y 5 existe un múltiplo de p cuyas cifras son todas nueves. y ] . que sólo se cumple si y ∈ (0.b2 sea también un elemento de M? Solución de Víctor González Alonso de Burgos. Como (1) y (2) deben cumplirse a la vez. Por tanto al haber p números y sólo p – 1 restos posibles módulo p. Esto contradice que x sea el mínimo de M. 1). 6i8 7 Sea ai el número compuesto por i nueves ai = 99K 9 . necesariamente estará acotado. Suponemos sin pérdida de generalidad que m > n y: m n 6 m 8 6 n 8 6 −8 6 n 8 7 7 7 7 p | am − an = 99K 9 − 99K 9 = 99K 9 00K 0 = am− n ·10n Como p ≠ 2 y p ≠ 5 ⇒ p / 10 n = 2n ·5n ⇒ p | am− n y como am – n pertenece al conjunto escogido por | ser m – n < n y m – n ≥ 1 se ha llegado a una contradicción. Supongamos x ≤ 0: Tenemos x ≤ 0 ⇒ 2x ≤ x ⇒ 2x – k2 < x (k cualquier número de M). Determinar el valor del ángulo ∠BCA. Supongamos que ∃p tal que | p / ai ∀i ∈ para probar por contradicción el enunciado. De (2) obtenemos que: 2y – y2 ≤ y ⇒ y – y2 ≤ 0 ⇒ y ≤ y2. se sabe que existen m. Por ejemplo si p = 13.K a } . En cualquier casi debe ser: (1) x ≤ 2x – y2 ≤ y y además (2) x ≤ 2y – y2 ≤ y . 1 2 p en este conjunto sabemos que no hay ningún ai ≡ 0 ( p ) (por hipótesis) . 999999 = 13·76923 Solución de Luis Hernández Corbato de Madrid. Pongamos M ⊂ [ x. Luego no hay raíces enteras. y basta probar los 8 divisores para comprobar que ninguno verifica la ecuación.Llamaremos A’ al punto en que la altura de A corta al lado BC del triángulo ABC. Problema 5 ¿Cuáles son las posibles áreas de un hexágono con todos los ángulos iguales y cuyos lados miden 1. Demostrar que tanto x como x2 son irracionales. entonces la tangente de C vale –1 y C = 135º. por tanto el valor de tg C = 1. y C = 45º. 5 y 6 en algún orden? Solución: . Finalmente. por tanto el ángulo ∠HCA’ es 90º . Veamos que x no puede ser racional por reducción al absurdo. Otro modo de verlo es comprobar que f(x) = x3 + 2x2 +10x – 20 es estrictamente creciente (su derivada es positiva para todo x) y además f(1) = 13 y f(2) = 36. este valor de C no es válido. Esto puede hacerse teniendo en cuenta que si lo fuese. por tanto el ángulo ∠HAC’ es 90º . El ángulo ∠HAC’ es igual al ángulo ∠A’AB del triángulo A’AB que es rectángulo por tanto el ángulo ∠A’BA es α. 3. El ángulo ∠CHA’ es igual al ángulo ∠AHC’. también los sería x en contra de lo probado.α . De aquí concluimos que los triángulos CHA’ y A’AB son semejantes. x sería entero lo que es imposible. Entonces los triángulos AA’B y A’CH son semejantes y tienen la hipotenusa igual. Supongamos que x = p/q con q ≥ 1 y p/q irreducible. luego son iguales y deducimos AA’ = A’C . Por tanto q = 1. Solución: Primero veamos que x no puede ser entero. Procediendo de modo análogo el ángulo ∠A’CH es igual al ángulo ∠C’CB . 2. si fuese C = 90º. Para la irracionalidad de x2 basta ver que 20 − 2 x 2 . En el triángulo CA’H. x ( x 2 + 10 ) = 20 − 2 x 2 ⇒ x = 2 x + 10 y si x2 fuese racional. y C’ al punto donde la altura de C corta al lado AB del triángulo ABC. En el triángulo C’CB el ángulo ∠CA’H es recto. A α A' C' α α C 90 − α B 90 − α B' Ángulo C > 90º. el ángulo ∠CA’H es recto. Entonces p 3 = 20q 3 − 10q 2 p − 2qp 2 = q ( 20q 2 − 10qp − 2 p 2 ) Si q fuera estrictamente mayor que 1.α y en el triángulo CC’B el ángulo ∠CC’B es recto y por tanto ∠C’BC es 90º . sería un divisor de 20. 4.α. C coincide con H y CH = 0. Luego x es irracional. y como CH = AB . la igualdad anterior estaría en contradicción con la hipótesis de que p/q es irreducible.α . por tanto el ángulo ∠A’HC es 90º . En el triángulo AHC’ el ángulo ∠HC’A es recto. son triángulos iguales de donde obtenemos que AA’ = CA’. El triángulo AA’B es rectángulo en A’ y por ello ∠BAA’ es α. H Problema 4 Sea x un número real tal que x3 + 2x2 +10x = 20. Como AB ≠ 0. las ë û 4 4 67 3 4 65 3 = 10 y el área 4 65 3 =11 y el área 4 67 3 = 12 y el área 4 Problema 6 Ensartamos 2n bolas blancas y 2n bolas formando una cadena abierta. es decir en el otro extremo. 5) y (b. de forma que la diferencia es x – y = 2k para k ∈ {−n. 4. x bolas blancas e y bolas negras con lo que la diferencia (blancas . para el mismo k.d.K n − 1. entonces l Si a + c + e = 4. . Como k sólo puede variar de 1 en 1 tiene que pasar por el cero ya que no se lo puede saltar. por tanto l = 7 + (a + c + e) / 3 El valor más pequeño de a + c + e es 6 y el más grande 15 así que 9 ≤ l ≤ 12 f e e d a Si a + c + e = 6. 2n blancas y 2n negras. Vamos moviéndonos de una en una posición hacia el extremo contrario. 6) a Si a + c + e = 9 el único caso posible es: (a . c. c. n} . c. a + b + c + d + e + f = 21 l=a+b+c=c+d+e=e+f+a 3l =21 + a + c + e. c.c. 6). Cogemos un grupo de un extremos con 2n bolas. d) = (2.q.K . 5. 6) Si a + c + e = 12 el único caso posible es (a . 4. 2. Tenemos la cadena con el total de 4n bolas. disminuye en 1 o no cambia. Es decir que k pasa de una posición a su opuesta con el mismo valor absoluto. e) = (1. 0. c c b Como el área del triángulo de lado l es l 2 áreas posibles son: Si a + c + e = 6. Demuestra que. La diferencia varía en 2 si la bola que se deja y que se coge son de distinto color y no se mantiene si son del mismo color. se haga en el orden que se haga. e) = (2.1 − n. La posición final. tendrá las bolas al revés. 6) Si a + c + e = 15 el único posible es (4. 5.(a + c + e )ú.La idea es prolongar los lados para formar un triángulo equilátero. entonces l = 9 y el área Si a + c + e = 9. es decir k aumente en 1. e) = (1. 3) y (b. en cada movimiento la diferencia varía en 2 o no varía. este grupo tendrá x bolas negras e y bolas blancas. entonces l 3 3 é2 2 2 2 ù y la del hexágono es l ê . entonces l Si a + c + e = 5. siempre es posible cortar un segmento de cadena exactamente con n bolas blancas y n bolas negras. En el momento en que k = 0.negras) será ahora y – x = –2k. 3. entonces son: (a. d) = (4. Solución de Mohammed Blanca Ruiz de Valencia. Siempre se podrá cortar un segmento de longitud 2n con n bolas blancas y n bolas negras. x = y = n.c. . · · 3.¿Existe alguna potencia de 2 que al escribirla en el sistema decimal tenga todos sus dígitos distintos de cero y sea posible reordenar los mismos para formar con ellos otra potencia de 2?.. y enteros se verifica: f(x + f(y)) = f(x) – y Segunda sesión (27 de marzo) 4..Tenemos un conjunto de 221 números reales cuya suma es 110721. 5. OZDT y OTAX. Z y T se forman cuatro cuadriláteros. Probar que la suma de los elementos de las cuatro esquinas vale 2004 2.Se representa por Z el conjunto de todos los enteros. OYCZ. P y Q los puntos medios de las diagonales BD y AC respectivamente.ABCD es un cuadrilátero cualquiera. la mediana desde B sea dividida en tras partes iguales por la circunferencia inscrita en el triángulo. es .. Los disponemos formando una tabla rectangular de modo que todas las filas y la primera y última columnas son progresiones aritméticas de más de un elemento. OXBY. Y.. Probar que los cuatro cuadriláteros tienen la misma área. Las paralelas por P y Q a la otra diagonal se cortan en O. Hallar todas las funciones f: Z → Z tales que para cualesquiera x.XL Olimpiada Matemática Española Fase nacional 2004 (Ciudad Real) Primera sesión (26 de marzo) 1.Demostrar que la condición necesaria y suficiente para que. Justificar la respuesta. Si unimos O con las cuatro puntos medios de los lados X. en el triángulo ABC. la de su derecha y la de su izquierda.6. y dar la vuelta a tres fichas: la elegida. Supongamos que inicialmente hay una sola ficha con la cara negra hacia arriba.Colocamos. formando una circunferencia. repitiendo el movimiento descrito. entre las cuales exactamente una tiene al comienzo la cara negra hacia arriba? . Un movimiento consiste en elegir una ficha con la cara negra hacia arriba. ¿Será posible. 2004 fichas bicolores: blancas por una cara y negras por la otra.. conseguir que todas las fichas tengan la cara blanca hacia arriba? ¿Y si tuviéramos 2003 fichas. . m para el de columnas y S para la suma de los n·m elementos.Olimpiada Española 2004 Problema 1 Tenemos un conjunto de 221 números reales e cuya suma es 110721.. Sn = m a1 + an n ·m 2 y sumando miembro a miembro queda: S = S1 + S 2 + K + S n = m 1 1 1 m m m m m  n·m 1 1 ( a1 + a2 + K + an ) + ( a1 + a2 + K + an )  = 4 ( a1 + an + a1 + an ) 2 1 1 m a1 + an + a1m + an = 4 S 4·110721 = = 2004 n·m 221 ....... resulta: a1 + a m S1 = 1 1 ·m 2 1 a + am S 2 = 2 2 ·m 2 . Los disponemos formando un rectángulo de modo que todas las filas y la primera y última columna son progresiones aritméticas de más de un elemento..... Probar que la suma de los elementos de las cuatro esquinas vale 2004 Solución Denotaremos por aij al elemento de la fila i-ésima y columna j-ésima del rectángulo Pongamos n para el número de filas. Con notación matricial queda: 1  a1  1 a M = 2 L  1 a  n a12 2 a2 L 2 an L L L L a1m  m a2  L  m an   Sumando por filas y llamando Sk a la suma de la fila k........... Y. La quebrada APC divide al cuadrilátero en dos partes de igual área pues AP es la mediana de ABD y PC lo es de CBD. P y Q los puntos medios de las diagonales BD y AC respectivamente. Tenemos ya probado que el área del cuadrilátero TPZD es A la cuarta parte del área del cuadrilátero inicial. P D luego los triángulos TPZ y TOZ tienen la misma área y lo B mismo les ocurre a los cuadriláteros TPZD y TOZD. Si unimos O con las cuatro puntos medios de los lados X. OYCZ. La quebrada TPZ divide al cuadrilátero APCB (sombreado) en dos partes de igual área pues PT es mediana de APD y PZ es mediana de CPD. A X P D Q Y O Z T C Solución 1 (“oficial”). BD Igualmente: XT = ZY = Y 2 Para probar el enunciado bastará probar que: C A X β Q α α O P β T T' Z' Z D . Probar que los cuatro cuadriláteros tienen la misma área. Z y T se forman cuatro cuadriláteros. Bastará probar que el área de cada cuadrilátero es la cuarta parte del área total. Las paralelas por P y Q a la otro diagonal se cortan en O.Problema 2 ABCD es un cuadrilátero cualquiera. OXBY. Del mismo modo se probaría para los otros tres O Q cuadriláteros. Y Z C Solución 2 (de la concursante Elisa García Lorenzo) La fórmula de la superficie del cuadrilátero es: AC ·BD·senα S= 2 AC Además ZT = XY = al ser ZT la paralela 2 media del triángulo ACD y XY la paralela B media del triángulo ABC. T X Finalmente TZ es paralela a OP por serlo ambas a AC. OZDT y B OTAX. pero el área de PXBY (en amarillo en la figura) es la cuarta parte del cuadrilátero inicial al ser semejantes con razón 2 del grande al pequeño. PXBY también tienen la misma área.AC ·BD XT · AO senα = 4 senβ 2 2 AC ·BD·senα = 2 XT · AO·senβ ACsenα = 2 AOsenβ AQsenα = AOsenβ AQ AO = senβ senα que es el teorema del seno en el triángulo AQO. Queda probado el enunciado por extensión de la demostración a los 4 cuadriláteros pequeños que resultan ser una cuarta parte de grande. los triángulos OXY. PXY tienen la misma base e igual altura y por tanto la misma área. C Z D . De ahí que los cuadriláteros OXBY. A X Q B O Y P T Al ser OP paralela a AC. Solución 3 (de Marco Castrillón López). y enteros se verifica: f ( x + f ( y )) = f ( x) − y. Poniendo k = 1 + f (1) ⋅ f (1) = 1 + f (1) 2 > 0. entonces: f ( x + n f ( y )) = f ( x + (n − 1) f ( y ) + f ( y )) = f ( x + (n − 1) f ( y )) − y = = f ( x) − (n − 1) y − y = f ( x) − ny. Para n = 0 es obvio. Hallar todas las funciones f : Z → Z . tales que para cualesquiera x.Problema 3 Se representa por Z el conjunto de todos los enteros. se tiene f ( x ) = f ( x + f (k )) = f ( x) − k . Deducimos que no existen funciones que satisfagan la condición requerida. . y por inducción suponiendo que para cada entero n ≥ 1 f ( x + ( n − 1) f ( y )) = f ( x) − ( n − 1) y. Solución: Primeramente observemos que f ( x + nf ( y )) = f ( x) − ny. Por tanto f (1 + f (1) ⋅ f (1)) = 0. que es una contradicción. Análogamente se prueba para cada entero n ≤ −1. Sin embargo al multiplicar la menor potencia de las dos por 2. y ambas dejan el mismo resto cuando se dividen por 9. Esto último se debe a que el resto de un número al dividirse por 9 es congruente. En primer lugar. 7. Claramente ninguna de las dos potencias es divisible por 3. cambia el resto cuando se divide por 9. 2048. con la suma de sus dígitos. . 4 u 8 (de otra manera no tendrían ambas el mismo número de dígitos). ambos deben tener las mismas cifras lo que implica que el número de cifras sea el mismo.16. No existe ningún número que cumpla las condiciones del enunciado. porque si fuera 16 se alteraría el número de cifras.512.. que haya dos potencias de 2 cuyas expresiones decimales sólo difieran en el orden de colocación de los dígitos. Efectivamente. Si y es múltiplo de 9. 256. las fórmulas serían: Para 4: (z son las llevadas en la segunda duplicación).. 2 ( 2 x − 9 y ) − 9 x = 0 ⇔ 4 x − 18 y − 9 z = 0 ⇔ 4 x = 9 ( 2 y + z ) Tampoco podría ser ya que 2y + z es un natural. Los restos de las sucesivas potencias de 2 al dividirse por 9 forman una sucesión periódica. absurdo. 2  4 x − 9 ( 2 y + z )  − 9a = 0 ⇔ 8 x = 9 ( 4 y + 8 z + a ) . que la suma de sus cifras sea la misma. Como duplicar un número implica que la nueva suma de sus cifras sea el doble de la antigua menos 9x. 4. reordenar los dígitos de una potencia de 2 para obtener otra potencia distinta de 2. Para 8: (a son las llevadas en la tercera duplicación). donde x es el número de llevadas ya que a cada llevada restas 10 a un número y sumas 1 al siguiente.32. por lo que ambas potencias dejan el mismo resto. 7.5.128. Esta sucesión tiene periodo 6. Para que la suma fuera igual tiene que cumplirse 2 y − 9x = 0 siendo y la suma de las cifras antiguas..1024.. y este número es divisible por 9. Para 4 y 8 el proceso es parecido. 2. porque para todo n entero positivo 2n + 6 − 2n = 2n (26 − 20 ) = 63 ⋅ 2n . Entonces y debe ser múltiplo de 9 para que se cumpla la ecuación anterior ya que en caso contrario habría “medias llevadas”. 4.Problema 4 ¿Existe alguna potencia de 2 que al escribirla en el sistema decimal tenga todos sus dígitos distintos de cero y sea posible reordenar los mismos para formar con ellos otra potencia de 2 ?. así que el cociente entre ambos no debe ser mayor que 8.. no habrá ningún número que cumpla esas características.1.8. es decir. los restos de: 2.   Y tampoco podría ser..1. son: 2. pero como estamos tratando potencias de 2. 4 u 8. 4096. No es posible por tanto.. el número también debería ser múltiplo de 9. Solución 2 (del concursante Lander Ramos Garrido). 4.5. Justificar la respuesta Solución 1 (“oficial”): Supongamos que exista tal potencia de 2. según el criterio de divisibilidad. Otra condición que han de cumplir es. módulo 9.8.8. obviamente. Por otra parte la mayor de ambas potencias se obtiene de la menor multiplicando ésta por 2. 64. de donde = c−a 4 a 5 Por lo tanto. BT – CT = c – b.se puede también obtener completando el triángulo ABC hasta obtener un paralelogramo ABCD). M y N. tales que BM = MN = NK = x Sea T el punto de tangencia del círculo inscrito con el lado BC. . evidentemente. es a b c = = 5 10 13 Solución. los puntos B y K están igualmente alejados del centro del círculo inscrito. Entonces resulta 2a 2 + 2c 2 − b 2 = 36 x 2 (1) La potencia del vértice B respecto del círculo inscrito se puede escribir de dos maneras: BT 2 = BM ·BN . con lo cual ( a + c − b) 2 = 8 x 2 (2) Como. resulta BC = KC. C a T I M B c A N b K Las siguientes relaciones se verifican en cualquier triángulo: a + c − b = 2 BT 2a 2 + 2c 2 − b 2 = 4 BK 2 (La primera se deduce sin más de BT + CT = a. la segunda –fórmula de Apolonio o de la mediana. de donde b = 2a Sustituyendo esta última igualdad en (1) y (2). x ≠ 0. a) la condición es necesaria. Sea ABC un triángulo tal que la mediana BK (K punto medio de AC) corte a la circunferencia inscrita en dos puntos. resulta c+a 9 c 13 = .Problema 5 Demostrar que la condición necesaria y suficiente para que. obtenemos c 2 − a 2 = 18 x 2 . la mediana desde B sea dividida en tras partes iguales por la circunferencia inscrita en el triángulo. en el triángulo ABC. en el triángulo del problema. (c − a) 2 = 8x 2 y ya que c − a ≠ 0. BT 2 = 16 = BM ·BN y en el inradio r= C a T I M B H S = 6 14 p b K N c A (calculando S por la fórmula de Herón). HI 2 = . entonces 22 × 47 BI = 4 + r = 14 2 2 2 4 .a b c = = . así que la bisectriz del ángulo C es también altura. Sea H = CI ∩ BK . b = 10. dividida en tres partes iguales por el círculo inscrito. 5 10 13 b) la condición es suficiente. en efecto. luego la mediana BK queda. resulta MN = 2 2 . en BIH. y finalmente en IHM. 7 Como H es el punto medio de MN. consideremos el triángulo rectángulo BIT. No hay pérdida de la generalidad en suponer que a = 5. El triángulo BCK es isósceles. Sustituyendo los valores de los lados en las fórmulas utilizadas en la parte a). por otra parte. resulta BK = 6 2. c = 13. HM 2 = r 2 − HI 2 = 2. Cada movimiento posible cambia el número de fichas negras en un número impar: BNB pasa a NBN : el número de fichas negras aumenta en 1 NNB pasa a BBN : el número de fichas negras disminuye en 1 BNN pasa a NBB: el número de fichas negras disminuye en 1 NNN pasa a BBB: el número de fichas negras disminuye en 3 Como inicialmente hay exactamente una ficha negra. y repitiéndolo sobre las fichas que ocupan los lugares 2 . BBB BB en la que todas las fichas tendrían la cara blanca hacia arriba. el número total de movimientos para tener las 2004 fichas con la cara blanca hacia arriba debe se impar. Por lo tanto. para que al final del proceso siga teniendo la cara blanca hacia arriba. Designamos por xi el número de movimientos realizados eligiendo la ficha i (que debe ser negra). formando una circunferencia.. y 2003 +2 se identifica con 1) El número total de movimientos será: N = ( x1 + x2 + x3 ) + x4 + K + ( x2002 + x2003 + x2004 ) Como 2004 es múltiplo de 3.. .. Por lo tanto. 3k+2... suma de números pares.2002 llegaríamos a la configuración NNN NNN ..... debería ser par: contradicción. pues N es impar. 5 . repitiendo el movimiento descrito.. La ficha que ocupa el lugar i cambia de color en los movimientos en que la elegimos a ella (xi). Un movimiento consiste en elegir una ficha negra. NNN BB Eligiendo ahora las fichas que ocupan los lugares 2.. N es la suma del número de veces que hemos dado la vuelta a las fichas en los lugares 2. todas ellas blancas al principio: así que N. a la de su izquierda (xi-1) o a la de su derecha (xi+1). (única negra al principio).Problema 6 Colocamos... no será posible conseguir que las 2004 fichas tengan la cara blanca hacia arriba.... (xi-1 + xi + xi + 1) es el número de veces que hemos dado la vuelta a la ficha que ocupa el lugar i (2004+1 se identifica con 1.. ¿Será posible.. 3k+2. y dar la vuelta a tres fichas: la elegida. Cada ficha inicialmente blanca debe ser “tocada” un número par de veces.. . 2003. la de su derecha y la de su izquierda.. 5 . 2000 tendríamos: BBB BBB . entre las cuales exactamente una tiene al comienzo la cara negra hacia arriba? Solución Numeremos las fichas desde 1 hasta 2004: la 1 es negra y las restantes son blancas... conseguir que todas las fichas tengan la cara blanca hacia arriba? ¿Y si tuviéramos 2003 fichas. 2004 fichas bicolores: blancas por una cara y negras por la otra. Supongamos que inicialmente hay una sola ficha con la cara negra hacia arriba.2001... Con 2003 fichas si es posible: iniciando el movimiento sobre la ficha 1.. XLI Olimpiada Matemática Española Fase nacional 2005 (Santiago de Compostela) Primera sesión (21 de marzo) 1. Sea ABC. cada uno de distinto color.¿Es posible colorear los puntos del plano cartesiano Oxy de coordenadas enteras con tres colores. Las n – 3 diagonales que salen del vértice A dividen al triángulo ZAB en n – 2 triángulos más pequeños.Sean a y b enteros. c el lado a es la media aritmética de b y c.XYZ un polígono regular de n lados con todos sus lados de longitud 1..Sean números reales cualesquiera.Probar que para todo entero positivo n. 2... Demostrar que la ecuación admite a lo sumo una solución entera. c) La distancia del circuncentro al lado a es R – r .. b. Probar que cada uno de esos triángulos es multiplicativo. b) La altura relativa al lado a es tres veces el inradio r. 3. Probar que: mín 6.. Segunda sesión (22 de marzo) 4. Probar: a) 0º ≤ A ≤ 60º. no estén alineados? Justificar la contestación.En un triángulo de lados a.. 5. . la expresión decimal de es periódica mixta. de tal modo que cada color aparezca infinitas veces en infinitas rectas paralelas al eje Ox y tres puntos cualesquiera.. siendo R el circunradio.Diremos que un triángulo es multiplicativo si el producto de las longitudes de dos de sus lados es igual a la longitud del tercer lado.. Solución.Olimpiada Española 2005 Problema 1 Sean a y b enteros. y ) de rojo si x + y es par. de blanco si x es impar e y es par y de azul si x es par e y es impar. ( p − a ) = −1 ⇒ a = p + 1 = 3   sustituyendo queda la ecuación p − b ) = −1 ⇒ b = p + 1 = 3  (  2 ( x − 3) 3 + 1 = x 3 − 9 x 2 + 27 x − 26 = 0 y después de separar la raíz 2 resulta x 2 − 7 x + 13 = 0 que no tiene raíces.. no estén alineados? Justificar la contestación.. Demostrar que la ecuación ( x − a )( x − b )( x − 3) + 1 = 0 admite a lo sumo una solución entera. cada uno de distinto color. Probemos que tal coloración es posible. Pintemos el punto ( x. ( p − a) = 1 ⇒ a = p −1 = 3   idéntico al anterior. -1- . entonces: ( x − a )( x − b )( x − 3) + 1 se anula para x = p. de tal modo que cada color aparezca infinitas veces en infinitas rectas paralelas al eje Ox y tres puntos cualesquiera. Finalmente. es decir ( p − a )( p − b )( p − 3) = −1 Distingamos varios casos 1. Claramente se satisface la condición de que cada color aparezca infinitas veces en infinitas rectas paralelas al eje OX . ( p − b ) = −1 ⇒ b = p + 1 = 5    2.( p − 3) = 1 ⇒ p = 4 entonces para los otros factores tenemos dos posibilidades: ( p − a ) = −1 ⇒ a = p + 1 = 5   sustituyendo queda la ecuación ( p − b) = 1 ⇒ b = p −1 = 3    ( x − 3) ( x − 5 ) + 1 = x3 − 11x 2 + 39 x − 44 = 0 y una vez separada la raíz 4 resulta la ecuación x 2 − 7 x + 11 = 0 que no tiene raíces enteras. Sea el entero p una raíz.( p − 3) = −1 ⇒ p = 2 entonces para los otros factores tenemos dos posibilidades:  ( p − a ) = 1 ⇒ a = p − 1 = 1 sustituyendo queda la ecuación ( p − b ) = 1 ⇒ b = p − 1 = 1   2 ( x − 1) ( x − 3) + 1 = x3 − 5 x 2 + 7 x − 2 = 0 y después de separar la raíz 2 resulta x 2 − 3 x + 1 = 0 que no tiene raíces enteras. Problema 2 ¿Es posible colorear los puntos del plano cartesiano Oxy de coordenadas enteras con tres colores. Solución oficial. Supongamos ahora que ( x1 , y1 ) sea rojo, ( x2 , y2 ) sea blanco y ( x3 , y3 ) sea azul. Entonces x2 − x1 e y2 − y1 tienen paridad opuesta y x3 − x2 e y3 − y2 son ambos impares. Por tanto: ( y3 − y2 ) ( y2 − y1 ) ( y2 − y1 )( x3 − x2 ) ≠ ( y3 − y2 )( x2 − x1 ), con lo cual : ≠ , lo que significa que tres ( x3 − x2 ) ( x2 − x1 ) puntos cualesquiera, cada uno de distinto color, no están alineados. Solución de Miguel Teixidó Román Consideremos una coloración de 2 d acuerdo con las siguientes reglas: -Azul si sus dos coordenadas son pares. -Verde si ambas coordenadas son impares. -Naranja cuando tienen una coordenada de cada paridad. Demostraremos que tal coloración cumple las condiciones requeridas. Y X uuu r uuur Observamos que tres puntos A, B, C de 2 están alineados si y sólo si existe k tal que AB = k AC . Caso 1. Supongamos que A azul, B verde y C naranja con la primera coordenada par están uuu r uuur alineados, la condición AB = k AC sobre la primera componente queda: bx − ax = k ( cx − ax ) y reduciéndola módulo 2 resulta: 1 − 0 = k ( 0 − 0 ) ⇔ 1 = 0·k contradicción que prueba que no existen tales puntos. Caso 2. Igual que el anterior con la primera coordenada de C impar y por tanto la segunda par. Razonando del mismo modo sobre la segunda componente tenemos: by − a y = k ( c y − a y ) y módulo 2 resulta 1 − 0 = k ( 0 − 0 ) ⇔ 1 = 0·k y la correspondiente contradicción. La segunda condición también se cumple: El azul se repite infinitamente en las rectas de la forma y = k para k par. El verde se repite infinitamente en las rectas de la forma y = k para k impar. El naranja se repite infinitamente en las rectas de la forma y = k para cualquier k . Problema 3 Diremos que un triángulo es multiplicativo si el producto de las longitudes de dos de sus lados es igual a la longitud del tercer lado. Sea ABC...XYZ un polígono regular de n lados con todos sus lados de longitud 1. Las n – 3 diagonales que salen del vértice A dividen al A Z P Q R B -2- triángulo ZAB en n – 2 triángulos más pequeños. Probar que cada uno de esos triángulos es multiplicativo. Solución oficial. El ángulo formado pos dos diagonales consecutivas con un extremo en A es el mismo por inscrito en el mismo arco α. Sean PQ y QR dos segmentos adyacentes sobre el segmento ZB, determinados por tres diagonales consecutivas. Entonces 1 1 S APQ = h·PQ = AP· AQ·senα 2 2 1 1 S AQR = h·QR = AQ· AR·senα 2 2 donde h es la altura común a todos los triángulos. Por tanto AP· AQ AQ· AR = , PQ QR es decir, la razón r del producto de dos lados a la base es la misma para cualquier par de triángulos adyacentes, y por tanto es la misma para todos ellos. Pero el primero y el último son claramente multiplicativo al tener dos lados iguales y el tercero igual a 1. Se deduce que r = 1 y todos los triángulos son multiplicativos. Solución de Anas El Barkani que mereció una mención especial del jurado. Vamos a probar el enunciado demostrando que si un triángulo A B es multiplicativo, también lo es el colindante. Obsérvese primero que el triángulo ABQ es isósceles puesto Q que los ángulos A y B abarcan el mismo arco. Es obvio que el P triángulo ABQ es multiplicativo pues AB = 1 . Z C QB = AQ· AB = AQ·1 = AQ (1) Ahora bien, si aplicamos el teorema de la bisectriz al triángulo APB tenemos que: PQ QB = = AQ ⇒ PQ = AQ· AP AP AB lo que prueba que APQ también es multiplicativo. c.q.d. Problema 4 Probar que para todo entero positivo n, la expresión decimal de 1 1 1 + + n n +1 n + 2 es periódica mixta. Solución: Tenemos 1 1 1 3n 2 + 6n + 2 + + = n n + 1 n + 2 n ( n + 1)( n + 2 ) Sabemos que para que una fracción origine un decimal periódico mixto, una vez reducida debe tener en el denominador algún factor primo del conjunto {2,5} y alguno que no sea ni 2 ni 5. -3- Veamos primero que la fracción anterior tiene en el denominador al menos una factor 2 más que en el numerador, en efecto Si n es par tenemos n = 2k que una vez sustituido resulta: 1 1 1 3n 2 + 6n + 2 12k 2 + 12k + 2 6 k 2 + 6k + 1 + + = = = n n + 1 n + 2 n ( n + 1)( n + 2 ) 2k ( 2k + 1)( 2k + 2 ) 2k ( 2k + 1)( k + 1) el numerador es impar y el denominados par. Si n es impar tenemos n = 2k + 1 que una vez sustituido resulta: 1 1 1 3n 2 + 6n + 2 12k 2 + 24k + 11 + + = = n n + 1 n + 2 n ( n + 1)( n + 2 ) ( 2k + 1)( 2k + 2 )( 2k + 3) el numerador es impar y el denominados par. En ambos casos el denominador tiene al menos un factor 2 que no está en el numerador. 1 1 1 3n 2 + 6n + 2 Además la expresión + + = muestra que el numerador no contiene el n n + 1 n + 2 n ( n + 1)( n + 2 ) factor primo 3 (da resto 2 al dividirlo entre tres) mientras el denominador al ser producto de tres números consecutivos es múltiplo de tres. Problema 5 Sean r , s, u , v números reales cualesquiera. Probar que: 1 mín { r − s 2 , s − u 2 , u − v 2 , v − r 2 } ≤ . 4 Solución. Supongamos que los cuatro números r − s 2 , s − u 2 , u − v 2 y v − r 2 son mayores estrictamente que 1 1 1 1 1 . Entonces r − s 2 + s − u 2 + u − v 2 + v − r 2 > + + + , pero esta expresión es equivalente a 4 4 4 4 4 1  1  1  1  0 >  − r  +  − s  +  − u  +  − v  que es una contradicción. 2  2  2  2  2 2 2 2 Problema 6 En un triángulo de lados a, b, c el lado a es la media aritmética de b y c. Probar: a) 0º ≤ A ≤ 60º. b) La altura relativa al lado a es tres veces el inradio r. c) La distancia del circuncentro al lado a es R – r. Solución. a) Por la desigualdad triangular: b+c b  b≤ + c ⇔ b ≤ 3c ⇔ ≤ 3  A  1 b 2 c ⇒ ≤ ≤3 b+c b 1 3 c c≤ + b ⇔ c ≤ 3b ⇔ ≥  2 c 3  Por el teorema de coseno: (b + c ) 4 2 T b R 3b 2 + 3c 2 − 2bc = b 2 + c 2 − 2bc cos A ⇒ cos A = 8bc r c ha I da a -4- B C B y C a los vértices opuestos a los lados a. 2 También puede localizarse el mínimo sin recurrir a la derivada teniendo en cuenta la desigualdad de las medias: 3x 2 − 2 x + 3 3 1 3 1 3 1 1 3 cos A = f ( x ) = = x− + = − +  x +  ≥ − + ·2 8x 8 4 8x 4 8 x 4 8 con igualdad para x = 1. resulta: y senA = Como 2 R = A senA 2tg 2 1+ 2R = a 4r 2 2 2 a 2 = a + r ⇒ a = 2 Rr − r 2 4r 4r 4 a que sustituida en (1) queda: da = R2 − 2 a2 2 = R 2 − 2 Rr + r 2 = ( R − r ) ⇔ d a = R − r 4 Solución de Elisa Lorenzo García. S al área y p al semiperímetro. I al incentro y ha a la altura correspondiente al lado a como se indica en la figura. 2 b) Designando A. b y c respectivamente. y de otra tg = 2 p−a a A 1 + tg a 2 . queda: c 3x 2 − 2 x + 3 3 1 3 1 = x− + con ≤ x ≤ 3 . Q es la intersección de la bisectriz de A con la mediatriz de a que está en el punto medio del arco BC. a2 2 De una parte d a = R 2 − (1) 4 A r 2r = . resulta -5- . tenemos: a+b+c 3ar  S = pr = r=  2 2  ⇒ 3ar = 1 ah ⇔ 3r = h  a a 1 2 2  S = aha  2  c) Pongamos da a la distancia entre el circuncentro y el lado a. Llamando x = PB . 1 Resumiendo queda ≤ cos A ≤ 1 ⇔ 60º ≥ A ≥ 0º .dividiendo numerador y denominador por c2 y llamando por comodidad de escritura x = cos A = f ( x ) = b . 8x 8 4 8x 3 1 Fácilmente se comprueba que f   = f ( 3) = 1 y que la derivada se anula en x = 1 donde hay un 3 1 mínimo que vale f (1) = . Los apartados a) y b) son esencialmente iguales a los de la solución oficial. Para el apartado c) se da la solución que sigue sin utilizar trigonometría. resulta que los triángulos PIQ y A’SQ son iguales y IP = A ' S = r de donde queda finalmente: OA ' = OS − A ' S = R − r -6- . 4 S de donde QA ' = PQ y como IPQ = QA ' S = 90º y IQP = A ' QS .a = x+ y = b+c 2   3c − b b= y+z ⇒ x = 4  c= z+x   A c I O Aplicando el teorema de la bisectriz: c b = BQ CQ b y BQ + CQ = a = además BA ' = Calculemos QA '− PQ = BA '− BQ − BQ + BP = b+c c 2 + bc 3c − b b − b + c + 3c c 2 + bc −2 + = − = c−c = 0 4 2b + 2c 4 4 b+c b+c c 2 + bc ⇒ BQ = 2 2b + 2c B P Q A' a C b+c por ser la mitad de a. Probar que el número de ortoedros con tal propiedad es finito. Probar que: Segunda sesión (25 de marzo) 4. siendo un número real positivo tal que . Pintamos toda su superficie (las seis caras).Sea los enteros un polinomio con coeficientes enteros.. lo cortamos mediante planos paralelos a las caras en cubos de una unidad de arista y observamos que exactamente la mitad de los cubos no tienen ninguna cara pintada. b y c a las distancias desde P a los lados BC.Las dimensiones de un ortoedro de madera son enteras. Llamamos a.ABC es un triángulo isósceles con AB = AC. 2.. AC y AB respectivamente. ). (Puede resultar útil tener en cuenta que ..XLII Olimpiada Matemática Española Fase nacional 2006 (Sevilla) Primera sesión (24 de marzo) 1. 3. Demostrar que si existe un entero k tal que ninguno de es divisible por k. Sea P un punto cualquiera de la circunferencia tangente a los lados AB en B y a AC en C.. entonces no tiene raíces enteras.Hallar todas las funciones que satisfacen la ecuación para todo par de números reales e positivos. Probar que el producto de cuatro naturales consecutivos no puede ser ni cuadrado ni cubo perfecto 6... Prueba que .5.Las diagonales y de un cuadrilátero convexo y del cuadrilátero se cortan en Denotamos por y a las áreas de los triángulos ¿Cuándo se alcanza la igualdad? respectivamente. K . Problema 3 ABC es un triángulo isósceles con AB = AC. Por reducción al absurdo. Solución. Si n fuese una raíz. Consiste en acotar el número de soluciones de la ecuación diofántica: abc = 2 ( a − 2 )( b − 2 )( c − 2 ) y para ello suponiendo que a ≤ b ≤ c . Probar que: . b sólo puede tomar un conjunto finito de valores. se pone en la forma: 3 1 a−2 b−2 c−2 = · · . (Puede resultar útil tener en cuenta que 3 2 Solución .8 ). Fijado a se acotan de modo análogo los posibles valores de b: De una parte a b−2 c−2 b−2 a b−2 = · ⇒ <  ≤ 2 ( a − 2) b c b  b  2 ( a − 2) y de ahí se sigue a b−2 a 5 < < ≤ <1 2 ( a − 2) b 2 ( a − 2) 6 2 con lo que para cada valor de a. Pintamos toda su superficie (las seis caras) y lo cortamos en cubos de una unidad de arista y observamos que exactamente la mitad de los pequeños cubos no tienen ninguna cara pintada. P ( k ) es divisible por k. demostrar que si existe un entero k tal que ninguno de los enteros P (1) . < 1. entonces P ( x ) no tiene raíces enteras. con 1 ≤ r ≤ k (basta hacer la división entera y en el caso de ser resto cero se rebaja el cociente en una unidad). P ( 2 ) . 79.Olimpiada Española 2006 Problema 1 Sea P ( x ) un polinomio con coeficientes enteros. b y c a las distancias desde P a los lados BC. Probar que el número de paralelepípedos con tal propiedad es finito. Pongamos a. por una parte tenemos P ( x) = ( x − n) Q ( x) y por otra siempre existen enteros q y r tales que n = kq + r . Finalmente fijados a y b a lo sumo hay un valor para c que cumpla la ecuación inicial lo que prueba que el número de soluciones es finito. y como 2 a b c 1 a−2 3 1  a−2 1 a−2 ⇒ < ≤   ≤ < a a 2 2  a  2 fácilmente se obtiene que 4 < a < 10. Problema 2 Las dimensiones de un paralelepípedo de madera son enteras.. 1 1. Sea P un punto cualquiera de la circunferencia tangente a los lados AB en B y a AC en C. entonces P ( r ) = ( r − n ) Q ( r ) = −kqQ ( r ) en contra de lo supuesto en el enunciado.. AC y AB respectivamente. a 2 = b·c Solución: Pongamos m = PB. n = PC. siendo λ un número real positivo tal que f (λ ) = 1. R y S las proyecciones de P sobre cada lado y sea P’ el punto diametralmente opuesto a P . De este ( f (1) − 1) 2 = 0 y así f (1) = 1. Sustituyendo ahora y = 1 en la ecuación funcional resulta λ λ  f ( x) f (1) + f   f (λ ) = 2 f ( x). x λ Entonces f ( x) f   = 1. ∞) → R que satisfacen la ecuación λ λ f ( x) f ( y ) + f   f   = 2 f ( xy ) x  y para todo par de números reales x e y positivos. x . Por la semejanza de los triángulos PBP’ y PBS se tiene: m 2r = ⇔ m 2 = 2cr c m A (1) De modo análogo por la semejanza de PCP’ y PBC se cumple: n 2r = ⇔ n 2 = 2br (2) b n S c P m a Q b R Por el teorema de los senos en PBC: ∧ n C sen( PBC ) = y en el triángulo rectángulo PQB: ∧ n 2r a m B sen( PBC ) = P' mn m2 n 2 ⇔ a2 = y por (1) y (2) queda finalmente: de donde a = 2r 4r 2 a2 = 2cr ·2br = b·c 4r 2 Problema 4. Hallar todas las funciones f : (0. por lo que f 2 ( x) = 1 para todo x > 0. Q. Tomemos a x x continuación y = λ x y observemos que λ f ( x) f   + x λ f   f ( x) = 2 f (λ ). Solución: Haciendo x = y = 1 en la ecuación funcional dada se tiene que modo f 2 (1) + f 2 (λ ) = 2 f (1). que es equivalente a f ( x) = f   para todo x > 0. S3 = nb. Entonces S1 = 1 1 1 1 mb. tenemos que probar que S1 + S 2 ≤ S1 + S 2 + S3 + S 4 . N = 24). ( t )+ f 2 Problema 5 Probar que el producto de cuatro naturales consecutivos no puede ser ni cuadrado ni cubo perfecto. Denotamos por S1 . Solución. ¿Cuándo se alcanza la igualdad? Solución: Denotando. como en el enunciado. d = DK .Sustituyendo f2 x = y = t de nuevo en la ecuación funcional inicial se tiene que  λ    = 2 f (t ) y debido a ser el miembro de la izquierda positivo f es positivo y  t f ( x) = 1 para todo x > 0. S1 = md . n + 1 es impar y por tanto n + 1 es primo con el producto M = ( n − 1) n ( n + 2 ) = n3 + n 2 − 2n que también debe ser cubo perfecto. Llamamos b = BL. 3 2 Prueba que S1 + S 2 ≤ S . Si N fuese cubo perfecto. 3 Finalmente. entonces n es primo con los otros tres factores y si N es cubo perfecto. como x > 2 ⇒ x 3 < x3 + x 2 − 2 x < ( x + 1) se sigue la contradicción. Distinguimos ahora dos casos: a) n impar. Elevando al cuadrado la anterior desigualdad. Estos puntos K y L pueden estar dentro o fuera del segmento AC. también lo es M = ( n − 1)( n + 1)( n + 2 ) = n3 + 2n 2 − n − 2 pero si n > 2 ⇒ n3 < n3 + 2n 2 − n − 2 < ( n + 1) y ya tenemos la contradicción pues entre dos cubos consecutivos no puede haber otro cubo. CDE y del cuadrilátero ABCD respectivamente. Problema 6 Las diagonales AC y BD de un cuadrilátero convexo ABCD se cortan en E. m = AE . la áreas de los triángulos BCE y DAE por S3 y S4 respectivamente. Se comprueba fácilmente que esta función constante e idénticamente igual a 1 es solución de la ecuación funcional del enunciado y además es la única. se obtiene la desigualdad equivalente 2 S1S 2 ≤ S3 + S 4 (1). Si el producto N = ( n − 1) n ( n + 1)( n + 2 ) fuese un cuadrado. b) si n es par. n = CE. Sean ahora K y L los pies de las perpendiculares en la diagonal AC trazadas desde D y B respectivamente. S 2 y S a las áreas de los triángulos ABE . S 2 = nd . basta ponerlo en la forma N = ( n − 1) n ( n + 1)( n + 2 ) = ( n 2 + n − 2 )( n 2 + n ) = ( n 2 + n − 1) − 1 de donde se sigue una contradicción (no hay dos cuadrados consecutivos). podemos suponer que n > 2 (si n = 2 . 2 2 2 2 . es decir el cuadrilátero dado es un trapecio con los lados paralelos AB y CD . Esta es la condición para que se alcance (1). b m Esta última desigualdad se alcanza si y sólo si nb = md ⇔ = (2).Sustituyendo esta expresión en la 1 ( nb + md ). Y recíprocamente por la semejanza de DE CE triángulos (3) se verifica si y sólo si AB y CD son paralelos. que es 2 precisamente la desigualdad entre la media aritmética y la media geométrica de los dos productos nb y md . La relación (2) se convierte en = (3). por la semejanza entre los triángulos BLE d DK DE BE AE y DKE. Así = = . desigualdad (1) se llega a mb ⋅ nd ≤ . d n b BL BE Las rectas BL y DK son paralelas. ..XLIII Olimpiada Matemática Española Fase nacional 2007 (Torrelodones) Primera sesión (23 de marzo) 1. Probar que se cumple: Segunda sesión (24 de marzo) 4. Sea E la .. como la suma de al menos dos números enteros positivos consecutivos? ¿Cuál es el menor de todos ellos? Ejemplo: el número 9 se escribe exactamente de dos maneras distintas: 9=4+5 9=2+3+4 5.Sean cinco números positivos en progresión aritmética de diferencia d.¿Cuáles son los números enteros positivos que se pueden obtener de exactamente 2007 maneras distintas.Sea un número real positivo y un entero mayor que 1. Probar que 2.Determinar todos los posibles valores enteros no negativos que puede tomar la expresión siendo y enteros no negativos tales que 3. Demostrar que 6.. La bisectriz que parte de A corta al lado opuesto en P. se considera una cuerda CD de longitud fija c..Sea O el circuncentro de un triángulo ABC..Dada una semicircunferencia de diámetro AB = 2R. . Probar que el segmento EF tiene longitud constante y dirección constante al variar la cuerda CD sobre la semicircunferencia. . Sea E la intersección de AC con BD y F la intersección de AD con BC.Dada una semicircunferencia de diámetro AB = 2R.6. se considera una cuerda CD de longitud fija c. a − d . La desigualdad dada puede escribirse como 3 3 3 3 10a2 ≤ a0 + 4a13 + 4a3 + a4 . Sean a0 . Probar que 3 a2 ≤ 1 3 ( a0 + 4a13 + 4a33 + a43 ) . 3 a2 ≤ 1  4  3  4  3  4  3  4  3  4  3  1 3 3 3 3 3   a0 +   a1 +   a2 +   a3 +   a4  = ( a0 + 4a1 + 6a2 + 4a3 + a4 ) . la progresión es a − 2d . convexa en ( 0. con  4  4 f  ∑ pk ak  ≤ ∑ pk f ( ak )  k =0  k =0 o equivalentemente. a4 cinco números positivos en progresión aritmética de razón d. +∞ ) . a4 están en progresión aritmética. a2 . a3 . a. 3 y sumando 6a2 a ambos miembros se convierte en 3 a2 ≤ 1 3 ( a0 + 4a13 + 6a23 + 4a33 + a43 ) . 1 2 3 4   16 2 k  0      Obsérvese que la igualdad tiene lugar cuando los cinco números son iguales y hemos terminado. a1 . Llamando a al término central y d a la diferencia.Olimpiada española 2007 Problema 1. a1 . a + 2d y tenemos: 3 a0 = ( a − 2d ) = a 3 − 6a 2 d + 12ad 2 − 8d 3 3 a4 = ( a + 2d ) = a 3 + 6a 2 d + 12ad 2 + 8d 3 3 3 4a13 = 4 ( a − d ) = 4a 3 − 12a 2 d + 12ad 2 − 4d 3 3 4a3 = 4 ( a + d ) = 4a 3 + 12a 2 d + 12ad 2 + 4d 3 3 3 sumando: 1 . a + d . 10 Solución 1 (del autor de la propuesta). a3 . 0 ≤ k ≤ 4 resulta k  2 f ( t ) = t 3 . 0 1  2  0   1   2   3   4   Aplicando la desigualdad de Jensen a la función  4 1 pk =   k . entonces 4 4 4  4  4 4  4  4   a0 +   a1 +   a2 +   a3 +   a4 = ( a0 + a4 )   + ( a1 + a3 )   +   a2 = 0 1 2  3  4 0 1  2  4   4   4   4   4    4 4  4 2a2   + 2a2   + a2   = a2   +   +   +   +    = 24 a2 . a2 . 16 Por otro lado como a0 . Solución 2. la desigualdad a probar se escribe como 3 24 24 2 2 3 3 3 a2 ( a2 − 4d 2 + 4a2 − 4d 2 ) ≤ 3a2 ⇔ 3a2 − d 2 ≤ 3a2 ⇔ − d 2 ≤ 0 5 5 5 la última desigualdad es cierta y hemos terminado. m 2 + mn + n 2 = k . el número k = 3 + 2 es un entero positivo si m = 0 ó m = 2. Problema 2. Determinar todos los posibles valores enteros no negativos que puede tomar la expresión m 2 + mn + n 2 . k ∈ N (naturales con el 0). siendo m y n enteros no negativos tales que mn ≠ 1. 10 cuyo segundo miembro es 3 4a2 − trasponiendo términos.8ad 2 ≥ 0 10 con independencia del valor de d. Solución 3. Como se trata de cinco términos en progresión aritmética. Entonces 3 3 3 a0 + a4 = ( a0 + a4 ) − 3a0 a4 ( a0 + a4 ) = 8a2 − 6a0 a4 a2 . se tiene a0 + a4 = 2a2 = a1 + a3 o también a0 a4 = ( a2 − 2d )( a2 + 2d ) . lo que hay que probar es 3 a2 ≤ 1 (8a23 − 6a0 a1a2 + 32a23 − 24a1a3a2 ) 10 6 a2 ( a0 a4 + 4a1a3 ) . de donde m −1 k = 0 ó k = 4 respectivamente. mn − 1 Solución. Sea 2 . mn − 1 3 En el caso m = n. 3 y 3 3 3 4 ( a13 + a3 ) = a ( a1 + a3 ) − 3a1a3 ( a1 + a3 )  = 4 ( 8a2 − 6a1a0 a2 ) .3 3 3 a0 + 4a13 + 4a3 + a4 = 10a 3 + 48ad 2 dividiendo por 10 queda 1 3 ( a0 + 4a13 + 4a33 + a43 ) − a3 = 4.   Entonces. n) tales que m > n > 0. Como la relación dada es equivalente a m 2 − (k − 1) mn + n 2 + k = 0. 3m − 1 Entonces los posibles valores k enteros no negativos de la expresión del enunciado son 0 . obtenemos 9k = 3m + 10 + 91 . obtenemos 4k = 2m + 5 − Para n = 3. n −1 2 k< m >n+ Si 3n < n 2 − 1. obtenemos k = m + 2 − 3 . Los triángulos ABM y APC son semejantes al tener dos A ángulos iguales. Entonces: c b O c AP = ⇔ bc = AM · AP AM b como AM = AP + PM. De este modo. Consideremos ahora las soluciones (m.El caso n = 0 lleva a que k = −m 2 y por tanto m = k = 0. que no conduce a ninguna solución. ( k − 1)n − m ) es también una solución. . < n mn − 1 n 3n para n > 1. esto es si n ≥ 4. k = 7. n) es una solución y n > ( k − 1) n − m > 0. n) con m > n ≥ 4 encontramos una solución ( n. entonces ( n. m −1 21 . observamos que si (m. Para n = 1. la desigualdad anterior es cierta y por tanto para cada solución (m. cada solución es tal que n ≤ 3. queda: P C B bc = AP ( AP + PM ) = AP 2 + AP·PM M 3 . p ) con n > p > 0. > n mn − 1 n n 3 > −m − n. 4 y 7. 2m − 1 Para n = 2. Probar que se cumple: AP 2 + OA2 − OP 2 = bc Solución: Prolongamos AP hasta que corte en M al circuncírculo. k = 4 ó k = 7. .(∠ACB = ∠AMB por inscritos en el mismo arco y ∠BAN = ∠CAN por bisectriz). La bisectriz que parte de A corta al lado opuesto en P. Del mismo modo la desigualdad n > ( k − 1) n − m es sucesivamente equivalente a: m + 2n m 2 + mn + n 2 m + 2n . m = 4 ó m = 2. m = 4 ó m = 11. Sea O el circuncentro de un triángulo ABC. La desigualdad ( k − 1) n − m > 0 es cierta en todos los casos porque se convierte sucesivamente en una desigualdad obvia: k> m + n m 2 + mn + n 2 m + n . Problema 3. AP·PM es la potencia de P respecto de la circunferencia circunscrita y su valor es OA2 – OP2 sólo queda sustituir y resulta: bc = AP 2 + OA2 − OP 2 Problema 4. ¿Cuáles son los números enteros positivos que se pueden obtener de exactamente 2007 maneras distintas, como la suma de al menos dos números enteros positivos consecutivos? ¿Cuál es el menor de todos ellos? Ejemplo: el número 9 se escribe exactamente de dos maneras distintas: 9=4+5 9=2+3+4 Solución N = a + ( a + 1) + K + ( a + n ) = ( n + 1)( 2a + n ) ⇔ 2 N = 2 ( n + 1)( 2a + n ) Si n es par ( n + 1) es impar y ( 2a + n ) es par. Si n es impar ( n + 1) es par y ( 2a + n ) es impar. Siempre que 2N en su descomposición en factores primos tenga un factor impar distinto de uno, existe una descomposición de N en suma de números consecutivos. En efecto, sea N = ( 2q + 1) Q con q, Q naturales y q ≥ 1 , Q > 1 , entonces: a) 2( 2q + 1).Q = ( n + 1)( 2a + n) , y hacemos la siguiente descomposición: 2q + 1 = n + 1 , es decir n = 2q y n > 1 2Q = 2a + 2q , de donde a = Q − q Si a > 0 ya hemos terminado, en otro caso Q − q ≤ 0 y entonces, b) 2Q = n + 1 , de donde n = 2Q − 1 , es decir n ≥ 1 y 2a + 2Q − 1 = 2q + 1 de donde a = 1 + q − Q , es decir a ≥ 1 y ya hemos terminado. Entonces necesitamos el menor número que en su descomposición tenga 2007 factores impares sin contar el 1. Si sólo tiene un factor primo es: 2 N = 3 2007.2 Si tiene más de uno p α q β r γ .. , el número de divisores que tiene es (α + 1)( β + 1)(γ + 1)...... , y este producto debe ser 2008 para que excepto l uno tenga 2007 divisores impares distintos. 2008 = 2 3.251 = (α + 1)( β + 1)(γ + 1)...... , entonces: N = 3250.57 , o bien N = 3 250.53.7 , o bien N = 3250.5.7.11 y este último es el menor. Problema 5. Sea a ≠ 1 un número real positivo y n un entero positivo. Demostrar que n 2 < Solución. an + a−n − 2 . a + a −1 − 2 4 n −   n 2 2 a − a  an + a−n − 2  , que a su vez equivale a La desigualdad dada n 2 < es equivalente a n 2 <  1 1 2 a + a −1 − 2 −   2 2 a − a    n −n α −α que n < , siendo α = a . Entonces, usando la desigualdad aritmético-geométrica, se tiene α − α −1 la desigualdad pedida: 2 α n − α −n α 2n − 1 = α 1− n 2 = α 1− n (1 + α 2 + α 4 + ... + α 2 n − 2 ) > α 1− n n n α 2+ 4+...+ (2 n − 2) = α 1− n nα n−1 = n. α − α −1 α −1 Problema 6. Dada una semicircunferencia de diámetro AB = 2R, se considera una cuerda CD de longitud fija c. Sea E la intersección de AC con BD y F la intersección de AD con BC. Probar que el segmento EF tiene longitud constante y dirección constante al variar la cuerda CD sobre la semicircunferencia. Solución. Como los triángulos EFC y EDF son rectángulos, el cuadrilátero EDFC es inscriptible y EF es el diámetro. Llamemos r al radio del circuncírculo de ECD, por el teorema de los senos en ECD: c EF = 2r = (1) sen E Pongamos α = ∠BOD y β = ∠COD. Entonces 180 − β β E = = 90 − 2 2 A E D C F β O α B expresión que prueba que el ángulo E es constante al serlo β y además el punto E se mueve en el β arco capaz de 90 − sobre AB. Sustituyendo en (1) queda: 2 c β c (3) ; eliminando β entre (2) y (3) y despejando EF = 2r = ( 2 ) , por otra parte sen = β 2 2R cos 2 2cR EF resulta: EF = expresión que muestra que EF es constante al serlo c y R. 4R2 − c2 Además F es el ortocentro del triángulo ABE como intersección de las alturas AD y BC, por ello EF que está sobre la tercera altura es siempre perpendicular a AB. También admite solución analítica aunque mucho más larga. 5 XLIV Olimpiada Matemática Española Fase nacional 2008 (Valencia) PRIMERA SESIÓN (28 de marzo) • 1.- Halla dos enteros positivos a y b conociendo su suma y su mínimo común múltiplo. Aplícalo en el caso de que la suma sea 3972 y el mínimo común múltiplo 985928. • 2.- Prueba que para cualesquiera números reales a, b tales que 0 < a, b < 1, se cumple la desigualdad siguiente: ab 2 + a 2 b + (1 − a)(1 − b) 2 + (1 − a ) 2 (1 − b) < 2 . • 3.- Sea p ≥ 3 un número primo. Se divide cada lado de un triángulo en p partes iguales y se une cada uno de los puntos de división con el vértice opuesto. Calcula el número máximo de regiones, disjuntas dos a dos, en que queda dividido el triángulo. No está permitido el uso de calculadoras. Cada problema se valora hasta 7 puntos. El tiempo de cada sesión es de tres horas y media. se trazan las rectas PA y PB que cortan a r en C y D respectivamente. M y N. tales que la media aritmética de todos los divisores positivos del número n = p a q b es un número entero.XLIV Olimpiada Matemática Española Fase nacional 2008 (Valencia) SEGUNDA SESIÓN (29 de marzo) • 4. Prueba que existen enteros positivos a y b. otro variable P y una recta r ... • 5. El tiempo de cada sesión es de tres horas y media. • 6.Sean p y q dos números primos positivos diferentes. B. tales que el producto CM ·DN sea constante al variar P. . ¿Existirá un trapecio inscriptible en una circunferencia cuyos vértices tengan todos el mismo color? No está permitido el uso de calculadoras.A cada punto del plano se le asigna un solo color entre siete colores distintos.Dada una circunferencia y en ella dos puntos fijos A. Determina dos puntos fijos de r . Cada problema se valora hasta 7 puntos.. b ] = (a + b) 2 − 4ab = (a − b) 2 ≥ 0. Repitiendo el proceso anterior llegaremos a obtener dos enteros A y B tales que a = dA. b ]= 0. b ] ⎪ d ⎩ 2 B son las raíces de la ecuación de segundo grado d t − (a + b) t + [ a. Si p a. al dividir p a la suma a + b. para obtener dos enteros a1 y b1 [ a. En efecto: ∆ = (a + b) 2 − 4 d [ a. 985928 ) = 4. tenemos que d = ( 3972. b = dB y ( A. también p b. Como p [ a. Observamos que el discriminante de esta ecuación es no negativo. b1 ]= tales que a1 = . b1 = p p p d = ( a. a b y [ a1 . SOLUCIÓN: Sea p un número primo que divide a la suma a + b y a su mínimo común múltiplo [ a. Entonces [ A. b ] al menos divide a uno de los dos enteros a ó b. b ] . Es decir A = 491 y B = 502 y los números buscados son a = 1964 (año de la primera OME) y b = 2008 (año de la actual edición de la OME). Aplícalo en el caso de que la suma sea 3972 y el mínimo común múltiplo 985928. Por tanto podemos dividir los dos números a y b por Si a y b son distintos. la ecuación anterior tiene por soluciones los dos enteros b a y B= . B ] = AB. Por tanto a = 4 A y b = 4 B siendo A y B las raíces de la ecuación 4 t 2 − 3972 t + 985928 = 0.XLIV Olimpiada Matemática Española Fase nacional 2008 (Valencia) PRIMERA SESIÓN (28 de marzo) 1. hubiéramos supuesto que p b)..b ]. positivos A = d d En particular cuando a + b = 3972 y [ a. b ] = 985928. Sea el máximo común divisor de a y b. (Obviamente el mismo razonamiento vale si p y también su mínimo común múltiplo [ a.b ]. Ahora es fácil a+b ⎧ ⎪A + B = d ⎪ determinar A y B a partir del sistema de ecuaciones ⎨ .b ). 1 . Es decir A y ⎪ AB = [ a. B ) = 1.Halla dos enteros positivos a y b conociendo su suma y su mínimo común múltiplo. . a+b < 1. b < 1. 2 .XLIV Olimpiada Matemática Española Fase nacional 2008 (Valencia) PRIMERA SESIÓN (28 de marzo) 2.Prueba que para cualesquiera números reales a. b tales que 0 < a. 2 utilizando la desigualdad anterior y aplicando la desigualdad entre las medias aritmética y geométrica. 2 ⎠ ⎝ o equivalentemente 1 ( 2 ab 2 + a 2 b + (1 − a)(1 − b) 2 + (1 − a) 2 (1 − b) < 1. Teniendo en cuenta que 0 < SOLUCIÓN: ⎛ a+b⎞ 3 ⎛ a+b⎞ (1 − a) (1 − b) ⎜1 − ⎟ < (1 − a ) (1 − b) ⎜1 − ⎟≤ 2 ⎠ 2 ⎠ ⎝ ⎝ a+b 1− a +1− b +1− 2 =1 − a + b . ≤ 3 2 Sumando las expresiones anteriores resulta ⎛a+b⎞ ab ⎜ ⎟ + ⎝ 2 ⎠ ⎛ a+b⎞ (1 − a ) (1 − b) ⎜1 − ⎟ < 1. ) de donde se obtiene inmediatamente la desigualdad del enunciado. se tiene: ⎛a+b⎞ a+b+⎜ ⎟ ⎛a+b⎞ 3 ⎛a+b⎞ ⎝ 2 ⎠ = a+b ab ⎜ ⎟ < ab ⎜ ⎟≤ 3 2 ⎝ 2 ⎠ ⎝ 2 ⎠ y Se verifica que x < 3 x para todo x ∈ (0.1). se cumple la desigualdad siguiente: ab 2 + a 2 b + (1 − a)(1 − b) 2 + (1 − a) 2 (1 − b) < 2 . el número máximo de regiones disjuntas dos a dos. Las cevianas trazadas desde B dividen cada uno de los p triángulos anteriores en p partes disjuntas. Calcula el número máximo de regiones. m∈{1. AB respectivamente. = . disjuntas dos a dos. en que queda dividido el triángulo ABC es p 2 + ( p − 1)(2 p − 1) = 3 p 2 − 3 p + 1. o equivalentemente. Si AX . teniendo en total p 2 regiones. 2 k l m + p m ( p − k − l ) = p ( p − k ) ( p − l ). que es imposible y nuestra afirmación inicial queda probada. Dibujando las cevianas desde el vértice A el triángulo ABC queda dividido en p triángulos. Z puntos de las divisiones interiores de los lados BC .. Se divide cada lado de un triángulo en p partes iguales y se une cada uno de los puntos de división con el vértice opuesto. Sea el triángulo ABC y X . 2 ( p − 1) + 1 = 2 p − 1. p − 1 }. = ...2. De aquí resulta que p divide al producto klm. por la forma en que hemos construido los puntos de división. Es decir. existen enteros positivos k .XLIV Olimpiada Matemática Española Fase nacional 2008 (Valencia) PRIMERA SESIÓN (28 de marzo) 3. Y . ZB XC YA Por otro lado. tales que AZ k BX l CY m = . Por tanto. AC . BY y CZ fueran concurrentes aplicando el teorema de Ceva tendríamos AZ BX CY ⋅ ⋅ = 1. 3 .. Cada ceviana trazada desde el vértice C aumenta el número de regiones en un número exactamente igual a su número de intersecciones con las rectas que encuentra (incluido el lado AB ). en que queda dividido el triángulo. SOLUCIÓN: En primer lugar veremos que tres de estos segmentos (cevianas) no pueden ser concurrentes. ZB p − k XC p − l YA p − m Sustituyendo en la expresión anterior k l m = ( p − k ) ( p − l ) ( p − m). l .Sea p ≥ 3 un número primo.. + q ) (q + 1) entero.... tales que la media aritmética de todos los divisores positivos del número n = p a q b es un número entero. + 2 a .+ q ) (1 + q + q 2 + q 3 + . + q b ).. El número n tiene (a + 1)(b + 1) divisores positivos y la media aritmética de todos ellos es (1 + p + p 2 + . + 2 q + q +.... + q q ) m= = 1 + 2 + 2 2 + .. análogamente al caso anterior se eligen a = p b = p 2 + p 4 + ... + p a ) (1 + q + q 2 + . el primero se puede escribir como sigue 1 + p + p 2 + p 3 + .. Entonces m = (1 + p 2 + p 4 + .. Entonces (1 + 2 + 2 2 + .... se obtiene una solución completa observando que si p y q son primos impares se toman a = b = 1 y si p = 2 y q primo q −1 impar.. 4. Análogamente el segundo factor 1 + q + q 2 + q 3 + . + p p −1 ). entonces se consideran a = y b = 1..- SOLUCIÓN: La suma de todos los divisores de n viene dada por la fórmula (1 + p + p 2 + . que es un entero positivo.. + q b ) m= . + q q −1 ). 2 1 .. Si p = 2 y q es impar. Efectivamente: cada factor 1 + p + p 2 + p 3 + . + p p = (1 + p ) + p 2 (1 + p ) + . + q q tiene un número par de sumandos y por ejemplo. + p p y 1 + q + q 2 + q 3 + . + p a ) (1 + q + q 2 + .. que es 2 4 q −1 (1 + q + q + . tomando a = p y b = q. y Alternativamente y de una manera casi directa.. se eligen 2 4 q −1 b = q y a + 1 = 1 + q 2 + q 4 + . es fácil ver que m es un entero....... + p p −1 (1 + p ) = (1 + p ) (1 + p 2 + .XLIV Olimpiada Matemática Española Fase nacional 2008 (Valencia) SEGUNDA SESIÓN (29 de marzo) Sean p y q dos números primos positivos diferentes. como se puede comprobar desarrollando los paréntesis...... Prueba que existen enteros positivos a y b... + q q −1 )... + q q = (1 + q) (1 + q 2 + . (a + 1) (b + 1) Si p y q son ambos impares... + p p −1 ) (1 + q 2 + q 4 + . + p p −1 y m es entero. Para q = 2 y p impar. + q q −1 ... se trazan las rectas PA y PB que cortan a r en C y D respectivamente. En efecto. MC BN cantidad que no depende de P.XLIV Olimpiada Matemática Española Fase nacional 2008 (Valencia) SEGUNDA SESIÓN (29 de marzo) 5. M y N.. B. Determina dos puntos fijos de r .Dada una circunferencia y en ella dos puntos fijos A. otro variable P y una recta r. con argumentos análogos a los anteriores. ∠AMC = ∠AB´B = ∠DNB. SOLUCIÓN: Trazamos las paralelas a r por A y B que cortan a la circunferencia en A´ y B´ respectivamente de modo que AA´BB´ es un trapecio isósceles. donde la primera igualdad es cierta por ser ángulos inscritos en el mismo arco y la segunda por ser BB´ paralela a r. Estableciendo la proporcionalidad de los lados resulta AM ND = ⇔ MC ⋅ ND = AM ⋅ BN . 2 . tales que el producto CM ·DN sea constante al variar P. los triángulos AMC y DNB (sombreados en la figura) son semejantes ya que tienen dos ángulos iguales: A' A D C r M B B' P N ∠MAC = ∠B´BP = ∠NDB. Las intersecciones de AB´ y BA´ con r determinan los puntos M y N buscados. es constante. indicado anteriormente. no se forma el triángulo AMC. Como el número total de posibles pares es 49.. estos cuatro puntos son los vértices de un trapecio inscriptible.4..5. Por tanto los cuatro puntos determinados por estos dos bloques tienen el mismo color. distancia). estando los cuatro puntos sobre la circunferencia C . De este modo a cada uno de los 50 bloques se le hace corresponder el par (color. (c + 1)(c 2 + 1) longitud l . considerando c + 1 2 2 puntos en vez de 7 + 1 y c + 1 bloques disjuntos de c + 1 puntos cada uno. Se elige un sentido dado (por ejemplo.. En este caso se tomaría λ. Y ahora λ debe cumplir que 0 < λ < . es decir que los arcos A i A i +1 i = 1. por el principio el palomar. distancia). 0 < l ≤ 2π r. 0 < λ < 3 .. ¿Existirá un trapecio inscriptible en una circunferencia cuyos vértices tengan todos el mismo color? SOLUCIÓN: La idea inicial es considerar una circunferencia C de radio r y sobre ella bloques de 8 puntos A1 . existirán dos bloques R y Q a los que se les asocia el mismo par (color. en vez de 7 + 1 bloques l de 7 + 1 puntos cada uno.7 }. NOTA: Este mismo razonamiento se podría hacer considerando un arco de circunferencia de radio r de l ... en este sentido antihorario. 400 Y también se podría generalizar a un número de colores c (c ≥ 2) cualesquiera. para lo cual se toma λ suficientemente pequeño. n ∈ {1.A cada punto del plano se le asigna un solo color entre siete colores distintos.6. por πr ejemplo 0 < λ < . A 8 igualmente espaciados. M = A = C . Análogamente este producto es cero si la recta r pasa por B o por los puntos A y B en cuyo caso CM = DN = 0.. el antihorario). En este caso CM = 0 y el producto CM ⋅ DN = 0. XLIV Olimpiada Matemática Española Fase nacional 2008 (Valencia) SEGUNDA SESIÓN (29 de marzo) 6. Y como los dos puntos del bloque R distan igual que los dos puntos del bloque Q.Se observa que si la recta r pasa por el punto A. Se disponen entonces...3. se le asigna también al bloque. necesariamente. 7 × 7 + 1 = 50 bloques de 7 + 1 = 8 puntos cada uno en la semicircunferencia superior de C . 400 Se observa que al menos hay dos puntos del mismo color en cada bloque. Se eligen dos de esos puntos y su color se le asocia al bloque. tales que dos bloques distintos no se intersequen o solapen. A 2 .2.7 tengan igual longitud λ > 0 (que se elegirá convenientemente) para cada uno de los bloques. Y la distancia entre estos dos puntos que es uno de los siete números d n = nλ . si para cada vértice del poliedro. + an = 2009. Se dice que una coloración de este tipo es buena.. 27 de marzo de 2009 PRIMERA SESIÓN 1. a n . Por otra parte... con n ≥ 3. a 2 . Demuestra que DI 2 = ( 2 R − ha ) ( ha − 2r ). ninguna cara del poliedro tiene todas sus aristas pintadas de rojo. 3. se dice que una coloración donde se pintan de rojo algunas de las aristas de un poliedro regular es completamente buena. Cada problema vale siete puntos. 2.Sean ABC un triángulo acutángulo. tales que a1 + a2 + .Se pintan de rojo algunas de las aristas de un poliedro regular. con D perteneciente al lado BC. ¿Para qué poliedros regulares es igual el número máximo de aristas que se pueden pintar en una coloración buena y en una completamente buena? Justifica la respuesta.. ... r su radio y R el radio del círculo circunscrito al triángulo ABC.Fase Nacional de la XLV Olimpiada Matemática Española Sant Feliu de Guixols (Girona). si.. además de ser buena. El tiempo de cada sesión es de tres horas y media. No está permitido el uso de calculadoras. existe una arista que concurre en dicho vértice y no está pintada de rojo.. Se traza la altura AD = ha . I el centro del círculo inscrito en el triángulo ABC .Halla todas las sucesiones finitas de n números naturales consecutivos a1 .. Se considera P un punto variable sobre esta circunferencia y se traza la cuerda PP´..Determina justificadamente todos los pares de números enteros ( x. y ) que verifican la ecuación x 2 − y 4 = 2009.En el interior de una circunferencia de centro O y radio r . Sea C el punto simétrico de B respecto de PP´.. perpendicular a AP. 5.. b.Sean a. Halla el lugar geométrico del punto Q. c números reales positivos tales que abc = 1. No está permitido el uso de calculadoras. 28 de marzo de 2009 SEGUNDA SESIÓN 4. . simétricos respecto de O. intersección de PP´ con AC . se toman dos puntos A y B. Prueba la desigualdad siguiente 3 ⎛ a ⎞ ⎛ b ⎞ ⎛ c ⎞ ⎟ +⎜ ⎟ +⎜ ⎟ ≥ ⎜ 4 ⎝ 1 + ab ⎠ ⎝ 1 + bc ⎠ ⎝ 1 + ca ⎠ 2 2 2 6. Cada problema vale siete puntos. El tiempo de cada sesión es de tres horas y media. al variar P sobre la circunferencia.Fase Nacional de la XLV Olimpiada Matemática Española Sant Feliu de Guixols (Girona). .. + k + (k + 1) + . + (k + n) ] − [1 + 2 + .. + an = 2009.... + 150. resulta k = 136 con lo que 2 2009 = 137 + 138 + .. (2k + n + 1) = 49 resulta k = 28 con lo que (2k + n + 1) = 41 resulta (4) Si n = 49 y 2 2009 = 17 + 18 + 19 + . + 69. Primera solución: Supongamos que N es la suma de n números naturales consecutivos empezando por k + 1. .. a 2 .. tales que a1 + a2 + . + k ] = (k + n)(k + n + 1) k (k + 1) n(2k + n + 1) − = ..... 27 de marzo de 2009 PRIMERA SESIÓN SOLUCIONES PROBLEMA 1.Halla todas las sucesiones finitas de n números naturales consecutivos a1 .. (2k + n + 1) = 287 (2) Si (3) Si n = 41 y 2 2009 = 29 + 30 + 31 + .. + (k + n) = [1 + 2 + .. a n . Entonces N = (k + 1) + (k + 2) + . k = 16 con lo que (5) Los otros casos dan valores de k que no verifican el enunciado.... + 65. con n ≥ 3.Fase Nacional de la XLV Olimpiada Matemática Española Sant Feliu de Guixols (Girona). n = 7 y 2k + n + 1 = 287.. 2 2 2 Teniendo en cuenta que 2009 = 1 × 2009 = 7 × 287 = 49 × 41 se tienen los siguientes casos: (1) Si n = 7 y resulta k = 283 con lo que 2 2009 = 284 + 285 + 286 + 287 + 288 + 289 + 290.. 29. pues cualquier otro divisor impar de 2009 es mayor o igual que 7 × 41 = 287..138. 41 ó 49. No hay otras sucesiones con un número impar de términos. 69. de donde 2009 = n a1 + 2009 ≥ n (n − 1) ..18. Se obtiene entonces 2009 n − 1 a1 = − .. Entonces 2 n (n − 1) (n − 1) 2 .. Entonces a1 = 137. y la única sucesión con número par de términos es 137. 65. es a n = a1 + n − 1. 29 y 17... con valores respectivos 284.. Supongamos en primer lugar que n es impar.150. Entonces divide a 2009.... y al ser > 2 2 64 2 = 4096 > 4018 > 3969 = 63 2 ... obviamente n divide a 2009 = 7 2 × 41 y n puede tomar los valores 7... resulta que 4018 < 64. n Supongamos finalmente que n es par. Entonces.Segunda solución: Claramente. 17. 290. luego n − 1 < 4018 . ya que cualquier otro valor de n ha de ser mayor o igual que 2 × 41 = 82 > 65. con lo que n < 65. es decir. .. lo que no es posible. se obtienen 2 n las tres sucesiones 284. 285. con lo que 2 n = 14.. 30. Fase Nacional de la XLV Olimpiada Matemática Española Sant Feliu de Guixols (Girona). Se traza la altura AD = ha . c. evidentemente. r su radio y R el radio del círculo circunscrito al triángulo ABC. con D perteneciente al lado BC. . y teniendo en cuenta los resultados anteriormente obtenidos resulta. respectivamente. bc ⋅ senA A A Por otra parte. obtenemos como AI = p bc 2 bc ( p − a ) (2). Primera solución: Sean E y M las proyecciones ortogonales de I sobre BC y AD .q. . Demuestra que DI 2 = ( 2 R − ha ) ( ha − 2r ). r= A A p sen p ⋅ sen 2 2 S es el área del triángulo ABC y p es su semiperímetro. finalmente. S S r Se tiene: AI = ⇒ AI = (1) donde.Como el cuadrilátero IEDM es un rectángulo. 2 DI 2 = ha + 2 R ( ha − 2r ) − 2ha ( ha − r ) = (2 R − ha ) ( ha − 2r ). S = = bc ⋅ sen ⋅ cos .d. la expresión (2) se r ha 2S ⎞ ⎛ ⎜ ⎟ 2 ⎜1 − ha ⎟ = 2 R ⋅ h ⎛1 − 2r ⎞ = 2 R (h − 2r ) ⎜ ⎟ escribe como AI = 2 R ⋅ ha a⎜ a ⎟ ⎜ S ⎟ ⎝ ha ⎠ ⎜ ⎟ r ⎠ ⎝ (3).Sean ABC un triángulo acutángulo. MD = IE = r. p = . AI 2 = p 2S S Teniendo en cuenta que bc = 2 R ⋅ ha . I el centro del círculo inscrito en el triángulo ABC . y ya que cos 2 A = p ( p − a ) .. 27 de marzo de 2009 PRIMERA SESIÓN SOLUCIONES PROBLEMA 2 . a = . Aplicando el teorema de Pitágoras generalizado a ADI tenemos 2 2 DI 2 = ha + AI 2 − 2ha ⋅ AM ⇔ DI 2 = ha + AI 2 − 2ha ( ha − MD ). así que (1) se puede escribir 2 2 2 A bc ⋅ cos 2 . tenemos que BT ⋅ AC 2 + CT ⋅ AB 2 2 AT = − BT ⋅ CT = BC 3b 2 + 3c 2 − a 2 − 2bc (b − c) 2 (b + c) − 4 2a con lo que b 2 + c 2 − a 2 + ab + ca (b − c) 2 (b + c) . Claramente. Por el se tiene que teorema de Pitágoras. 2a 2ab + 2bc + 2ca − a 2 − b 2 − c 2 Ahora bien. a+b+c a+b+c (b + c − a ) (c + a − b) (a + b − c) 2 R ha = bc y 2 R ha = .Segunda solución: Sean a. r2 = . c las longitudes de los lados BC. 2 2 Por el teorema de Stewart. al ser IT // AD ⊥ DT . CA y AB respectivamente. CT = . y sea T el punto donde la circunferencia inscrita es tangente al lado BC. se . 2 S − 2ar (b + c − a ) r ha − 2r = = = a a 2r sen . A 2R 2 A sen 2 R sen 2 2 A B C donde S es el área de ABC y se ha utilizado que r = 4 R sen sen sen . DI 2 = r 2 + IA 2 − ( ha − r ) 2 = IA 2 − ha ( ha − 2r ) = ( 2 R − ha ) ( ha − 2r ). b. Obsérvese que 4 Rr + r 2 = 4 c+a −b a+b−c sabe que BT = . 2 2 2 Sea ahora el punto P en el que la paralela a BC por I corta a la altura AD. es conocido que ah abc r (a + b + c) 1 (a + b + c) (b + c − a) (c + a − b) (a + b − c) = a . Ahora bien. S= = = 4R 2 4 2 (b + c − a ) (c + a − b) (a + b − c) 2abc De aquí se deduce que 4 Rr = . llamando S al área de ABC . = 2 2a e identificando términos se comprueba que esto coincide con el valor de 2 ( R + r ) ha y de este modo la igualdad requerida queda demostrada. 2 2 2 2 2 2 DI = r + DT = r + AT − AD y la igualdad a demostrar es equivalente a AT 2 + 4 Rr + r 2 = 2 ( R + r ) ha . AT 2 + 4 Rr + r 2 = Tercera solución: Nótese en primer lugar que B C sen r2 IA 2 2 2 = = . por lo tanto. El número máximo de aristas pintadas de rojo en una coloración 2 buena sería por lo tanto × 6 = 4. tales que en cada vértice confluyen 3 de ellas. en el dodecaedro existen 30 aristas. existe una arista que concurre en dicho vértice y no está pintada de rojo. La figura muestra una 3 coloración completamente buena con 8 aristas rojas. pues en caso contrario existiría algún vértice 3 2 de las aristas estuvieran pintadas de rojo. tales que en cada vértice confluyen 3 de ellas. El número máximo de aristas pintadas de rojo en una 2 coloración buena sería. Por otra parte. con lo que si el máximo número de aristas que se pueden pintar de rojo para obtener una coloración buena se puede alcanzar con una coloración completamente buena. En el caso de un tetraedro. todas las donde más de 3 aristas estarían pintadas de rojo. las coloraciones completamente buenas son un subconjunto de las coloraciones buenas. ¿Para qué poliedros regulares es igual el número máximo de aristas que se pueden pintar en una coloración buena y en una completamente buena? Justifica la respuesta. De igual forma. La figura muestra una coloración completamente buena de un tetraedro con 4 aristas rojas (el tetraedro ha sido deformado para poder ser dibujado en el plano).Se pintan de rojo algunas de las aristas de un poliedro regular. si además de ser buena. ninguna cara del poliedro tiene todas sus aristas pintadas de rojo..Fase Nacional de la XLV Olimpiada Matemática Española Sant Feliu de Guixols (Girona). Finalmente. existen 6 aristas. si para cada vértice del poliedro. Solución: Claramente. Se dice que una coloración de este tipo es buena. 27 de marzo de 2009 PRIMERA SESIÓN SOLUCIONES PROBLEMA 3. tales que en cada vértice confluyen 3 de ellas. es decir. El número máximo de aristas pintadas de rojo en una . se dice que una coloración donde se pintan de rojo algunas de las aristas de un poliedro regular es completamente buena. en el cubo existen 12 aristas. la pregunta del enunciado tiene respuesta afirmativa. × 12 = 8. NOTA: En cada caso véase el recuadro de las figuras al final de la solución. Finalmente. . Ahora bien. La figura muestra una coloración buena de un octaedro 4 con 9 aristas rojas. por lo tanto. con lo que hay. La figura muestra una 3 coloración completamente buena de un dodecaedro con 12 aristas rojas. obtenemos 48 = 2 × 20 + 8. existen. Por lo tanto. como cada arista pertenece a dos caras. el número máximo de aristas rojas en una coloración buena se alcanza con una coloración completamente buena. que sumando el número de aristas rojas de cada cara para todas las caras. tales que en cada vértice confluyen 5 de ellas. y una coloración buena con el número máximo de 24 aristas pintadas de rojo nunca puede ser completamente buena. tales que en cada vértice confluyen 4 de ellas. De lo anterior se deduce que para el tetraedro. los poliedros tales que en cada vértice confluyen exactamente 3 aristas) y los que no la tienen son el octaedro e icosaedro (en cuyos vértices confluyen más de 3 aristas).2 × 30 = 20. obtenemos 18 = 2 × 8 + 2. coloración buena sería. La figura muestra una 5 coloración buena de un icosaedro con 24 aristas rojas. Ahora bien. El número máximo de aristas pintadas de rojo en una 4 coloración buena es por lo tanto × 30 = 24. el cubo y el dodecaedro (es decir. el cubo y el dodecaedro. El número máximo de aristas pintadas de rojo en una coloración buena es 3 por lo tanto × 12 = 9. 8 caras con todas sus aristas rojas. dos caras con 3 aristas pintadas de rojo y una coloración buena con el número máximo de 9 aristas pintadas de rojo nunca puede ser completamente buena. es decir. al menos. En el octaedro existen 12 aristas. en el icosaedro existen 30 aristas. sumando para las 8 caras el número de aristas pintadas de rojo en dicha cara. los poliedros regulares que tienen la propiedad descrita en el enunciado son el tetraedro. eso quiere decir. al menos. FIGURAS FIGURAS . Determina justificadamente todos los pares de números enteros ( x. . es claro que también son soluciones ( x. con lo que existen enteros no negativos u y v tales que x = 7u . y = 7v y 2 2 (u + 7v )(u − 7v ) = 41. Solución: Dada una solución ( x. y ) que verifican la ecuación x 2 − y 4 = 2009. se tiene que 10 u + 7v 2 = 41 y u − 7v 2 = 1.Fase Nacional de la XLV Olimpiada Matemática Española Sant Feliu de Guixols (Girona).− y ). y ≥ 0. y ) cualquiera.− y ). con lo que se puede asumir sin pérdida de generalidad que x. con lo que u = 21 y v 2 = . absurdo pues 312 = 961 < 1004 < 322. y las únicas soluciones en enteros son ( x. y ) y (− x. No existen pues 7 soluciones enteras en este caso. con lo que y 2 = 1004. 28 de marzo de 2009 SEGUNDA SESIÓN SOLUCIONES PROBLEMA 4. que produce x = 45. un posible caso es que x − y 2 = 1 y x + y 2 = 2009. Resta entonces tan sólo el caso en que x − y 2 = 41 y x + y 2 = 49. Si x − y 2 y x + y 2 no son primos entre sí. . Si x − y 2 y x + y 2 son primos entre sí. Como ambos factores han de ser enteros. y 2 = 4. luego es 7 y divide a ( x + y 2 ) + ( x − y 2 ) = 2 x y a ( x + y 2 ) − ( x − y 2 ) = 2 y 2 . Es claro que ( x − y 2 ) ( x + y 2 ) = 7 2 ⋅ 41. y ) = (± 45. Supongamos entonces que es así. con lo que la única solución con enteros no negativos es x = 45 e y = 2. ± 2). (− x. su máximo común divisor al cuadrado divide a 2009 = 7 2 ⋅ 41. c(1 + a) a (1 + b) b(1 + c) 2 . c números reales positivos tales que abc = 1. ⎢⎜ 3 ⎢⎝ 1 + a ⎠ ⎝ 1 + b ⎠ ⎝ 1 + c ⎠ ⎥ 3 ⎢ ⎝ 1 + a ⎠ ⎝ 1 + b ⎠ ⎝ 1 + c ⎠ ⎥ ⎣ ⎦ ⎣ ⎦ Así es suficiente demostrar que ab bc ca 3 o equivalentemente + + ≥ 1+ a 1+ b 1+ c 2 abc abc abc 3 + + ≥ . Por tanto la ⎟ =⎜ ⎜ ⎝1+ b ⎠ ⎝ 1 + ca ⎠ 2 2 3 ⎛ ab ⎞ ⎛ bc ⎞ ⎛ ca ⎞ ⎟ +⎜ ⎟ +⎜ ⎟ ≥ .Fase Nacional de la XLV Olimpiada Matemática Española Sant Feliu de Guixols (Girona). Análogamente se ⎟ =⎜ ⎟ =⎜ ⎝1+ c ⎠ ⎝ abc + c ⎠ ⎝ 1 + ab ⎠ ⎛ ab ⎞ ⎛ b ⎞ ⎟ y ⎟ =⎜ ⎜ ⎝1+ a ⎠ ⎝ 1 + bc ⎠ desigualdad requerida se convierte en obtienen 2 2 2 2 2 2 2 2 ⎛ bc ⎞ ⎛ c ⎞ ⎟ . entonces ⎜ ⎟ . se obtiene 2 2 2 1 ⎡⎛ ab ⎞ ⎛ bc ⎞ ⎛ ca ⎞ ⎤ 1 ⎡ ⎛ ab ⎞ ⎛ bc ⎞ ⎛ ca ⎞ ⎤ ⎟ +⎜ ⎟ +⎜ ⎟ ⎥≥ ⎜ ⎟+⎜ ⎟+⎜ ⎟ . Prueba la desigualdad siguiente 3 ⎛ a ⎞ ⎛ b ⎞ ⎛ c ⎞ ⎜ ⎟ +⎜ ⎟ +⎜ ⎟ ≥ 4 ⎝ 1 + ab ⎠ ⎝ 1 + bc ⎠ ⎝ 1 + ca ⎠ Solución: 2 2 2 ⎛ ca ⎞ ⎛ ca ⎞ ⎛ a ⎞ Como abc = 1. ⎢⎜ 3 ⎢⎝ 1 + a ⎠ ⎝ 1 + b ⎠ ⎝ 1 + c ⎠ ⎥ 2 ⎣ ⎦ Usando ahora la desigualdad entre las medias aritmética y cuadrática. que a su vez equivale a que c(1 + a) a(1 + b) b(1 + c) 2 1 1 1 3 + + ≥ . b. equivalente a ⎜ 4 ⎝1+ a ⎠ ⎝1+ b ⎠ ⎝1+ c ⎠ 2 2 2 1 ⎡⎛ ab ⎞ ⎛ bc ⎞ ⎛ ca ⎞ ⎤ 1 ⎟ +⎜ ⎟ +⎜ ⎟ ⎥≥ . . 28 de marzo de 2009 SEGUNDA SESIÓN SOLUCIONES PROBLEMA 5.Sean a. −1 . γ = . β = y ⎜ y z⎟ ⎜x y⎟ 2 x y ⎝ z x⎠ ⎠ ⎠ ⎝ ⎝ 1 3 α β γ + + ≥ .Poniendo −1 a= x y . se llega a la desigualdad de Nessbit β +γ γ +α α + β 2 z La igualdad se alcanza si y sólo si a = b = c = 1.b= y z −1 y c= z x en la última desigualdad resulta ⎛ x x⎞ ⎛z z⎞ 3 1 1 ⎛ y y⎞ ⎜ + ⎟ + ⎜ + ⎟ + ⎜ + ⎟ ≥ . Sustituyendo ahora α = . al ser PP ´ la mediatriz de BC . Se considera un punto variable P sobre esta circunferencia y se traza la cuerda PP´. intersección de PP´ con AC . Se tiene CP´= P´B = AP´´. Primera solución: C P' Establezcamos primero que AC es constante. perpendicular a AP. al variar P sobre la circunferencia.En el interior de una circunferencia de centro O y radio r . Halla el lugar geométrico del punto Q. se toman dos puntos A y B. Método 1. . AA´= 2 MP. P' B O A P '' .Fase Nacional de la XLV Olimpiada Matemática Española Sant Feliu de Guixols (Girona). Se obtiene C a partir de A aplicando un giro de 180º con centro en O seguido de la simetría de eje PP´. C P Método 2 Prolongamos PA hasta que corte de nuevo a la circunferencia en P´´. Finalmente. es decir AC = 2 r . resulta que el triángulo e2 AA´C es rectángulo en A´ y además: B O e1 A' A de donde A´C = 2 OM . 28 de marzo de 2009 SEGUNDA SESIÓN SOLUCIONES PROBLEMA 6. con independencia de la posición de P. Además P´B es paralelo a PP´´. 2 2 2 2 2 AC = 4 OM + 4 MP = 4 OP = 4 r . resulta recto y AC = P´P´´= 2 r. P Descomponiendo el giro en producto de dos M simetrías de ejes perpendiculares e1 paralelo a AP y Q e2 perpendicular a AP. Sea C el punto simétrico de B respecto de PP´. luego el segmento CA es la imagen del segmento P´P´´ mediante la traslación de vector P´´A P y como ∠P´PP´´ es P´P´´ es un diámetro. simétricos respecto de O. y1 ) ⎧ x1 = 2 x 0 + a . 1+ x a 0 0 ⎝ (a − x) 2 y2 + =1 (ax − 1) 2 (ax − 1) 2 y mediante operaciones se transforma en la ecuación x 2 + ecuación de una elipse. Entonces la ecuación de la a las coordenadas de C . y0 = . llamando ( x1 . con 0 < a < 1. y 0 ) con la condición x0 + y 0 = 1. que es la 1− a2 Demostraremos en primer lugar que. y las coordenadas de A (a. es una circunferencia de centro el punto medio de AB y que tiene a los puntos A y B en su interior. B del plano. x0 y0 x +a y − 0 . Las coordenadas del punto Q.0) y sea P = ( x. B = (−d . Tercera solución: y2 = 1. se tiene ⎨ ⎩ y1 = 2 y 0 y recta CA es y = 0 ( x − a ).0).QC = QB.0). La recta PP´ es la tangente en Q a la elipse. k Así que si PA 2 + PB 2 = k ≥ 4 d 2 . Por Las pendientes de P´B y de P´P son respectivamente x0 + a y0 tanto la ecuación de P´P es y 0 y + x ( x 0 + a) − ax0 + 1 = 0. supongamos A = (d . sea P´ ( x 0 . se tiene que x 2 + y 2 = − d 2 ≥ d 2 . el conjunto de los puntos P (del mismo plano) tales que PA 2 + PB 2 es constante y mayor que AB 2 . y a las coordenadas de Q y ⎟ ⎠ a−x −y x0 = . 2 . Se tiene entonces PA 2 + PB 2 = 2 x 2 + 2 y 2 + 2 d 2 . P´ es el punto medio de BC. y ) cualquier punto. B (− a.0). intersección de AC y P´P son: ⎞ ⎟. es decir x0 y − y 0 x + y 0 a = 0. se deduce entonces que QB + QA = QC + QA = AC = 2 r y Q describe la elipse de focos A y B y constante 2 r. 2 2 En vez de empezar por P. despejando los valores de x0 e y 0 se obtiene ax − 1 ax − 1 2 2 Imponiendo ahora la condición x0 + y 0 = 1. Por las condiciones del problema. En efecto. Segunda solución: Tomamos r = 1 y unos ejes de coordenadas en los que la ecuación de la circunferencia es x 2 + y 2 = 1. se llega a ⎛ x 0 + a y 0 (1 − a 2 ) Q⎜ ⎜ 1+ x a . dados dos puntos A. Denotando por x. Sea entonces ahora R el punto donde BC corta a PP´ (que es perpendicular a BC ). Como CP´⊥ PP´⊥ AP. . P ' S = AC = 2r. se tiene que AS es paralelo a CP´ y ASP´C es un paralelogramo. es decir. con A y B como focos. luego R está en la circunferencia y es distinto de P . Este punto R satisface PR 2 = BP 2 − BR 2 = AR 2 − AP 2 . con lo que R = P´. se tiene AP 2 + BP´2 −( AP − BP´) 2 k − PP´2 −( AP − BP´)2 k − AB 2 . Finalmente. r2 − d 2 = − 2d 2 = 2 2 con lo que el segundo punto S en el que AP corta a la circunferencia es tal que AS = BP´= CP´. el lugar de Q es la elipse interiormente tangente a la circunferencia dada. Como AQ + BQ = AC = 2 r . la potencia de A respecto de la circunferencia es k k − AB 2 . Además. PP´2 + PS 2 = AP´2 + AS 2 + 2 ⋅ AS ⋅ AP = AP´2 + BP´2 + k − AB 2 = 2 k − 4 d 2 = 4 r 2 . AP ⋅ BP´= = = 2 2 2 donde k = AP 2 + BP 2 = AP´2 + BP´2 . Ahora bien. u Determina el menor entero n tal que f (n) = 2010. ı • Problema 3 Sea ABCD un cuadril´tero convexo.03 = 13. As´ 1. respectivamente. BC.001 = −3. Sean E. El angulo a o ´ AP D = 60◦ . F . cuya suma es un cubo perfecto. NOTA: La parte entera de un n´mero real x.98 = 1. u El tiempo de cada sesi´n es de tres horas y media. G y H los puntos medios de los lados AB. CD y DA respectivamente. ¿Cu´ntas sucesiones pucelanas tienen a solamente n´meros de tres cifras? u • Problema 2 Sean N0 y Z el conjunto de todos los enteros no negativos y el conjunto de todos los o enteros.Olimpiada Matemática Española RSME XLVI Olimpiada Matem´tica Espa˜ola a n Fase nacional 2010 (Valladolid) Primera sesi´n (26 de marzo) o • Problema 1 Una sucesi´n pucelana es una sucesi´n creciente de diecis´is n´meros impares positivos o o e u consecutivos. o . −2. a Cada problema se punt´a sobre siete puntos. Halla el mayor n´mero real positivo k tal que u EG + 3HF ≥ kd + (1 − k)s siendo s el semiper´ ımetro del cuadril´tero ABCD y d la suma de las longitudes de sus a diagonales. denotada por x es el mayor entero que u no supera a x. Sea P la intersecci´n de AC y BD. Sea f : N0 → Z la funci´n que a cada elemento n de N0 le asocia como imagen el entero f (n) definido por f (n) = −f n 3 −3 n 3 donde x es la parte entera del n´mero real x y {x} = x − x su parte decimal. ¿Cu´ndo se alcanza la igualdad? a No est´ permitido el uso de calculadoras. 7π − 8. u El tiempo de cada sesi´n es de tres horas y media. AB. CA. c tres n´meros reales positivos. b. y sean A . a Cada problema se punt´a sobre siete puntos. C puntos respectivos de las rectas BC. x2 . con x1 . • Problema 6 Sea p un n´mero primo y A un subconjunto infinito de los n´meros naturales.Olimpiada Matemática Española RSME XLVI Olimpiada Matem´tica Espa˜ola a n Fase nacional 2010 (Valladolid) Segunda sesi´n (27 de marzo) o • Problema 4 Sean a. u o ¿Existe alg´n n´mero natural N tal que fA (n) sea constante para todo n > N ? u u No est´ permitido el uso de calculadoras. Demuestra que u a + 3b + c 3a + b + c 15 a + b + 3c + + ≥ 3a + 3b + 2c 3a + 2b + 3c 2a + 3b + 3c 8 • Problema 5 Sea P un punto cualquiera de la bisectriz del angulo A en el tri´ngulo ABC. P B es perpendicular a CA y P C es perpendicular a AB. . o . xp ∈ A. Demuestra que P A y B C se cortan sobre la mediana AM . Sea fA (n) el u u n´mero de soluciones distintas de la ecuaci´n x1 +x2 +· · ·+xp = n. . siendo M el punto medio de BC. . ´ a B . . tales que P A es perpendicular a BC. 3t + 2 = bk .98 = 1. Determina u el menor entero n tal que f (n) = 2010. . n + 2. entonces a o k k f (n) = j=0 j impar aj − j=0 j par aj En efecto. Por lo tanto hay exactamente tres sucesiones pucelanas. b0 3 . 485 y 849. a0 )3 . 1000 y 1728. si n = (ak ak−1 . b0 2 3 . b0 0 3 . . Los cubos pares entre 232 y 1968 son 512. o respectivamente. • Problema 2 Sean N0 y Z el conjunto de todos los enteros no negativos y el conjunto de todos los enteros. b0 1 3 . . ¿Cu´ntas sucesiones pucelanas tienen solamente a n´meros de tres cifras? u Soluci´n: o Sea la sucesi´n n. .001 = −3. . que corresponden a valores de n de 241. 26 y 27 de Marzo) Soluciones oficiales • Problema 1 Una sucesi´n pucelana es una sucesi´n creciente de diecis´is n´meros impares positivos cono o e u secutivos. −2. n + 30. . Entonces la suma es 1 16(2n + 30) = 8(2n + 30). . 3t = bk . Por o 2 tanto. Por lo tanto. 7π − 8. Supongamos que. Entonces. ı Soluci´n: o Se prueba f´cilmente por inducci´n que. . .03 = 13. para todo n menor que 3t. f (1) = −1. NOTA: La parte entera de un n´mero real x. . f (n) = k j=0 j impar aj − k j=0 j par aj . 3t + 1 = bk . si t = bk . Ahora hay que contar el n´mero de u tales n que son impares y verifican 101 ≤ n ≤ 969. f (2) = −2. Sea f : N0 → Z la funci´n que a cada elemento n de N0 le asocia como imagen el entero f (n) definido por f (n) = −f n 3 −3 n 3 donde x es la parte entera del n´mero real x y {x} = x − x su parte decimal. f (0) = 0. . . cuya suma es un cubo perfecto. .Olimpiada Matemática Española RSME XLVI Olimpiada Matem´tica Espa˜ola a n Fase nacional 2010 (Valladolid. . denotada por x es el mayor entero que no u supera a x. . As´ 1. es necesario que 2n + 30 sea un cubo perfecto. . Por otro lado. Sean E.como f (3t) = −f (t). f (3t + 2) = −f (t) − 2. se obtiene que: WP = PA + PB . ABCD es un rect´ngulo. f (n) = j impar j par u De esta forma. CD y DA respectivamente. X. Sea P la intersecci´n de AC y BD. F . se obtiene el menor n = 2020 . D Z H A W P E G Y B F X C Sean W . Luego. 20 3 . . f (3t + 1) = −f (t) − 1. a0 )3 . BC. por el teorema de Ptolomeo. Los cuadril´teros W AP B. Halla el mayor n´mero real positivo k tal que u EG + 3HF ≥ kd + (1 − k)s siendo s el semiper´ ımetro del cuadril´tero ABCD y d la suma de las longitudes de sus a diagonales. aj − Luego. XBP C. Este n´mero contiene 1005 doses. su valor en base decimal es: 3· 32010 − 1 4 • Problema 3 Sea ABCD un cuadril´tero convexo. √ ZP 3 = P D + P A . . tal que f (n) = 2010. . y s´lo si. El angulo a o ´ ◦ AP D = 60 . k k j=0 j=0 aj . Y CP D y ZDP A son c´ a ıclicos. el tri´ngulo BCX sea is´sceles en X. Y P = PC + PD . Y y Z cuatro puntos exteriores a ABCD de modo que los tri´ngulos ABW y a DCY sean equil´teros. ¿Cu´ndo se alcanza la igualdad? a Soluci´n: o √ o a Probaremos que k = 1 + 3 y que la igualdad se da si. W P Y = W P B + 60◦ + CP Y = W AB + 60◦ + CDY = 180◦ √ XP 3 = P B + P C . G y H los puntos medios de los lados AB. la propiedad sigue siendo cierta para todo entero n menor que 3t + 2. para todo n = (ak ak−1 . el tri´ngulo AZD sea is´sceles a a o a o en Z y BXC = AZD = 120◦ . y s´lo si. ABCD a o es un rect´ngulo. lo que implica que k ≥ l. a o a F . BC y AD tambi´n deben ser a e paralelas. Y est´n alineados y. por otro lado. b. c tres n´meros reales positivos. X. Demuestra que u a + 3b + c 3a + b + c 15 a + b + 3c + + ≥ 3a + 3b + 2c 3a + 2b + 3c 2a + 3b + 3c 8 . La igualdad se dar´ si. ABCD es un rect´ngulo. entonces se da la igualdad. P . Luego: a a a W Y = W P + P Y = P A + P B + P C + P D = AC + BD 1 1 XZ = XP + P Z = √ (P B + P C + P D + P A) = √ (AC + BD) 3 3 Por la desigualdad. luego ABCD debe ser un paralelogramo. de forma an´loga. Como W E es perpendicular a AB y GY es perpendicular e a a DC. W . si k = 1 + 3. P . por un lado. Luego. E. o a • Problema 4 Sean a. EG + 3F H ≥ 1 + √ √ 3 d−s 3 √ + EG + DC 3 2 √ 3 2 √ Luego. lo que implica que ABCD es un rect´ngulo y se comprueba f´cilmente que si ABCD a a es un rect´ngulo. la igualdad se da si. la recta EG es perpendicular a a DC. Adem´s. Y est´n alineados y. y s´lo si. Luego el n´mero u √ real buscado es k = 1 + 3 y la igualdad se da si. sumando. Luego: √ 3 3 + EG + DC .Luego W . a Ahora. AC + BD ≤ AB 2 2 √ Por lo tanto. AB y DC deben ser paralelas y. Z. Entonces. si ABCD u es un rect´ngulo. entonces EG + 3F H ≥ kd + (1 − k)s. Z tambi´n est´n alineados. AC + BD ≤ AB 3(AC + BD) ≤ BC √ 3 2 √ 3 2 √ XZ ≤ XF + F H + HZ 1 BC AD √ (AC + BD) ≤ √ + F H + √ 3 2 3 2 3 + 3F H + AD √ √ 1 + 3 (AC + BD) ≤ EG + 3F H + s 3 o sea. y s´lo si. G. de forma an´loga. H. sea un n´mero real positivo l tal que EG+3HF ≥ ld +(1 −l)s. X est´n alineados. triangular: W Y ≤ W E + EG + GY . a kd + (1 − k)s ≥ ld + (1 − l)s o sea k(d − s) ≥ l(d − s) Pero la desigualdad triangular implica que d > s. y (3s − xk ) k=1 k=1 63 1 ≥ 3s − xk 8 de donde resulta que S= 15 s + 2x1 s + 2x2 s + 2x3 63 −6= + + ≥ 3s − x1 3s − x2 3s − x3 8 8 La igualdad tiene lugar cuando x1 = x2 = x3 . • Problema 5 Sea P un punto cualquiera de la bisectriz del angulo A en el tri´ngulo ABC. Si P se mueve sobre la bisectriz AI (I es el o e ı incentro). la recta B C es la recta de Simson de F . cuando a = b = c. resulta que el lado izquierdo de la desigualdad se escribe como S= Por otro lado. x2 . La bisectriz AI corta a la circunferencia circunscrita a ABC en F . si P = F . c = x3 y llamando s = x1 + x2 + x3 . . . tales que P A es perpendicular a BC. . . luego el lugar geom´trico de E es la mediana AAm . B . entonces s = S+6 = 7 8 3 1 8 3 k=1 3 (3s − xk ). 3 s + 2x2 s + 2x3 s + 2x1 + + 3s − x1 3s − x2 3s − x3 3 S+6 = k=1 s + 2xk +2 3s − xk = k=1 7s 3s − xk 3 k=1 con lo que 7s 7s 7s + + = 7s S+6 = 3s − x1 3s − x2 3s − x3 Dado que 3 k=1 1 3s − xk (3s − xk ) = 8s. u o ¿Existe alg´n n´mero natural N tal que fA (n) sea constante para todo n > N ? u u Soluci´n: o . Demuestra que P A y B C se cortan sobre la mediana AM . que se proyecta en el punto medio Am de BC. con x1 . CA. Sea fA (n) el u u n´mero de soluciones distintas de la ecuaci´n x1 + x2 + · · · + xp = n. AB. P B es perpendicular a CA y P C es perpendicular a AB. b = x2 . y sean A . xp ∈ A. es decir. e • Problema 6 Sea p un n´mero primo y A un subconjunto infinito de los n´meros naturales. siendo M el punto medio de BC.Soluci´n: o Haciendo a = x1 . Luego E describe una recta que pasa por A. ´ a C puntos respectivos de las rectas BC. Soluci´n: o Sea E el punto de intersecci´n de P A y B C . la figura P B C E es homot´tica de s´ misma con respecto al punto A. . . Sea S = (s1 .Para demostrar el enunciado procederemos por contradicci´n. el n´mero de o u p! . . . puesto que el cociente de factoriales es un n´mero natural y en o u el denominador no hay ning´n t´rmino m´ltiplo de p. s2 . En otras palabras. p o o Por lo tanto fA (pa) ≡ 1 (m´d p): la soluci´n (a. . Por otro lado no existen soluciones de x1 +x2 +· · ·+xp = pa +1 u 1 u ıa con todas las xi iguales (su valor tendr´ que ser a + ). . En particular. Por hip´tesis. Supongamos que existe un o n´mero N para el que se cumpla la propiedad anterior. sp ∈ A Entonces. . . sp ) soluci´n de la ecuaci´n x1 + x 2 + · · · + x p = n . Diremos que una soluci´n S = o o (s1 . s2 . . y as´ de manera sucesiva hasta llegar a qr1 +r2 +···+rk = qp . . . sp ) una soluci´n o o del problema y sea Q = (q1 . . . Q se obtiene a partir de S agrupando los valores si que son iguales. . r1 + r2 + · · · + rk = p. s2 . . . u tomemos a ∈ A mayor que N . Vamos a estudiar el valor de fA (pa) y fA (pa + 1). s2 . qp ) una soluci´n asociada a S con la propiedad que ı q1 = q2 = · · · = qr1 = qr1 +1 = qr1 +2 = · · · qr1 +r2 . Con esta notaci´n. soluciones asociadas a S (contando tambi´n S) es igual a e r1 !r2 ! · · · rk ! Obs´rvese que si todos los ri son estrictamente menores que p. . . . . . p . se cumple que fA (pa) = fA (pa + 1). . a) es soluci´n de x1 + x2 + · · · + xp = pa. . u e u a Ya tenemos todas las herramientas que necesit´bamos. . sp ) es asociada a una soluci´n S = s1 . o o o Sea S = (s1 . . . entonces dicha expresi´n es e o congruente con 0 m´dulo p. Volviendo al problema original. sp si la primera se obtiene a partir de la segunda mediante permutaci´n de ´ o ındices. a) no se asocia a ninguna otra. . a. . Como el conjunto A es infinito. . . cualquier permutaci´n de los ´ o ındices da lugar a una nueva soluci´n de la ecuaci´n o o (posiblemente repetida si se permutan valores iguales). . a. q2 . o observar que (a. mientras que cualquier otra soluci´n de la ecuaci´n x1 + x2 + · · · + xp = pa tiene un n´mero o o u m´ltiplo de p de asociadas. s 1 . con lo que seg´n lo anterior p o o fA (pa + 1) ≡ 0 (m´d p) y llegamos a una contradicci´n. . s 2 .
Copyright © 2024 DOKUMEN.SITE Inc.